diff --git "a/data/es/es_test_casimedicos.jsonl" "b/data/es/es_test_casimedicos.jsonl" new file mode 100644--- /dev/null +++ "b/data/es/es_test_casimedicos.jsonl" @@ -0,0 +1,125 @@ +{"id": 190, "year": 2013, "question_id_specific": 94, "full_question": "Ni\u00f1o de 18 meses de edad, con calendario vacunal completo hasta la fecha, que consulta en el Servicio de urgencias por tumefacci\u00f3n de la rodilla derecha tras jugar en el parque, sin traumatismo evidente. En la anamnesis dirigida, la madre refiere que un t\u00edo de ella ten\u00eda problemas similares. La exploraci\u00f3n ecogr\u00e1fica es compatible con hemartros y en la anal\u00edtica que se realiza s\u00f3lo destaca un alargamiento del APTT de 52\u2019\u2019 (normal 25-35\u2019\u2019). \u00bfCu\u00e1l es la hip\u00f3tesis diagn\u00f3stica m\u00e1s probable?", "full_answer": "Aunque otras enfermedades hemorr\u00e1gicas pueden tener el tiempo de tromboplastina alargado, por la intensidad de la lesi\u00f3n y por el sexo del ni\u00f1o y los antecedentes familiares, el diagn\u00f3stico m\u00e1s probable es el de hemofilia.", "type": "PEDIATR\u00cdA", "options": {"1": "S\u00edndrome de Marfan.", "2": "Enfermedad de Von-Willebrand.", "3": "Enfermedad de Ehlers-Danlos.", "4": "Hemofilia A.", "5": "Enfermedad de Bernard-Soulier."}, "correct_option": 4, "explanations": {"1": {"exist": false, "char_ranges": [], "word_ranges": [], "text": ""}, "2": {"exist": false, "char_ranges": [], "word_ranges": [], "text": ""}, "3": {"exist": false, "char_ranges": [], "word_ranges": [], "text": ""}, "4": {"exist": true, "char_ranges": [[0, 222]], "word_ranges": [[0, 35]], "text": "Aunque otras enfermedades hemorr\u00e1gicas pueden tener el tiempo de tromboplastina alargado, por la intensidad de la lesi\u00f3n y por el sexo del ni\u00f1o y los antecedentes familiares, el diagn\u00f3stico m\u00e1s probable es el de hemofilia."}, "5": {"exist": false, "char_ranges": [], "word_ranges": [], "text": ""}}} +{"id": 150, "year": 2012, "question_id_specific": 201, "full_question": "Nos consulta una mujer de 84 a\u00f1os, por insomnio de conciliaci\u00f3n. Tras fracasar a medidas de higuiene del sue\u00f1o, se decide iniciar tratamiento farmacol\u00f3gico. \u00bfCu\u00e1l de los siguientes f\u00e1rmacos selecionar\u00eda para la paciente?", "full_answer": "Sabiendo que hemos tomado medidas de higiene del sue\u00f1o, hemos explorado que le provoca el no dormir, y no duerme nueve horas y a\u00fan quiere dormir m\u00e1s,..entonces la respuesta menos lesiva es la 2. \u00bfPor qu\u00e9? Porque es el de vida media m\u00e1s corta y no quiere que la se\u00f1ora se caiga al d\u00eda siguiente.", "type": "PSIQUIATR\u00cdA", "options": {"1": "Diacepam.", "2": "Lormetacepam.", "3": "Fenobarbital.", "4": "Clordiacep\u00f3xido.", "5": "Cloracepato."}, "correct_option": 2, "explanations": {"1": {"exist": false, "char_ranges": [], "word_ranges": [], "text": ""}, "2": {"exist": true, "char_ranges": [[160, 294]], "word_ranges": [[27, 55]], "text": "la respuesta menos lesiva es la 2. \u00bfPor qu\u00e9? Porque es el de vida media m\u00e1s corta y no quiere que la se\u00f1ora se caiga al d\u00eda siguiente."}, "3": {"exist": false, "char_ranges": [], "word_ranges": [], "text": ""}, "4": {"exist": false, "char_ranges": [], "word_ranges": [], "text": ""}, "5": {"exist": false, "char_ranges": [], "word_ranges": [], "text": ""}}} +{"id": 454, "year": 2018, "question_id_specific": 153, "full_question": "Una mujer de 65 a\u00f1os consulta por debilidad en la mano derecha que se ha extendido en pocos meses a otros territorios musculares de ambos brazos y piernas, de predominio distal. En la exploraci\u00f3n hay atrofia y fasciculaciones en distintos territorios metam\u00e9ricos con sensibilidad conservada. Existe un signo de Babinski bilateral. \u00bfCual es la prueba diagn\u00f3stica que confirmar\u00eda el diagn\u00f3stico de sospecha?", "full_answer": "La respuesta es la 2, un EMG, que ser\u00eda la prueba diagn\u00f3stica para confirmar la sospecha diagn\u00f3stica de ELA, las pruebas de imagen tanto el TC, pero sobre todo la RM nos ayudar\u00edan a descartar, pero no nos confirmar\u00edan el diagn\u00f3stico.", "type": "NEUROLOG\u00cdA", "options": {"1": "TC cerebral.", "2": "Estudio electromiografico.", "3": "RM cerebral.", "4": "Potenciales evocados multimodales.", "5": null}, "correct_option": 2, "explanations": {"1": {"exist": true, "char_ranges": [[110, 233]], "word_ranges": [[19, 41]], "text": "las pruebas de imagen tanto el TC, pero sobre todo la RM nos ayudar\u00edan a descartar, pero no nos confirmar\u00edan el diagn\u00f3stico."}, "2": {"exist": true, "char_ranges": [[0, 107]], "word_ranges": [[0, 19]], "text": "La respuesta es la 2, un EMG, que ser\u00eda la prueba diagn\u00f3stica para confirmar la sospecha diagn\u00f3stica de ELA,"}, "3": {"exist": true, "char_ranges": [[109, 233]], "word_ranges": [[19, 41]], "text": "las pruebas de imagen tanto el TC, pero sobre todo la RM nos ayudar\u00edan a descartar, pero no nos confirmar\u00edan el diagn\u00f3stico."}, "4": {"exist": true, "char_ranges": [[109, 233]], "word_ranges": [[19, 41]], "text": "las pruebas de imagen tanto el TC, pero sobre todo la RM nos ayudar\u00edan a descartar, pero no nos confirmar\u00edan el diagn\u00f3stico."}, "5": {"exist": false, "char_ranges": [], "word_ranges": [], "text": ""}}} +{"id": 541, "year": 2021, "question_id_specific": 5, "full_question": "Paciente de 25 a\u00f1os de edad, que sufre un accidente de moto un viernes por la noche. Es trasladado a urgencias y diagnosticado de traumatismo abdominal (eco-fast negativa), un traumatismo craneal leve (Glasgow = 14) y una fractura intracapsular desplazada de cadera derecha. Hemodin\u00e1micamente est\u00e1 estable. \u00bfCu\u00e1l ser\u00eda el tratamiento de elecci\u00f3n?:", "full_answer": "El manejo de la fractura intracapsular desplazada de cadera en un paciente joven es quir\u00fargico y debe hacerse precozmente para reducir el riesgo de necrosis avascular de la cadera. No nos est\u00e1n hablando de riesgo vital como para aplazar un acto quir\u00fargico necesario o como para plantearnos un fijador externo siguiendo una pol\u00edtica de contenci\u00f3n de da\u00f1o por lo que descartamos la opci\u00f3n 3. Se trata de un paciente de 25 a\u00f1os de modo que tenemos que perseguir la reducci\u00f3n y fijaci\u00f3n evitando las opciones de reemplazo (descartamos opci\u00f3n 2 y 4).", "type": "TRAUMATOLOG\u00cdA", "options": {"1": "Reducci\u00f3n, abierta si se precisa, y osteos\u00edntesis de la fractura en las primeras 24-36 horas.", "2": "Artroplastia total de superficializaci\u00f3n de la cadera el lunes de forma programada.", "3": "Esperar a que mejore del traumatismo craneal y programar la semana siguiente una cirug\u00eda reglada consistente en la reducci\u00f3n y osteos\u00edntesis de la fractura.", "4": "Ante el riesgo de no consolidaci\u00f3n de estos tipos de fractura le implantar\u00eda de urgencias una hemiartroplastia bipolar de cadera.", "5": null}, "correct_option": 1, "explanations": {"1": {"exist": true, "char_ranges": [[0, 180]], "word_ranges": [[0, 29]], "text": "El manejo de la fractura intracapsular desplazada de cadera en un paciente joven es quir\u00fargico y debe hacerse precozmente para reducir el riesgo de necrosis avascular de la cadera."}, "2": {"exist": true, "char_ranges": [[390, 545]], "word_ranges": [[64, 92]], "text": "Se trata de un paciente de 25 a\u00f1os de modo que tenemos que perseguir la reducci\u00f3n y fijaci\u00f3n evitando las opciones de reemplazo (descartamos opci\u00f3n 2 y 4)."}, "3": {"exist": true, "char_ranges": [[181, 389]], "word_ranges": [[29, 64]], "text": "No nos est\u00e1n hablando de riesgo vital como para aplazar un acto quir\u00fargico necesario o como para plantearnos un fijador externo siguiendo una pol\u00edtica de contenci\u00f3n de da\u00f1o por lo que descartamos la opci\u00f3n 3."}, "4": {"exist": true, "char_ranges": [[390, 545]], "word_ranges": [[64, 92]], "text": "Se trata de un paciente de 25 a\u00f1os de modo que tenemos que perseguir la reducci\u00f3n y fijaci\u00f3n evitando las opciones de reemplazo (descartamos opci\u00f3n 2 y 4)."}, "5": {"exist": false, "char_ranges": [], "word_ranges": [], "text": ""}}} +{"id": 602, "year": 2022, "question_id_specific": 112, "full_question": "Mujer de 61 a\u00f1os, administrativa, con antecedentes de sobrepeso, hipertensi\u00f3n, dislipemia y s\u00edndrome metab\u00f3lico, que consulta por dolor en ambas nalgas, regi\u00f3n trocant\u00e9rea izquierda, cara lateral del muslo izquierdo hasta la rodilla y pierna izquierda hasta el tercio medio. El dolor aparece cuando levantamos el miembro inferior con la rodilla extendida, pero se alivia al flexionar la rodilla. \u00bfCu\u00e1l es la primera sospecha cl\u00ednica?:", "full_answer": "En este caso al hablar de \u201creaparici\u00f3n de la cl\u00ednica cuando levantamos el miembro inferior con la rodilla extendida\u201d nos est\u00e1n describiendo la Maniobra de Las\u00e8gue. Dicha maniobra es positiva en los casos de lumbociatalgia, ya que provoca un estiramiento del nervio ci\u00e1tico por lo tanto la respuesta correcta es la n\u00famero 3. El resto de patolog\u00edas no cursan con una maniobra de Las\u00e8gue positiva, por lo que dichas opciones no ser\u00edan correctas.", "type": "TRAUMATOLOG\u00cdA", "options": {"1": "Artritis gotosa de cadera izquierda.", "2": "Artrosis coxofemoral izquierda.", "3": "Lumbalgia irradiada / lumbociatalgia.", "4": "Claudicaci\u00f3n por estenosis de canal.", "5": null}, "correct_option": 3, "explanations": {"1": {"exist": true, "char_ranges": [[324, 442]], "word_ranges": [[53, 73]], "text": "El resto de patolog\u00edas no cursan con una maniobra de Las\u00e8gue positiva, por lo que dichas opciones no ser\u00edan correctas."}, "2": {"exist": true, "char_ranges": [[324, 442]], "word_ranges": [[53, 73]], "text": "El resto de patolog\u00edas no cursan con una maniobra de Las\u00e8gue positiva, por lo que dichas opciones no ser\u00edan correctas."}, "3": {"exist": true, "char_ranges": [[0, 323]], "word_ranges": [[0, 53]], "text": "En este caso al hablar de \u201creaparici\u00f3n de la cl\u00ednica cuando levantamos el miembro inferior con la rodilla extendida\u201d nos est\u00e1n describiendo la Maniobra de Las\u00e8gue. Dicha maniobra es positiva en los casos de lumbociatalgia, ya que provoca un estiramiento del nervio ci\u00e1tico por lo tanto la respuesta correcta es la n\u00famero 3."}, "4": {"exist": true, "char_ranges": [[324, 442]], "word_ranges": [[53, 73]], "text": "El resto de patolog\u00edas no cursan con una maniobra de Las\u00e8gue positiva, por lo que dichas opciones no ser\u00edan correctas."}, "5": {"exist": false, "char_ranges": [], "word_ranges": [], "text": ""}}} +{"id": 127, "year": 2012, "question_id_specific": 77, "full_question": "Un paciente de 32 a\u00f1os, diab\u00e9tico en tratamiento con insulina y un buen control de sus cifras de glucemia, acude a su consulta por presentar hormigueo en ambas manos, con sensaci\u00f3n de acorchamiento e insensibilidad t\u00e9rmica de instauraci\u00f3n progresiva en el curso de 2 semanas. No refiere trastornos visuales, d\u00e9ficit de fuerza, torpeza motora, ni otros s\u00edntomas. En la exploraci\u00f3n encuentra una anestesia para el dolor y la temperatura en ambas manos y porci\u00f3n distal de antebrazos; la sensibilidad posicional y la vibratoria est\u00e1n conservadas. No presenta atrofia muscular ni d\u00e9ficit de fuerza. Los reflejos musculares son normales y sim\u00e9tricos. No se observa dismetr\u00eda, disdiadococinesia ni temblor intencional. El resto de la exploraci\u00f3n neurol\u00f3gica es rigurosamiente normal. Indique cu\u00e1l es el diagn\u00f3stico m\u00e1s probable en este caso:", "full_answer": "La pregunta pu\u00f1etera de la secci\u00f3n de Neurolog\u00eda de este a\u00f1o. Presenta un paciente diab\u00e9tico pero a tratamiento insul\u00ednico con buen control (aunque las neuropat\u00edas diab\u00e9ticas no s\u00f3lo se desarrollan en pacientes con mal control) con un cuadro sensitivo limitado a miembros superiores y de corto tiempo de instauraci\u00f3n, sin cl\u00ednica motora ni a otro nivel. En la exploraci\u00f3n hay una clara disociaci\u00f3n con anestesia termoalg\u00e9sica y conservaci\u00f3n de artrocin\u00e9tica y vibratoria. Los reflejos son normales, ni abolidos ni exaltados. Y adem\u00e1s el resto de exploraci\u00f3n es rigurosamente normal. Con todo ello creo que la respuesta correcta es la 5, o sea una lesi\u00f3n siringomi\u00e9lica, cuya caracter\u00edstica inicial es la disociaci\u00f3n sensitiva con anestesia para la termoalg\u00e9sica y conservaci\u00f3n de la cordonal posterior. La EM y resto de lesiones medulares se acompa\u00f1ar\u00edan de otros signos exploratorios como exaltaci\u00f3n de los reflejos, para el t\u00fanel carpiano la exploraci\u00f3n excede el territorio distal al carpo, y en cuanto a la respuesta 1, si bien existen formas de polineuropat\u00eda perif\u00e9rica \u201cpseudosiringomi\u00e9licas\u201d suelen ser de inicio lumbar. El tiempo de evoluci\u00f3n tan corto (2 semanas), el respeto de miembros inferiores y la conservaci\u00f3n de los reflejos musculares hacen que descarte esta respuesta a pesar de que la distribuci\u00f3n sea \u201cen guante\u201d.", "type": "NEUROLOG\u00cdA Y NEUROCIRUG\u00cdA", "options": {"1": "Neuropat\u00eda perif\u00e9rica sensitvo sim\u00e9trica distal de causa diab\u00e9tica.", "2": "Lesi\u00f3n compresiva medular cervical.", "3": "S\u00edndrome del t\u00fanel del carpo bilateral.", "4": "Enfermedad desmielinizante tipo esclerosis m\u00faltiple.", "5": "Lesi\u00f3n medular central cervical."}, "correct_option": 5, "explanations": {"1": {"exist": true, "char_ranges": [[1024, 1128]], "word_ranges": [[160, 173]], "text": "si bien existen formas de polineuropat\u00eda perif\u00e9rica \u201cpseudosiringomi\u00e9licas\u201d suelen ser de inicio lumbar."}, "2": {"exist": true, "char_ranges": [[803, 916]], "word_ranges": [[123, 141]], "text": "La EM y resto de lesiones medulares se acompa\u00f1ar\u00edan de otros signos exploratorios como exaltaci\u00f3n de los reflejos,"}, "3": {"exist": true, "char_ranges": [[918, 992]], "word_ranges": [[141, 153]], "text": "para el t\u00fanel carpiano la exploraci\u00f3n excede el territorio distal al carpo,"}, "4": {"exist": true, "char_ranges": [[803, 916]], "word_ranges": [[123, 141]], "text": "La EM y resto de lesiones medulares se acompa\u00f1ar\u00edan de otros signos exploratorios como exaltaci\u00f3n de los reflejos,"}, "5": {"exist": true, "char_ranges": [[597, 802]], "word_ranges": [[92, 123]], "text": "creo que la respuesta correcta es la 5, o sea una lesi\u00f3n siringomi\u00e9lica, cuya caracter\u00edstica inicial es la disociaci\u00f3n sensitiva con anestesia para la termoalg\u00e9sica y conservaci\u00f3n de la cordonal posterior."}}} +{"id": 316, "year": 2016, "question_id_specific": 138, "full_question": "Un hombre de 37 a\u00f1os presenta un cuadro de artritis de las metacarpofal\u00e1ngicas de ambas manos y una pleuritis derecha. En la exploraci\u00f3n se aprecia eritema malar bilateral. Se detectan anticuerpos antinucleares positivos (titulo 1/640), con anticuerpos anti-DNA nativo tambi\u00e9n positivos; anti-Sm negativos \u00bfCu\u00e1l ser\u00eda el tratamiento inicial de elecci\u00f3n de este paciente?", "full_answer": "Es un LES con afectaci\u00f3n articular, cut\u00e1nea y serositis. El tratamiento son CE a moderadas-altas dosis. El micofenolato no es nunca un tratamiento en fase aguda sino en el mantenimiento, ni ha demostrado utilidad en afectaci\u00f3n articular.", "type": "REUMATOLOG\u00cdA", "options": {"1": "Glucocorticoides a altas dosis.", "2": "Glucocorticoides y micofenolato.", "3": "Antiinflamatorios no esteroideos y antipal\u00fadicos.", "4": "El cuadro probablemente ser\u00e1 autolimitado y no precisa tratamiento.", "5": null}, "correct_option": 1, "explanations": {"1": {"exist": true, "char_ranges": [[0, 103]], "word_ranges": [[0, 16]], "text": "Es un LES con afectaci\u00f3n articular, cut\u00e1nea y serositis. El tratamiento son CE a moderadas-altas dosis."}, "2": {"exist": true, "char_ranges": [[104, 237]], "word_ranges": [[16, 37]], "text": "El micofenolato no es nunca un tratamiento en fase aguda sino en el mantenimiento, ni ha demostrado utilidad en afectaci\u00f3n articular."}, "3": {"exist": false, "char_ranges": [], "word_ranges": [], "text": ""}, "4": {"exist": false, "char_ranges": [], "word_ranges": [], "text": ""}, "5": {"exist": false, "char_ranges": [], "word_ranges": [], "text": ""}}} +{"id": 153, "year": 2012, "question_id_specific": 74, "full_question": "Una mujer de 75 a\u00f1os consulta por lesiones viol\u00e1ceas en manos y cuello junto a debilidad muscular progresiva de 3 meses de evoluci\u00f3n. \u00bfQu\u00e9 pruebas diagn\u00f3sticas, entre las que se indica, pueden ser de utilidad para el diagn\u00f3stico?", "full_answer": "Ante este cuadro cl\u00ednico, una posibilidad es que se trate de una dermatomiositis. De las pruebas propuestas, la determinaci\u00f3n de la aldolasa s\u00e9rica puede ser de utilidad. Su elevaci\u00f3n es caracter\u00edstica de esta enfermedad. La biopsia deber\u00eda ser muscular y no subcut\u00e1nea. Los anticuerpos anti-m\u00fasculo liso no son los propios de esta enfermedad.", "type": "REUMATOLOG\u00cdA", "options": {"1": "Determinaci\u00f3n de aldolasa s\u00e9rica.", "2": "Electroencefalograma.", "3": "Biopsia de tejido celular subcut\u00e1neo.", "4": "Determinaci\u00f3n de anticuerpos anti-m\u00fasculo liso.", "5": "Estudio gen\u00e9tico de sus descendientes."}, "correct_option": 1, "explanations": {"1": {"exist": true, "char_ranges": [[0, 221]], "word_ranges": [[0, 34]], "text": "Ante este cuadro cl\u00ednico, una posibilidad es que se trate de una dermatomiositis. De las pruebas propuestas, la determinaci\u00f3n de la aldolasa s\u00e9rica puede ser de utilidad. Su elevaci\u00f3n es caracter\u00edstica de esta enfermedad."}, "2": {"exist": false, "char_ranges": [], "word_ranges": [], "text": ""}, "3": {"exist": true, "char_ranges": [[222, 270]], "word_ranges": [[34, 42]], "text": "La biopsia deber\u00eda ser muscular y no subcut\u00e1nea."}, "4": {"exist": true, "char_ranges": [[271, 343]], "word_ranges": [[42, 53]], "text": "Los anticuerpos anti-m\u00fasculo liso no son los propios de esta enfermedad."}, "5": {"exist": false, "char_ranges": [], "word_ranges": [], "text": ""}}} +{"id": 369, "year": 2016, "question_id_specific": 122, "full_question": "Acude a la consulta un hombre de 67 a\u00f1os, ex\u00ad fumador, con el diagn\u00f3stico de EPOC grave (\u00edndice multidimensional BODE 5, FEVl 38%, \u00edndice de masa corporal 23, \u00edndice de disnea seg\u00fan la escala mMRC 3, distancia recorrida en la prueba de los 6 minutos marcha 260 m) que ha tenido 3 ingresos hospitalarios por exacerbaci\u00f3n de su EPOC en los \u00faltimos 7 meses. Adem\u00e1s presenta antecedentes de HTA, cardiopat\u00eda isqu\u00e9mica con IAM hace 5 a\u00f1os y claudicaci\u00f3n intermitente. En la exploraci\u00f3n cl\u00ednica destaca una disminuci\u00f3n del murmullo vesicular con sibilancias espiratorias en ambos campos pulmonares y una saturaci\u00f3n en la oximetr\u00eda del 88%. \u00bfCu\u00e1l de las siguientes estrategias terap\u00e9uticas NO ser\u00eda recomendable para este paciente?:", "full_answer": "Los corticoides sist\u00e9micos en estos pacientes deben de utilizarse principalmente en las exacerbaciones agudas, no para el control de las exacerbaciones.", "type": "NEUMOLOG\u00cdA Y CIRUG\u00cdA TOR\u00c1CICA", "options": {"1": "Ajuste del tratamiento inhalado con bronco\u00addilatadores de larga duraci\u00f3n combinando anticolin\u00e9rgicos y beta-2 adren\u00e9rgicos con glucocorticoides inhalados.", "2": "Iniciar pauta de glucocorticoides orales durante 6 meses para el control de las exacerbaciones.", "3": "Comprobar que el paciente realiza correctamente la t\u00e9cnica inhalatoria.", "4": "Iniciar pauta de oxigenoterapia cr\u00f3nica domiciliaria.", "5": null}, "correct_option": 2, "explanations": {"1": {"exist": false, "char_ranges": [], "word_ranges": [], "text": ""}, "2": {"exist": true, "char_ranges": [[0, 152]], "word_ranges": [[0, 21]], "text": "Los corticoides sist\u00e9micos en estos pacientes deben de utilizarse principalmente en las exacerbaciones agudas, no para el control de las exacerbaciones."}, "3": {"exist": false, "char_ranges": [], "word_ranges": [], "text": ""}, "4": {"exist": false, "char_ranges": [], "word_ranges": [], "text": ""}, "5": {"exist": false, "char_ranges": [], "word_ranges": [], "text": ""}}} +{"id": 413, "year": 2018, "question_id_specific": 68, "full_question": "Mujer de 87 a\u00f1os con antecedentes de hipertensi\u00f3n arterial, ingresada hace 48 horas en la unidad coronaria por infarto agudo de miocardio con elevaci\u00f3n de ST de localizaci\u00f3n anterior. Avisa por disnea. En la exploraci\u00f3n destaca soplo sist\u00f3lico con fr\u00e9mito, irradiado al borde esternal derecho que no estaba presente al ingreso. \u00bfQu\u00e9 complicaci\u00f3n sospecha usted?", "full_answer": "Rotura del tabique interventricular. Las complicaciones mec\u00e1nicas son t\u00edpicas en infartos en mujeres ancianas. Asimismo, un soplo que no estaba presente indica un flujo de sangre anormal: las complicaciones m\u00e1s frecuentes son la insuficiencia mitral, que cursar\u00eda con un soplo musical o piante, y la CIV, que cursa con un soplo irradiado al borde derecho (piensa en la direcci\u00f3n del flujo de sangre).", "type": "CARDIOLOG\u00cdA Y CIRUG\u00cdA CARDIOVASCULAR", "options": {"1": "Insuficiencia cardiaca por necrosis extensa.", "2": "Aneurisma anterior.", "3": "Rotura de la pared libre del ventr\u00edculo izquierdo.", "4": "Rotura en el tabique interventricular.", "5": null}, "correct_option": 4, "explanations": {"1": {"exist": false, "char_ranges": [], "word_ranges": [], "text": ""}, "2": {"exist": false, "char_ranges": [], "word_ranges": [], "text": ""}, "3": {"exist": false, "char_ranges": [], "word_ranges": [], "text": ""}, "4": {"exist": true, "char_ranges": [[188, 400]], "word_ranges": [[27, 63]], "text": "las complicaciones m\u00e1s frecuentes son la insuficiencia mitral, que cursar\u00eda con un soplo musical o piante, y la CIV, que cursa con un soplo irradiado al borde derecho (piensa en la direcci\u00f3n del flujo de sangre)."}, "5": {"exist": false, "char_ranges": [], "word_ranges": [], "text": ""}}} +{"id": 314, "year": 2016, "question_id_specific": 178, "full_question": "Paciente de 66 a\u00f1os diagnosticado de adenocarcinoma de p\u00e1ncreas estadio IV hace 8 meses. Sigue tratamiento con morfina de liberaci\u00f3n retardada 200 mg/12 horas v\u00eda oral, parafina y lactulosa. Desde hace 15 d\u00edas refiere parestesias y dolor lancinante ocasional en zona lumbar derecha y periumbilical que no cede con ingesta de morfina de rescate. La exploraci\u00f3n neurol\u00f3gica es normal. Se\u00f1ale cual ser\u00eda el manejo m\u00e1s apropiado:", "full_answer": "Tenemos delante a un paciente con dolor neurop\u00e1tico, que por el origen del tumor, lo m\u00e1s probable es que se deba a afectaci\u00f3n del plexo celiaco. Es importante tener un diagn\u00f3stico radiol\u00f3gico de cara a un posible tratamiento local, como el tratamiento neurol\u00edtico o bloqueo del plexo. No obstante no debemos dejar al paciente sin un tratamiento que le pueda aliviar sino solucionar la situaci\u00f3n actual. Se encuentra tomando analgesia del tercer escal\u00f3n de la OMS que es claramente insuficiente, con lo que precisar\u00e1 de coadyuvantes; aunque los opi\u00e1ceos para el dolor neurop\u00e1tico no son muy eficaces, no disminuiremos la dosis sino que asociaremos los coadyuvantes. Manejar\u00edamos al paciente con gabapentina y corticoides (respuesta 4 verdadera); no le disminuir\u00edamos la dosis de morfina (respuesta 3 falsa), y mientras tanto le ir\u00edamos pidiendo el TAC (o RM abdominal) para posteriormente valorar el bloqueo del plexo (respuestas 1 y 2 para el manejo futuro).", "type": "ONCOLOG\u00cdA", "options": {"1": "Realizar tomograf\u00eda axial computarizada y evaluar compresi\u00f3n nerviosa ya que se trata de dolor neurop\u00e1tico.", "2": "Valorar tratamiento neurol\u00edtico ya que el dolor neurop\u00e1tico es de dif\u00edcil control.", "3": "Disminuir la dosis de morfina ya que es ineficaz en este tipo de dolor.", "4": "Administrar amitriptilina o gabapentina, dexametasona y aumentar la dosis de morfina.", "5": null}, "correct_option": 4, "explanations": {"1": {"exist": true, "char_ranges": [[809, 958]], "word_ranges": [[127, 152]], "text": "mientras tanto le ir\u00edamos pidiendo el TAC (o RM abdominal) para posteriormente valorar el bloqueo del plexo (respuestas 1 y 2 para el manejo futuro)."}, "2": {"exist": true, "char_ranges": [[809, 958]], "word_ranges": [[127, 152]], "text": "mientras tanto le ir\u00edamos pidiendo el TAC (o RM abdominal) para posteriormente valorar el bloqueo del plexo (respuestas 1 y 2 para el manejo futuro)."}, "3": {"exist": true, "char_ranges": [[745, 805]], "word_ranges": [[116, 126]], "text": "no le disminuir\u00edamos la dosis de morfina (respuesta 3 falsa),"}, "4": {"exist": true, "char_ranges": [[533, 744]], "word_ranges": [[86, 116]], "text": "aunque los opi\u00e1ceos para el dolor neurop\u00e1tico no son muy eficaces, no disminuiremos la dosis sino que asociaremos los coadyuvantes. Manejar\u00edamos al paciente con gabapentina y corticoides (respuesta 4 verdadera);"}, "5": {"exist": false, "char_ranges": [], "word_ranges": [], "text": ""}}} +{"id": 263, "year": 2014, "question_id_specific": 37, "full_question": "Juan tiene 60 a\u00f1os, fuma de 2 paquetes/d\u00eda desde hace a\u00f1os y refiere desde hace 6 meses tos persistente. Comprueba que su p\u00e1rpado izquierdo est\u00e1 m\u00e1s ca\u00eddo y que la pupila de ese ojo es m\u00e1s peque\u00f1a. Juan refiere que la parte medial de su mano izquierda est\u00e1 adormecida y con menos fuerza. Su m\u00e9dico comprueba la ptosis palpebral y la miosis izquierda; comprueba que Puede cerrar con fuerza ambos p\u00e1rpados sim\u00e9tricamente y que las dos pupilas responden correctamente a la luz. Adem\u00e1s comprueba que no suda por la hemicara izquierda, que siente menos el pinchazo en la superficie interna de dicha mano y que tiene menos fuerza en la prensi\u00f3n de dicha mano. Respecto a la sintomatolog\u00eda ocular, \u00bfd\u00f3nde se localiza la lesi\u00f3n?", "full_answer": "La pregunta seria mas de neurologia. Se trata de un sindrome de claude bernard horner y probablemente la correcta sea la 1. Pero por seguridad es mejor que lo revise un neurologo.", "type": "NEUMOLOG\u00cdA", "options": {"1": "Las fibras simp\u00e1ticas, en alg\u00fan nivel que abarcar\u00eda desde el hipot\u00e1lamo a la columna de Clark interinedio-lateral de la m\u00e9dula dorsal.", "2": "El nervio motor ocular com\u00fan izquierdo en el mesenc\u00e9falo.", "3": "El n\u00facleo de Edinger-Westphal, encima del n\u00facleo del nervio motor ocular com\u00fan izquierdo.", "4": "Las fibras parasimp\u00e1ticas, en alg\u00fan nivel que abarcar\u00eda desde el n\u00facleo de Edinger-Westphal hasta el m\u00fasculo constrictor de la pupila izquierda.", "5": "El m\u00fasculo tarsal exclusivamente."}, "correct_option": 1, "explanations": {"1": {"exist": true, "char_ranges": [[37, 123]], "word_ranges": [[6, 22]], "text": "Se trata de un sindrome de claude bernard horner y probablemente la correcta sea la 1."}, "2": {"exist": false, "char_ranges": [], "word_ranges": [], "text": ""}, "3": {"exist": false, "char_ranges": [], "word_ranges": [], "text": ""}, "4": {"exist": false, "char_ranges": [], "word_ranges": [], "text": ""}, "5": {"exist": false, "char_ranges": [], "word_ranges": [], "text": ""}}} +{"id": 344, "year": 2016, "question_id_specific": 158, "full_question": "Una mujer de 24 a\u00f1os, primigesta, sufre un aborto espont\u00e1neo a las 7 semanas de gestaci\u00f3n. El estudio anatomopatol\u00f3gico de los restos abortivos indica enfermedad molar. Le debemos informar que:", "full_answer": "No debe quedar embarazada hasta realizar controles peri\u00f3dicos y haber pasado un a\u00f1o con niveles de beta-HCG negativos. Las pacientes ser\u00e1n controladas semanalmente con la dosificaci\u00f3n de la hcg hasta que se convierta indetectable, por tres veces consecutivas. Despu\u00e9s el control ser\u00e1 mensual, durante seis meses y luego cada dos meses otros seis meses m\u00e1s, antes de nueva gestaci\u00f3n.", "type": "GINECOLOG\u00cdA Y OBSTETRICIA", "options": {"1": "El riesgo de una nueva gestaci\u00f3n molar en un futuro embarazo es del 50%.", "2": "No debe quedar embarazada hasta realizar controles peri\u00f3dicos y haber pasado un a\u00f1os con niveles de BHCG negativos.", "3": "No es necesario realizar controles posteriores si la evacuaci\u00f3n del tejido trofobl\u00e1stico fue completa.", "4": "Es necesario que se realice controles peri\u00f3dicos ya que en el 40% de los casos desarrollar\u00e1 una neoplasia trofobl\u00e1stica gestacional.", "5": null}, "correct_option": 2, "explanations": {"1": {"exist": false, "char_ranges": [], "word_ranges": [], "text": ""}, "2": {"exist": true, "char_ranges": [[0, 382]], "word_ranges": [[0, 59]], "text": "No debe quedar embarazada hasta realizar controles peri\u00f3dicos y haber pasado un a\u00f1o con niveles de beta-HCG negativos. Las pacientes ser\u00e1n controladas semanalmente con la dosificaci\u00f3n de la hcg hasta que se convierta indetectable, por tres veces consecutivas. Despu\u00e9s el control ser\u00e1 mensual, durante seis meses y luego cada dos meses otros seis meses m\u00e1s, antes de nueva gestaci\u00f3n."}, "3": {"exist": false, "char_ranges": [], "word_ranges": [], "text": ""}, "4": {"exist": false, "char_ranges": [], "word_ranges": [], "text": ""}, "5": {"exist": false, "char_ranges": [], "word_ranges": [], "text": ""}}} +{"id": 43, "year": 2011, "question_id_specific": 151, "full_question": "Ni\u00f1a de 10 a\u00f1os que viene a revisi\u00f3n de salud. La exploraci\u00f3n f\u00edsica es normal con un peso y una talla en el percentil 50 y TA 109/65. Un an\u00e1lisis de orina de rutina muestra una densidad espec\u00edfica de 1035 pH6 sangre 2+ sin prote\u00ednas. El sedimento urinario muestra 5-10 hemat\u00edes por campo. \u00bfCu\u00e1l ser\u00eda la conducta m\u00e1s apropiada a seguir?", "full_answer": "La respuesta correcta es la 4. Lo que no me queda muy claro es lo de \u201can\u00e1lisis de orina de rutina\u201d porque no se suelen hacer en un ni\u00f1o sano asintom\u00e1tico. La microhematuria sin s\u00edntomas asociados, puede ser un hallazgo sin significado patol\u00f3gico y hay que confirmarla en un nuevo sedimento posterior (aunque no s\u00e9 si a los 15 d\u00edas o m\u00e1s tarde).", "type": "PEDIATR\u00cdA", "options": {"1": "Determinar creatinina y nitr\u00f3geno en sangre.", "2": "Derivar a la ni\u00f1a para la realizaci\u00f3n de una cistoscopia.", "3": "Determinar anticuerpos antinucleares y complemento.", "4": "Volver a repetir el sedimento de orina en 15 d\u00edas.", "5": "Realizar una tomograf\u00eda axial computarizada abdominal."}, "correct_option": 4, "explanations": {"1": {"exist": false, "char_ranges": [], "word_ranges": [], "text": ""}, "2": {"exist": false, "char_ranges": [], "word_ranges": [], "text": ""}, "3": {"exist": false, "char_ranges": [], "word_ranges": [], "text": ""}, "4": {"exist": true, "char_ranges": [[155, 344]], "word_ranges": [[31, 63]], "text": "La microhematuria sin s\u00edntomas asociados, puede ser un hallazgo sin significado patol\u00f3gico y hay que confirmarla en un nuevo sedimento posterior (aunque no s\u00e9 si a los 15 d\u00edas o m\u00e1s tarde)."}, "5": {"exist": false, "char_ranges": [], "word_ranges": [], "text": ""}}} +{"id": 616, "year": 2022, "question_id_specific": 119, "full_question": "Mujer de 13 a\u00f1os, sin antecedentes relevantes, con menarquia hace 3 meses, seguida desde los 10 a\u00f1os por escoliosis idiop\u00e1tica que ha empeorado. En la exploraci\u00f3n f\u00edsica presenta una giba de 7 grados en el test de Adams y en el escoliograma una curva toracolumbar T4-L1 de 35 grados de Cobb y un Risser 0. La actitud correcta a tomar ser\u00e1:", "full_answer": "Ortesis. Angulo de Cobb entre 25\u00ba \u2013 45\u00ba. Esqueleto inmaduro (Risser 0).", "type": "TRAUMATOLOG\u00cdA", "options": {"1": "Recomendar nataci\u00f3n y revisi\u00f3n en tres meses.", "2": "Prescribir una ortesis tipo cors\u00e9.", "3": "Derivar a fisioterapia para elastificaci\u00f3n del raquis.", "4": "Revisar en 6 meses con una nueva radiograf\u00eda.", "5": null}, "correct_option": 2, "explanations": {"1": {"exist": false, "char_ranges": [], "word_ranges": [], "text": ""}, "2": {"exist": true, "char_ranges": [[0, 71]], "word_ranges": [[0, 12]], "text": "Ortesis. Angulo de Cobb entre 25\u00ba \u2013 45\u00ba. Esqueleto inmaduro (Risser 0)."}, "3": {"exist": false, "char_ranges": [], "word_ranges": [], "text": ""}, "4": {"exist": false, "char_ranges": [], "word_ranges": [], "text": ""}, "5": {"exist": false, "char_ranges": [], "word_ranges": [], "text": ""}}} +{"id": 489, "year": 2020, "question_id_specific": 105, "full_question": "Paciente de 67 a\u00f1os que en los \u00faltimos 6 meses, en dos anal\u00edticas de rutina, presenta linfocitosis progresiva. En la \u00faltima, hemoglobina 15,4 g/dl; leucocitos 18,5 x103/\u03bcL con 82 % de linfocitos maduros que por citometr\u00eda de flujo expresan los ant\u00edgenos CD5/CD19/CD23 y plaquetas 240 x103/\u03bcL. \u00bfQu\u00e9 actitud le parece correcta?:", "full_answer": "El paciente ya est\u00e1 diagnosticado de leucemia linf\u00e1tica cr\u00f3nica B (LLC-B), presenta linfocitosis que se mantiene mas de 3 meses y por citometr\u00eda de flujo presenta un fenotipo compatible con la misma. Por lo tanto debemos conocer primeramente los factores de riesgo que nos van a indicar con que prioridad va a necesitar iniciar tratamiento, la intensidad de los mismos y pron\u00f3stico, de esto depende mucho la presencia de mutaciones TP53 o delecciones del 17p. El aspirado de medula o biopsia s\u00f3lo esta indicado si hay otras citopenias acompa\u00f1ante que no se justifique por la propia LLC-B, el PET-TC s\u00f3lo esta indicando en caso que se sospeche un s\u00edndrome de Ritcher y no nos dan datos suficientes para sospecharlo, no hay s\u00edntomas B, ni se habla de elevaci\u00f3n de la LDH.", "type": "HEMATOLOGIA", "options": {"1": "Estudio de mutaciones de TP53 para establecer el pron\u00f3stico.", "2": "Aspirado/biopsia \u00f3sea para confirmar el diagn\u00f3stico.", "3": "PET/CT para establecer la actitud terap\u00e9utica.", "4": "Nuevo control cl\u00ednico y anal\u00edtico en 6 meses.", "5": null}, "correct_option": 4, "explanations": {"1": {"exist": false, "char_ranges": [], "word_ranges": [], "text": ""}, "2": {"exist": true, "char_ranges": [[460, 587]], "word_ranges": [[75, 97]], "text": "El aspirado de medula o biopsia s\u00f3lo esta indicado si hay otras citopenias acompa\u00f1ante que no se justifique por la propia LLC-B,"}, "3": {"exist": true, "char_ranges": [[589, 769]], "word_ranges": [[97, 131]], "text": "el PET-TC s\u00f3lo esta indicando en caso que se sospeche un s\u00edndrome de Ritcher y no nos dan datos suficientes para sospecharlo, no hay s\u00edntomas B, ni se habla de elevaci\u00f3n de la LDH."}, "4": {"exist": true, "char_ranges": [[0, 459]], "word_ranges": [[0, 75]], "text": "El paciente ya est\u00e1 diagnosticado de leucemia linf\u00e1tica cr\u00f3nica B (LLC-B), presenta linfocitosis que se mantiene mas de 3 meses y por citometr\u00eda de flujo presenta un fenotipo compatible con la misma. Por lo tanto debemos conocer primeramente los factores de riesgo que nos van a indicar con que prioridad va a necesitar iniciar tratamiento, la intensidad de los mismos y pron\u00f3stico, de esto depende mucho la presencia de mutaciones TP53 o delecciones del 17p."}, "5": {"exist": false, "char_ranges": [], "word_ranges": [], "text": ""}}} +{"id": 534, "year": 2021, "question_id_specific": 119, "full_question": "Una paciente de 45 a\u00f1os con antecedentes de reflujo gastroesof\u00e1gico presenta desde hace un a\u00f1o episodios de palidez en algunos dedos de las manos con la exposici\u00f3n al fr\u00edo. Recientemente se le hab\u00eda prescrito prednisona a dosis de 20 mg/d\u00eda por dolor articular e induraci\u00f3n cut\u00e1nea en manos y brazos. Presenta desde hace 48 horas malestar general y cefalea intensa, por lo que acude a urgencias. En la exploraci\u00f3n destaca \u00fanicamente una taquicardia r\u00edtmica a 100 lpm, no se aprecia focalidad neurol\u00f3gica. La presi\u00f3n arterial es de 200/110 mmHg. En la anal\u00edtica destaca \u00fanicamente una creatinina de 2,5 mg/dl. Indique cu\u00e1l de los siguientes autoanticuerpos se relaciona mejor con el proceso descrito:", "full_answer": "Se trata de un caso cl\u00ednico claro de esclerosis sist\u00e9mica (Raynaud, afectaci\u00f3n esof\u00e1gica, induraci\u00f3n cut\u00e1nea) que presenta una crisis renal. Quitando los Ac anti-proteinasa 3, que no se relacionan con esclerodermia (respuesta 3 incorrecta), los m\u00e1s probables ser\u00e1n los Ac antiRNA polimerasa III, que aparecen en pacientes m\u00e1s j\u00f3venes y con crisis renales.", "type": "REUMATOLOG\u00cdA", "options": {"1": "Anticuerpos anti-RNA polimerasa III.", "2": "Anticuerpos anti-centr\u00f3mero.", "3": "Anticuerpos anti-proteinasa 3.", "4": "Anticuerpos anti-PM-Scl.", "5": null}, "correct_option": 1, "explanations": {"1": {"exist": true, "char_ranges": [[0, 355]], "word_ranges": [[0, 53]], "text": "Se trata de un caso cl\u00ednico claro de esclerosis sist\u00e9mica (Raynaud, afectaci\u00f3n esof\u00e1gica, induraci\u00f3n cut\u00e1nea) que presenta una crisis renal. Quitando los Ac anti-proteinasa 3, que no se relacionan con esclerodermia (respuesta 3 incorrecta), los m\u00e1s probables ser\u00e1n los Ac antiRNA polimerasa III, que aparecen en pacientes m\u00e1s j\u00f3venes y con crisis renales."}, "2": {"exist": false, "char_ranges": [], "word_ranges": [], "text": ""}, "3": {"exist": true, "char_ranges": [[150, 239]], "word_ranges": [[21, 34]], "text": "los Ac anti-proteinasa 3, que no se relacionan con esclerodermia (respuesta 3 incorrecta),"}, "4": {"exist": false, "char_ranges": [], "word_ranges": [], "text": ""}, "5": {"exist": false, "char_ranges": [], "word_ranges": [], "text": ""}}} +{"id": 273, "year": 2016, "question_id_specific": 71, "full_question": "Joven de 18 a\u00f1os con antecedentes de asma, alergia a p\u00f3lenes, \u00e1caros y pelo de gato, acude a urgencias refiriendo sensaci\u00f3n de detenci\u00f3n de alimentos a nivel retroesternal con pr\u00e1ctica incapacidad para deglutir su propia saliva. Refiere episodios similares en otras ocasiones que han cedido de forma espont\u00e1nea en pocos minutos. \u00bfCu\u00e1l de los siguientes es el diagn\u00f3stico m\u00e1s probable?", "full_answer": "Se trata de una disfagia intermitente. El es\u00f3fago de Barrett no implica necesariamente una estenosis p\u00e9ptica, pero suponi\u00e9ndola, esta es progresiva. La esofagitis infecciosa es m\u00e1s propia de pacientes inmunodeprimidos. Quedan el anillo esof\u00e1gico distal y la esofagitis eosinof\u00edlica; ambas son posibles, pero la insistencia en la carga at\u00f3pica del paciente indica la probabilidad de que se trate de la \u00faltima de las opciones.", "type": "APARATO DIGESTIVO", "options": {"1": "Es\u00f3fago de Barrett.", "2": "Anillo esof\u00e1gico distal (Schatzki).", "3": "Esofagitis infecciosa.", "4": "Esofagitis eosinof\u00edlica.", "5": null}, "correct_option": 4, "explanations": {"1": {"exist": true, "char_ranges": [[39, 148]], "word_ranges": [[6, 21]], "text": "El es\u00f3fago de Barrett no implica necesariamente una estenosis p\u00e9ptica, pero suponi\u00e9ndola, esta es progresiva."}, "2": {"exist": true, "char_ranges": [[219, 424]], "word_ranges": [[30, 64]], "text": "Quedan el anillo esof\u00e1gico distal y la esofagitis eosinof\u00edlica; ambas son posibles, pero la insistencia en la carga at\u00f3pica del paciente indica la probabilidad de que se trate de la \u00faltima de las opciones."}, "3": {"exist": true, "char_ranges": [[149, 218]], "word_ranges": [[21, 30]], "text": "La esofagitis infecciosa es m\u00e1s propia de pacientes inmunodeprimidos."}, "4": {"exist": true, "char_ranges": [[219, 424]], "word_ranges": [[30, 64]], "text": "Quedan el anillo esof\u00e1gico distal y la esofagitis eosinof\u00edlica; ambas son posibles, pero la insistencia en la carga at\u00f3pica del paciente indica la probabilidad de que se trate de la \u00faltima de las opciones."}, "5": {"exist": false, "char_ranges": [], "word_ranges": [], "text": ""}}} +{"id": 56, "year": 2011, "question_id_specific": 76, "full_question": "Ni\u00f1a de 14 a\u00f1os que consulta por disminuci\u00f3n del crecimiento desde hace 2-3 a\u00f1os previamente normal (aporta datos) y que las dem\u00e1s ni\u00f1as de su edad tienen un mayor desarrollo f\u00edsico y sexual. \u00daltimamente tiene cefaleas y problemas visuales que nota en clase y al estudiar. No ha tenido la menarquia ni polidipsia o poliuria. Padres con talla normal. Exploraci\u00f3n: talla baja situada en -2.1 desviaciones est\u00e1ndar, proporciones corporales normales, poco vello pubiano y desarrollo mamario. La campimetr\u00eda muestra hemianopsia parcial temporal izquierda. Edad \u00f3sea: retraso de 2 a\u00f1os. Anal\u00edtica general normal. Gonadotrofinas (FSH y LH) y estradiol bajos. \u00bfCu\u00e1l le parece la respuesta m\u00e1s id\u00f3nea?", "full_answer": "La respuesta 2 es falsa (S\u00edndrome Turner: estradiol bajo y gonadotrofinas elevadas), los tumores cerebrales que afectan hipot\u00e1lamo-hip\u00f3fisis sin dan gonadotrofinas bajas (5 falsa), y parece obvio que los defectos de refracci\u00f3n no dan hemianopsia (3 y 4 falsas).", "type": "ENDOCRINOLOG\u00cdA", "options": {"1": "La disminuci\u00f3n del crecimiento y desarrollo sexual, edad \u00f3sea retrasada, cefalea y alteraci\u00f3n visual sugieren d\u00e9ficit hormonal y compromiso del quiasma \u00f3ptico.", "2": "Al ser una ni\u00f1a en edad puberal, lo m\u00e1s probable es que su disminuci\u00f3n de crecimiento y retraso sexual sean debidos a un s\u00edndrome de Turner.", "3": "No debe tener un tumor hipotal\u00e1mico por la ausencia de poliuria y polidipsia. Seguramente tenga retraso constitucional y su problema visual sea de refracci\u00f3n.", "4": "Un d\u00e9ficit de hormona de crecimiento puede explicar el retraso del desarrollo y estradiol bajo. Valorar si precisa gafas, por sus cefaleas y alteraciones visuales.", "5": "Podr\u00eda tener un craneofaringioma, pero ser\u00eda raro que no hubiera dado s\u00edntomas antes. Adem\u00e1s, no justificar\u00eda las gonadotrofinas y estradiol bajos."}, "correct_option": 1, "explanations": {"1": {"exist": false, "char_ranges": [], "word_ranges": [], "text": ""}, "2": {"exist": true, "char_ranges": [[0, 83]], "word_ranges": [[0, 12]], "text": "La respuesta 2 es falsa (S\u00edndrome Turner: estradiol bajo y gonadotrofinas elevadas),"}, "3": {"exist": true, "char_ranges": [[200, 261]], "word_ranges": [[28, 39]], "text": "los defectos de refracci\u00f3n no dan hemianopsia (3 y 4 falsas)."}, "4": {"exist": true, "char_ranges": [[200, 261]], "word_ranges": [[28, 39]], "text": "los defectos de refracci\u00f3n no dan hemianopsia (3 y 4 falsas)."}, "5": {"exist": true, "char_ranges": [[85, 179]], "word_ranges": [[12, 24]], "text": "los tumores cerebrales que afectan hipot\u00e1lamo-hip\u00f3fisis sin dan gonadotrofinas bajas (5 falsa),"}}} +{"id": 409, "year": 2018, "question_id_specific": 191, "full_question": "Acude al domicilio de un paciente oncol\u00f3gico que \u00faltimamente no tiene bien controlado el dolor. En la exploraci\u00f3n f\u00edsica parece que no hay evidencia de progresi\u00f3n tumoral, no revela ning\u00fan dato de inter\u00e9s de lo conocido previamente. En la anamnesis el cuidador principal manifiesta que el paciente tiene dolor a las 8 h de haber recibido la dosis basal pautada de morfina cada 12 h. Esta situaci\u00f3n cl\u00ednica se denomina:", "full_answer": "La respuesta correcta es la 1. El enunciado define perfectamente un fen\u00f3meno de tolerancia en el que el paciente se ha \u201cacostumbrado\u201d a la dosis y necesita m\u00e1s para poder tener el mismo efecto analg\u00e9sico.", "type": "ATENCI\u00d3N PRIMARIA", "options": {"1": "Tolerancia", "2": "Hiperalgesia", "3": "Dependencia", "4": "Ineficacia", "5": null}, "correct_option": 1, "explanations": {"1": {"exist": true, "char_ranges": [[31, 204]], "word_ranges": [[6, 35]], "text": "El enunciado define perfectamente un fen\u00f3meno de tolerancia en el que el paciente se ha \u201cacostumbrado\u201d a la dosis y necesita m\u00e1s para poder tener el mismo efecto analg\u00e9sico."}, "2": {"exist": false, "char_ranges": [], "word_ranges": [], "text": ""}, "3": {"exist": false, "char_ranges": [], "word_ranges": [], "text": ""}, "4": {"exist": false, "char_ranges": [], "word_ranges": [], "text": ""}, "5": {"exist": false, "char_ranges": [], "word_ranges": [], "text": ""}}} +{"id": 179, "year": 2013, "question_id_specific": 98, "full_question": "Una mujer de 33 a\u00f1os consulta por epistaxis de repetici\u00f3n, petequias y equimosis. Las pruebas de laboratorio muestran trombocitopenia con un recuento plaquetario de 4000 plaque\u00adtas/microlitro. El diagn\u00f3stico de presunci\u00f3n inicial es de p\u00farpura trombocitop\u00e9nica inmuni\u00adtaria cr\u00f3nica (PTI). \u00bfCu\u00e1l de las siguientes afirmaciones es FALSA respecto al diagn\u00f3stico de PTI?", "full_answer": "Ante una trombocitopenia aislada no hay que perder de vista la exploraci\u00f3n f\u00edsica; as\u00ed, la presencia de adenopat\u00edas nos obliga a descartar la presencia de un linfoma, por ejemplo. La 1 es correcta. Al realizar una biopsia-aspirado de m\u00e9dula \u00f3sea se ve que el n\u00famero de megacariocitos es normal o incluso est\u00e1 aumentado, el fallo no est\u00e1 en la m\u00e9dula sino en la sangre perif\u00e9rica, donde se destruyen las plaquetas. As\u00ed que... La respuesta falsa es la 2.", "type": "HEMATOLOG\u00cdA", "options": {"1": "La presencia de adenopat\u00edas o esplenomega\u00adlia en la exploraci\u00f3n f\u00edsica sugiere un diag\u00adn\u00f3stico diferente de PTI.", "2": "El an\u00e1lisis de m\u00e9dula \u00f3sea muestra un n\u00famero disminuido de megacariocitos sin otras alteraciones.", "3": "El hemograma completo muestra trombocitopenia aislada con plaquetas a menudo gran\u00addes, sin anemia salvo que exista una hemo\u00adrragia importante o hem\u00f3lisis autoinmune asociada (s\u00edndrome de Evans).", "4": "El diagn\u00f3stico de PTI se establece por exclu\u00adsi\u00f3n de otros procesos causantes de tromboci\u00adtopenia.", "5": "La determinaci\u00f3n de anticuerpos antiplaque\u00ad tarios no es precisa para establecer el diag\u00adn\u00f3stico."}, "correct_option": 2, "explanations": {"1": {"exist": true, "char_ranges": [[0, 179]], "word_ranges": [[0, 29]], "text": "Ante una trombocitopenia aislada no hay que perder de vista la exploraci\u00f3n f\u00edsica; as\u00ed, la presencia de adenopat\u00edas nos obliga a descartar la presencia de un linfoma, por ejemplo."}, "2": {"exist": true, "char_ranges": [[198, 413]], "word_ranges": [[33, 70]], "text": "Al realizar una biopsia-aspirado de m\u00e9dula \u00f3sea se ve que el n\u00famero de megacariocitos es normal o incluso est\u00e1 aumentado, el fallo no est\u00e1 en la m\u00e9dula sino en la sangre perif\u00e9rica, donde se destruyen las plaquetas."}, "3": {"exist": false, "char_ranges": [], "word_ranges": [], "text": ""}, "4": {"exist": false, "char_ranges": [], "word_ranges": [], "text": ""}, "5": {"exist": false, "char_ranges": [], "word_ranges": [], "text": ""}}} +{"id": 353, "year": 2016, "question_id_specific": 161, "full_question": "Mujer de 27 a\u00f1os remitida a consulta de ginecolog\u00eda para su valoraci\u00f3n refiriendo dispareunia desde hace unos 8 meses, junto con disquecia y rectorragia ocasional coincidiendo con la menstruaci\u00f3n desde hace 3-4 meses. Tambi\u00e9n refiere dismenorrea desde hace a\u00f1os que controla bien con ibuprofeno. Lleva intentando quedarse embarazada unos 16 meses sin haberlo conseguido a\u00fan. En la exploraci\u00f3n ginecol\u00f3gica tan s\u00f3lo se aprecia dolor al presionar fondo de saco vaginal posterior. \u00bfQu\u00e9 prueba considera Ud que le permitir\u00eda llegar al diagn\u00f3stico de certeza de su patolog\u00eda?", "full_answer": "Laparoscopia diagn\u00f3stica. Es un caso t\u00edpico de endometriosis profunda, con implantes diferentes partes de la pelvis. El diagn\u00f3stico definitivo nos lo dar\u00e1 la anatom\u00eda patol\u00f3gica, as\u00ed que para ello tenemos que hacer Laparoscopia y enviar a AP.", "type": "GINECOLOG\u00cdA Y OBSTETRICIA", "options": {"1": "Ecograf\u00eda transvaginal.", "2": "Laparoscopia diagn\u00f3stica.", "3": "Resonancia magn\u00e9tica.", "4": "Colonoscopia.", "5": null}, "correct_option": 2, "explanations": {"1": {"exist": false, "char_ranges": [], "word_ranges": [], "text": ""}, "2": {"exist": true, "char_ranges": [[26, 242]], "word_ranges": [[2, 37]], "text": "Es un caso t\u00edpico de endometriosis profunda, con implantes diferentes partes de la pelvis. El diagn\u00f3stico definitivo nos lo dar\u00e1 la anatom\u00eda patol\u00f3gica, as\u00ed que para ello tenemos que hacer Laparoscopia y enviar a AP."}, "3": {"exist": false, "char_ranges": [], "word_ranges": [], "text": ""}, "4": {"exist": false, "char_ranges": [], "word_ranges": [], "text": ""}, "5": {"exist": false, "char_ranges": [], "word_ranges": [], "text": ""}}} +{"id": 438, "year": 2018, "question_id_specific": 119, "full_question": "Mujer de 38 a\u00f1os de edad de profesi\u00f3n veterinaria, encargada de la vigilancia de animales salvajes y de ayudar a partos de ganado dom\u00e9stico. Comienza con un cuadro de fiebre alta con escalofr\u00edos, cefalea, mialgias y tos no productiva que interpreta como un proceso gripal. Acude por presentar dolor tor\u00e1cico. En la radiograf\u00eda de t\u00f3rax se objetivan infiltrados pulmonares bilaterales en campos inferiores. Se realiza una prueba serol\u00f3gica con elevaci\u00f3n de titulos de anticuerpos frente a ant\u00edgenos en fase II. \u00bf Cual de las siguientes afirmaciones es CIERTA?", "full_answer": "Como posible enfermedad considerar\u00eda la Brucelosis, t\u00edpica del ganado (leche, aborto, secreciones genitales\u2026). Se presenta con una sintomatolog\u00eda de cuadro pseudogripal (cefalea, artalgias, mialgias, fiebre\u2026) y otros s\u00edntomas en funci\u00f3n de la v\u00eda de entrada. En el caso de nuestro caso, al asistir partos de animales la entrada es por v\u00eda respiratoria por lo que los infiltrados pulmonares y el dolor tor\u00e1cico se deben a la bacteria. El tratamiento var\u00eda en funci\u00f3n de la afectaci\u00f3n y la mortalidad es muy baja, casi nula. Siendo m\u00e1s preocupante su morbilidad y la afectaci\u00f3n en embarazadas. Importante: es muy frecuente en trabajadores de ganado (veterinarios, granjeros, mataderos\u2026) si lees algo tipo \u201casiste partos de animales de ganado\u201d, \u201ctrabaja en granja\u201d, \u201ces veterinario\u201d sospecha Brucelosis.", "type": "EPIDEMIOLOG\u00cdA Y MEDICINA PREVENTIVA", "options": {"1": "La forma de trasmisi\u00f3n de esta entidad es por garrapatas.", "2": "Tanto la doxiciclina como la hidroxicloroquina son eficaces para tratar las formas agudas de esta enfermedad.", "3": "En su forma aguda tambi\u00e9n presenta, generalmente, elevaci\u00f3n de anticuerpos frente a ant\u00edgenos en fase l.", "4": "La mortalidad en las formas agudas es casi inexistente.", "5": null}, "correct_option": 4, "explanations": {"1": {"exist": false, "char_ranges": [], "word_ranges": [], "text": ""}, "2": {"exist": false, "char_ranges": [], "word_ranges": [], "text": ""}, "3": {"exist": false, "char_ranges": [], "word_ranges": [], "text": ""}, "4": {"exist": true, "char_ranges": [[259, 522]], "word_ranges": [[35, 83]], "text": "En el caso de nuestro caso, al asistir partos de animales la entrada es por v\u00eda respiratoria por lo que los infiltrados pulmonares y el dolor tor\u00e1cico se deben a la bacteria. El tratamiento var\u00eda en funci\u00f3n de la afectaci\u00f3n y la mortalidad es muy baja, casi nula."}, "5": {"exist": false, "char_ranges": [], "word_ranges": [], "text": ""}}} +{"id": 579, "year": 2022, "question_id_specific": 108, "full_question": "Var\u00f3n de 49 a\u00f1os que ingresa en UCI por un traumatismo craneoencef\u00e1lico tras un accidente laboral. En la exploraci\u00f3n f\u00edsica no abre ojos, no emite sonidos antes de ser intubado y presenta extensi\u00f3n de extremidades al est\u00edmulo nociceptivo. Se coloca un sensor de presi\u00f3n intracraneal y debe realizarse una craneotom\u00eda descompresiva por una hemorragia intraparenquimatosa. \u00bfCu\u00e1l de las siguientes afirmaciones es correcta?:", "full_answer": "La hipercapnia y la acidosis producen vasodilataci\u00f3n que provocan aumento del flujo sangu\u00edneo cerebral. Seg\u00fan la exploraci\u00f3n descrita el Glasgow es de 4 (O1V1M2). La presi\u00f3n de perfusi\u00f3n cerebral es la presi\u00f3n arterial media menos la presi\u00f3n intracraneal. El edema cerebral en el que se produce edema celular por introducci\u00f3n de l\u00edquido extracelular a compartimento intracelular es el citot\u00f3xico.", "type": "NEUROLOG\u00cdA", "options": {"1": "A su llegada se encuentra en un coma de la escala de Glasgow de 7.", "2": "La presi\u00f3n de perfusi\u00f3n cerebral es la presi\u00f3n arterial media m\u00e1s la presi\u00f3n intracraneal.", "3": "El edema cerebral vasog\u00e9nico se produce por edema celular, rotura de la membrana y muerte celular.", "4": "El flujo vascular cerebral aumenta con la hipercapnia y la acidosis.", "5": null}, "correct_option": 4, "explanations": {"1": {"exist": true, "char_ranges": [[104, 162]], "word_ranges": [[14, 24]], "text": "Seg\u00fan la exploraci\u00f3n descrita el Glasgow es de 4 (O1V1M2)."}, "2": {"exist": true, "char_ranges": [[163, 255]], "word_ranges": [[24, 38]], "text": "La presi\u00f3n de perfusi\u00f3n cerebral es la presi\u00f3n arterial media menos la presi\u00f3n intracraneal."}, "3": {"exist": true, "char_ranges": [[256, 396]], "word_ranges": [[38, 59]], "text": "El edema cerebral en el que se produce edema celular por introducci\u00f3n de l\u00edquido extracelular a compartimento intracelular es el citot\u00f3xico."}, "4": {"exist": true, "char_ranges": [[0, 103]], "word_ranges": [[0, 14]], "text": "La hipercapnia y la acidosis producen vasodilataci\u00f3n que provocan aumento del flujo sangu\u00edneo cerebral."}, "5": {"exist": false, "char_ranges": [], "word_ranges": [], "text": ""}}} +{"id": 212, "year": 2014, "question_id_specific": 88, "full_question": "A un hombre de 45 a\u00f1os de edad se le realiza una vagotom\u00eda troncular y antrectom\u00eda con reconstrucci\u00f3n tipo Billroth II, por padecer una enfermedad ulcerosa p\u00e9ptica cr\u00f3nica con estenosis p\u00edloro-duodenal. Seis semanas despu\u00e9s de la cirug\u00eda refiere que poco tiempo despu\u00e9s (menos de media hora) de las ingestas presenta estado nauseoso, astenia y sudoraci\u00f3n, mareos y retortijones abdominales generalmente acompa\u00f1ados de despe\u00f1os diarreicos. \u00bfCu\u00e1l de las siguientes es la actitud m\u00e1s adecuada para su manejo inicial?", "full_answer": "Las respuestas 1, 2 y 5 son tratamientos adecuados para el s\u00edndrome de dumping o postgastrectom\u00eda, pero la pregunta est\u00e1 enfocada al manejo inicial, por lo que la respuesta m\u00e1s adecuada parece la 2.", "type": "CIRUG\u00cdA GENERAL", "options": {"1": "Aplicar tratamiento con un inhibidor de la somatostatina (octre\u00f3tido).", "2": "Seguir unas medidas diet\u00e9ticas concretas.", "3": "Realizar un tratamiento de prueba con una benzodiacepina.", "4": "Realizar la b\u00fasqueda de un probable tumor neuroendocrino (p.ej. carcinoide).", "5": "Indicar tratamiento quir\u00fargico para efectuar una gastroyeyunostom\u00eda antiperist\u00e1ltica en Y de Roux."}, "correct_option": 2, "explanations": {"1": {"exist": true, "char_ranges": [[0, 198]], "word_ranges": [[0, 34]], "text": "Las respuestas 1, 2 y 5 son tratamientos adecuados para el s\u00edndrome de dumping o postgastrectom\u00eda, pero la pregunta est\u00e1 enfocada al manejo inicial, por lo que la respuesta m\u00e1s adecuada parece la 2."}, "2": {"exist": true, "char_ranges": [[0, 198]], "word_ranges": [[0, 34]], "text": "Las respuestas 1, 2 y 5 son tratamientos adecuados para el s\u00edndrome de dumping o postgastrectom\u00eda, pero la pregunta est\u00e1 enfocada al manejo inicial, por lo que la respuesta m\u00e1s adecuada parece la 2."}, "3": {"exist": false, "char_ranges": [], "word_ranges": [], "text": ""}, "4": {"exist": false, "char_ranges": [], "word_ranges": [], "text": ""}, "5": {"exist": true, "char_ranges": [[0, 198]], "word_ranges": [[0, 34]], "text": "Las respuestas 1, 2 y 5 son tratamientos adecuados para el s\u00edndrome de dumping o postgastrectom\u00eda, pero la pregunta est\u00e1 enfocada al manejo inicial, por lo que la respuesta m\u00e1s adecuada parece la 2."}}} +{"id": 507, "year": 2020, "question_id_specific": 89, "full_question": "Un hombre de 46 a\u00f1os con trastorno bipolar es llevado a urgencias tras una sobreingesta de carbonato de litio. En la exploraci\u00f3n destaca temblor intenso, ataxia, disartria, mioclon\u00edas y fasciculaciones. La litemia es de 4,1 mEq/L (toxicidad > 1,6 mEq/L). \u00bfCu\u00e1l de las siguientes opciones terap\u00e9utica estar\u00eda m\u00e1s indicada?:", "full_answer": "Ojito con el litio. Se han visto intoxicaciones con litemias normales. No es rara -sobre todo por interacciones-. Un f\u00e1rmaco muy eficaz si todo el mundo est\u00e1 prevenido.", "type": "PSIQUIATR\u00cdA", "options": {"1": "Aminofilina asociado a un cat\u00e1rtico.", "2": "Carb\u00f3n activado.", "3": "Hemodi\u00e1lisis.", "4": "Diuresis forzada.", "5": null}, "correct_option": 3, "explanations": {"1": {"exist": false, "char_ranges": [], "word_ranges": [], "text": ""}, "2": {"exist": false, "char_ranges": [], "word_ranges": [], "text": ""}, "3": {"exist": false, "char_ranges": [], "word_ranges": [], "text": ""}, "4": {"exist": false, "char_ranges": [], "word_ranges": [], "text": ""}, "5": {"exist": false, "char_ranges": [], "word_ranges": [], "text": ""}}} +{"id": 552, "year": 2022, "question_id_specific": 89, "full_question": "Ni\u00f1a de 4 a\u00f1os que presenta fiebre alta de 6 d\u00edas de evoluci\u00f3n. En la exploraci\u00f3n cl\u00ednica presenta un exantema eritematoso maculopapular en tronco y zona genital, con tendencia a la confluencia, sin llegar a ser escarlatiniforme; inyecci\u00f3n conjuntival sin secreciones y labios rojos con lengua aframbuesada. Presenta tambi\u00e9n eritema con edema en manos y pies y una adenopat\u00eda cervical unilateral de 2 cm de di\u00e1metro. El diagn\u00f3stico cl\u00ednico de sospecha m\u00e1s probable es:", "full_answer": "No hay lugar a dudas, no est\u00e1n dando los criterios diagn\u00f3sticos de una enfermedad de Kawasaki (fiebre de varios d\u00edas de evoluci\u00f3n, exantema, inyecci\u00f3n conjuntival, labios rojos, lengua aframbuesada, edema de partes acrales y adenopat\u00eda).", "type": "DERMATOLOG\u00cdA", "options": {"1": "Enfermedad de Kawasaki.", "2": "Sarampi\u00f3n.", "3": "Rubeola.", "4": "Escarlatina.", "5": null}, "correct_option": 1, "explanations": {"1": {"exist": true, "char_ranges": [[0, 237]], "word_ranges": [[0, 35]], "text": "No hay lugar a dudas, no est\u00e1n dando los criterios diagn\u00f3sticos de una enfermedad de Kawasaki (fiebre de varios d\u00edas de evoluci\u00f3n, exantema, inyecci\u00f3n conjuntival, labios rojos, lengua aframbuesada, edema de partes acrales y adenopat\u00eda)."}, "2": {"exist": false, "char_ranges": [], "word_ranges": [], "text": ""}, "3": {"exist": false, "char_ranges": [], "word_ranges": [], "text": ""}, "4": {"exist": false, "char_ranges": [], "word_ranges": [], "text": ""}, "5": {"exist": false, "char_ranges": [], "word_ranges": [], "text": ""}}} +{"id": 598, "year": 2022, "question_id_specific": 181, "full_question": "Var\u00f3n de 67 a\u00f1os que presenta un cuadro de 3 meses de duraci\u00f3n de astenia y febr\u00edcula, a la que se a\u00f1ade en el \u00faltimo mes obstrucci\u00f3n nasal con emisi\u00f3n de moco con algunos co\u00e1gulos. En los \u00faltimos d\u00edas nota dolor en el ojo derecho y asimetr\u00eda respecto al contralateral. En la exploraci\u00f3n f\u00edsica se objetiva proptosis del globo ocular derecho y la inspecci\u00f3n de las fosas nasales constata una mucosa eritematosa con costras serohem\u00e1ticas. El resto de la exploraci\u00f3n es normal. Los an\u00e1lisis de sangre (hemograma, funci\u00f3n renal y hep\u00e1tica) son normales, a excepci\u00f3n de una VSG de 65 mm/h; en el an\u00e1lisis de orina se detecta microhematuria y proteinuria de 520 mg/24h. \u00bfCu\u00e1l es el diagn\u00f3stico inicial m\u00e1s probable?", "full_answer": "En este paciente, presenta una granulomatosis con poliangitis antes conocida como granulomatosis de Wegener. Nos comentan en el caso cl\u00ednico la afectaci\u00f3n otorrinolaringol\u00f3gica que est\u00e1 presente en el 92% de los pacientes. A nivel pulmonar nos comenta que ha presentado moco con alg\u00fan co\u00e1gulo, este tipo de afectaci\u00f3n est\u00e1 presente en el 85% de los pacientes. Tambi\u00e9n al acabar el caso, nos comentan el an\u00e1lisis de orina con una glomerulonefritis que est\u00e1n presentes en el 77% de los pacientes.", "type": "REUMATOLOG\u00cdA", "options": {"1": "Granulomatosis eosinof\u00edlica con poliangitis.", "2": "Poliangitis microsc\u00f3pica.", "3": "Granulomatosis con poliangitis.", "4": "Poliarteritis nudosa.", "5": null}, "correct_option": 3, "explanations": {"1": {"exist": false, "char_ranges": [], "word_ranges": [], "text": ""}, "2": {"exist": false, "char_ranges": [], "word_ranges": [], "text": ""}, "3": {"exist": true, "char_ranges": [[0, 494]], "word_ranges": [[0, 79]], "text": "En este paciente, presenta una granulomatosis con poliangitis antes conocida como granulomatosis de Wegener. Nos comentan en el caso cl\u00ednico la afectaci\u00f3n otorrinolaringol\u00f3gica que est\u00e1 presente en el 92% de los pacientes. A nivel pulmonar nos comenta que ha presentado moco con alg\u00fan co\u00e1gulo, este tipo de afectaci\u00f3n est\u00e1 presente en el 85% de los pacientes. Tambi\u00e9n al acabar el caso, nos comentan el an\u00e1lisis de orina con una glomerulonefritis que est\u00e1n presentes en el 77% de los pacientes."}, "4": {"exist": false, "char_ranges": [], "word_ranges": [], "text": ""}, "5": {"exist": false, "char_ranges": [], "word_ranges": [], "text": ""}}} +{"id": 133, "year": 2012, "question_id_specific": 168, "full_question": "Mujer de 75 a\u00f1os diagnosticada hace 3 a\u00f1os de drusas blandas en el fondo de ojo. Refiere presentar, desde hace 2 semanas, metamorfopsia y p\u00e9rdida visual importante en su ojo derecho que le impide leer. Se\u00f1ale el diagn\u00f3stico m\u00e1s probable:", "full_answer": "De las cinco opciones, cuatro podr\u00edan ser compatibles con la cl\u00ednica referida. Podr\u00edamos descartar inicialmente la coriorretinopat\u00eda serosa central porque por definici\u00f3n sucede en gente joven (hasta los 55 a\u00f1os, m\u00e1s o menos). La clave para diferenciar entre las cuatro restantes es que ten\u00eda drusas blandas en el fondo de ojo. Que si bien no es un \u201cdiagn\u00f3stico\u201d (la pregunta pod\u00eda estar mejor redactada) es un hallazgo relacionado con la degeneraci\u00f3n macular asociada a la edad, o degeneraci\u00f3n macular senil. La forma seca o atr\u00f3fica cursa con p\u00e9rdida visual lentamente progresiva. Pero la forma h\u00fameda o exudativa produce de forma aguda o r\u00e1pidamente progresiva una p\u00e9rdida visual severa (normalmente por hemorragia subrretiniana acompa\u00f1ado de metamorfopsia (se deforma la imagen por el levantamiento de la retina). Una membrana epirretiniana suele llevar un curso m\u00e1s lento. En un agujero macular es m\u00e1s relevante la p\u00e9rdida visual (puede referir tambi\u00e9n metamorfopsia, pero es menos probable en el momento agudo). En la trombosis de la vena central de la retina es m\u00e1s importante la p\u00e9rdida visual y no suelen explicar metamorfopsia (pero la puede tener, por el edema macular).", "type": "OFTALMOLOG\u00cdA", "options": {"1": "Membrana epirretiniana macular.", "2": "Agujero macular.", "3": "Trombosis de la vena central de la retina.", "4": "Degeneraci\u00f3n macular senil.", "5": "Coriorretinopat\u00eda serosa central."}, "correct_option": 4, "explanations": {"1": {"exist": true, "char_ranges": [[817, 876]], "word_ranges": [[125, 134]], "text": "Una membrana epirretiniana suele llevar un curso m\u00e1s lento."}, "2": {"exist": true, "char_ranges": [[877, 1016]], "word_ranges": [[134, 156]], "text": "En un agujero macular es m\u00e1s relevante la p\u00e9rdida visual (puede referir tambi\u00e9n metamorfopsia, pero es menos probable en el momento agudo)."}, "3": {"exist": true, "char_ranges": [[1017, 1180]], "word_ranges": [[156, 185]], "text": "En la trombosis de la vena central de la retina es m\u00e1s importante la p\u00e9rdida visual y no suelen explicar metamorfopsia (pero la puede tener, por el edema macular)."}, "4": {"exist": true, "char_ranges": [[226, 508]], "word_ranges": [[33, 80]], "text": "La clave para diferenciar entre las cuatro restantes es que ten\u00eda drusas blandas en el fondo de ojo. Que si bien no es un \u201cdiagn\u00f3stico\u201d (la pregunta pod\u00eda estar mejor redactada) es un hallazgo relacionado con la degeneraci\u00f3n macular asociada a la edad, o degeneraci\u00f3n macular senil."}, "5": {"exist": true, "char_ranges": [[79, 225]], "word_ranges": [[12, 33]], "text": "Podr\u00edamos descartar inicialmente la coriorretinopat\u00eda serosa central porque por definici\u00f3n sucede en gente joven (hasta los 55 a\u00f1os, m\u00e1s o menos)."}}} +{"id": 186, "year": 2013, "question_id_specific": 67, "full_question": "Una paciente de 56 a\u00f1os consult\u00f3 por dolores en columna dorso-lumbar y dificultad progresiva para realizar las tareas habituales. En los \u00faltimos 5 a\u00f1os aument\u00f3 de peso, tiene equ\u00edmosis con facilidad y se detect\u00f3 hipertensi\u00f3n arterial. Exploraci\u00f3n f\u00edsica: Obesidad de predominio central, facies redondeada, aumento de la grasa supraclavicular, disminuci\u00f3n de la fuerza muscular proximal y algunas estr\u00edas rojizas en abdomen. Tiene glucemia de 136 mg/dL y el estudio radiol\u00f3gico mostr\u00f3 osteoporosis y aplastamientos vertebrales. \u00bfCu\u00e1l le parece la interpretaci\u00f3n y actitud m\u00e1s coherentes?", "full_answer": "Pregunta un poco enrevesada de diagn\u00f3stico del S\u00edndrome de Cushing pero f\u00e1cil si se tienen claro los pasos. Todos estamos de acuerdo en que parece que la paciente tenga un S. Cushing (descartamos la 1). Para el diagn\u00f3stico hay que medir el cortisol libre urinario, hacer un test de supresi\u00f3n con dexametasona o medir el cortisol en saliva nocturno. Si se confirma el hipercortisolismo, habr\u00eda que buscar el origen; la medici\u00f3n de ACTH orienta el origen: suprimida en el hipercortisolismo de origen adrenal o uso prolongado de corticoides y elevada o normal si el origen es hipofisario o por secreci\u00f3n ect\u00f3pica de ACTH. (Por tanto, la 4 y la 5 son falsas). Si la ACTH es baja habr\u00eda que hacer un TAC de abdomen para localizar origen. Si la ACTH es normal o alta, habr\u00eda que hacer una RMN de hip\u00f3fisis (los adenomas hipofisarios responsables de S. de Cushing son muy peque\u00f1os y el TAC es la hip\u00f3fisis es menos sensible que la RMN). Es muy importante realizar bien el orden para llegar a un diagn\u00f3stico adecuado: 1\u00ba: confirmar hipercortisolismo o s. Cushing, 2\u00ba: medir ACTH para orientar etiolog\u00eda. 3 \u00ba: prueba de imagen seg\u00fan niveles de ACTH. Por tanto, la respuesta 2 es falsa (el TAC de abdomen no es una prueba que sirva para descartar s. Cushing); s\u00f3lo se debe hacer una prueba de imagen una vez diagn\u00f3stico claro de hipercortisolismo. Por tanto, la verdadera es la 3.", "type": "ENDOCRINOLOG\u00cdA", "options": {"1": "Osteoporosis post-menop\u00e1usica, diabetes mellitus tipo 2 e hipertensi\u00f3n arterial esencial, con disminuci\u00f3n de fuerza por polineuropatia diab\u00e9tica.", "2": "Es necesario descartar enfermedad de Cushing mediante test de supresi\u00f3n con dexametasona y realizar una TC craneal.", "3": "Sugiere un Cushing. Determinar cortisol libre urinario y ACTH basal, que sirve para orientar su etiolog\u00eda y seleccionar la t\u00e9cnica de imagen m\u00e1s apropiada.", "4": "Parece un Cushing. Si la ACTH basal es alta, puede ser por el uso de corticoides o un tumor suprarrenal, debiendo realizar una resonancia magn\u00e9tica.", "5": "Probablemente tiene un Cushing. Si la ACTH basal es baja, probablemente tenga un micro-adenoma hipofisario, debiendo realizar una TC craneal."}, "correct_option": 3, "explanations": {"1": {"exist": true, "char_ranges": [[109, 202]], "word_ranges": [[18, 35]], "text": "Todos estamos de acuerdo en que parece que la paciente tenga un S. Cushing (descartamos la 1)."}, "2": {"exist": true, "char_ranges": [[1152, 1249]], "word_ranges": [[200, 219]], "text": "la respuesta 2 es falsa (el TAC de abdomen no es una prueba que sirva para descartar s. Cushing);"}, "3": {"exist": true, "char_ranges": [[1250, 1337]], "word_ranges": [[219, 233]], "text": "s\u00f3lo se debe hacer una prueba de imagen una vez diagn\u00f3stico claro de hipercortisolismo."}, "4": {"exist": true, "char_ranges": [[349, 655]], "word_ranges": [[59, 112]], "text": "Si se confirma el hipercortisolismo, habr\u00eda que buscar el origen; la medici\u00f3n de ACTH orienta el origen: suprimida en el hipercortisolismo de origen adrenal o uso prolongado de corticoides y elevada o normal si el origen es hipofisario o por secreci\u00f3n ect\u00f3pica de ACTH. (Por tanto, la 4 y la 5 son falsas)."}, "5": {"exist": true, "char_ranges": [[349, 655]], "word_ranges": [[59, 112]], "text": "Si se confirma el hipercortisolismo, habr\u00eda que buscar el origen; la medici\u00f3n de ACTH orienta el origen: suprimida en el hipercortisolismo de origen adrenal o uso prolongado de corticoides y elevada o normal si el origen es hipofisario o por secreci\u00f3n ect\u00f3pica de ACTH. (Por tanto, la 4 y la 5 son falsas)."}}} +{"id": 7, "year": 2011, "question_id_specific": 208, "full_question": "Durante una colonoscopia se detecta un tumor de 5 cm localizado en colon derecho en un hombre de 48 a\u00f1os. No se han encontrado otras lesiones. Su abuela materna tambi\u00e9n padeci\u00f3 un c\u00e1ncer de colon. Las biopsias son superficiales y muestran un tumor pobremente diferenciado con abundantes c\u00e9lulas inflamatorias en el estroma que se diagnostica como un carcinoma de tipo medular.", "full_answer": "Pegunta complicada sobre un adenocarcinoma poco habitual. Por descarte, la 1 ser\u00eda f\u00e1cil de eliminar, el tratamiento del c\u00e1ncer de colon es quir\u00fargico (con neoadyuvancia en los de recto). Las biopsias si confirman adenocarcinoma no hace falta repetirla, ya que lo necesario es tener la pieza completa. La mayor parte de los ca. de colon se desarrollan sobre adenomas con lo cual esta la podr\u00edamos eliminar. Entre las otras 2 hay que saber que la 5 es correcta ya que suelen presentar casi siempre inestabilidad de microsat\u00e9lites incluso sin cumplir criterios de Amsterdam y tienen mejor pron\u00f3stico frente a otros ca de colon poco diferenciados.", "type": "DIGESTIVO", "options": {"1": "La quimioterapia es el tratamiento de elecci\u00f3n.", "2": "Puesto que la biopsia es superficial debe repetirse antes de proceder al tratamiento.", "3": "El pron\u00f3stico del tumor depende principalmente de su alto grado de anaplasia.", "4": "Es poco probable que este tumor se haya desarrollado sobre un adenoma previo.", "5": "Debe estudiarse la inestabilidad de microsat\u00e9lites, y los genes de reparaci\u00f3n de errores del DNA."}, "correct_option": 5, "explanations": {"1": {"exist": true, "char_ranges": [[58, 187]], "word_ranges": [[7, 29]], "text": "Por descarte, la 1 ser\u00eda f\u00e1cil de eliminar, el tratamiento del c\u00e1ncer de colon es quir\u00fargico (con neoadyuvancia en los de recto)."}, "2": {"exist": true, "char_ranges": [[188, 301]], "word_ranges": [[29, 47]], "text": "Las biopsias si confirman adenocarcinoma no hace falta repetirla, ya que lo necesario es tener la pieza completa."}, "3": {"exist": true, "char_ranges": [[302, 406]], "word_ranges": [[47, 66]], "text": "La mayor parte de los ca. de colon se desarrollan sobre adenomas con lo cual esta la podr\u00edamos eliminar."}, "4": {"exist": true, "char_ranges": [[407, 644]], "word_ranges": [[66, 105]], "text": "Entre las otras 2 hay que saber que la 5 es correcta ya que suelen presentar casi siempre inestabilidad de microsat\u00e9lites incluso sin cumplir criterios de Amsterdam y tienen mejor pron\u00f3stico frente a otros ca de colon poco diferenciados."}, "5": {"exist": true, "char_ranges": [[407, 644]], "word_ranges": [[66, 105]], "text": "Entre las otras 2 hay que saber que la 5 es correcta ya que suelen presentar casi siempre inestabilidad de microsat\u00e9lites incluso sin cumplir criterios de Amsterdam y tienen mejor pron\u00f3stico frente a otros ca de colon poco diferenciados."}}} +{"id": 152, "year": 2012, "question_id_specific": 72, "full_question": "Un paciente hiperuric\u00e9mico que habitualmente toma 100 mg de alopurinol al d\u00eda acude a urgencias con dolor agudo y signos inflamatorios en rodilla derecha. Se realiza artrocentesis y en el microscopio de luz polarizada se observan cristales intracelulares con birrefringencia negativa. \u00bfQu\u00e9 actitud terap\u00e9utica entre las que a continuaci\u00f3n se indican, es la m\u00e1s adecuada en este caso?", "full_answer": "Ante un artritis gotosa aguda (la presencia de cristales intracelulares con birrefringencia negativa lo confirma) en un paciente hiperuric\u00e9mico que previamente segu\u00eda tratamiento con alopurinol se debe a\u00f1adir un AINE hasta que la crisis remita. No se deben hacer cambios en el tratamiento hipouricemiante durante este periodo porque se altera la cadena metab\u00f3lica del \u00e1cido \u00farico y puede empeorar la situaci\u00f3n.", "type": "REUMATOLOG\u00cdA", "options": {"1": "Suspender alopurinol e iniciar tratamiento con colchicina.", "2": "Suspender alopurinol y comenzar con AINE.", "3": "A\u00f1adir un AINE hasta que la crisis remita.", "4": "Aumentar la dosis de alopurinol a 300 mg/d\u00eda.", "5": "Sustituir el alopurinol por un uricos\u00farico."}, "correct_option": 3, "explanations": {"1": {"exist": true, "char_ranges": [[245, 410]], "word_ranges": [[35, 61]], "text": "No se deben hacer cambios en el tratamiento hipouricemiante durante este periodo porque se altera la cadena metab\u00f3lica del \u00e1cido \u00farico y puede empeorar la situaci\u00f3n."}, "2": {"exist": true, "char_ranges": [[245, 410]], "word_ranges": [[35, 61]], "text": "No se deben hacer cambios en el tratamiento hipouricemiante durante este periodo porque se altera la cadena metab\u00f3lica del \u00e1cido \u00farico y puede empeorar la situaci\u00f3n."}, "3": {"exist": true, "char_ranges": [[0, 244]], "word_ranges": [[0, 35]], "text": "Ante un artritis gotosa aguda (la presencia de cristales intracelulares con birrefringencia negativa lo confirma) en un paciente hiperuric\u00e9mico que previamente segu\u00eda tratamiento con alopurinol se debe a\u00f1adir un AINE hasta que la crisis remita."}, "4": {"exist": true, "char_ranges": [[245, 410]], "word_ranges": [[35, 61]], "text": "No se deben hacer cambios en el tratamiento hipouricemiante durante este periodo porque se altera la cadena metab\u00f3lica del \u00e1cido \u00farico y puede empeorar la situaci\u00f3n."}, "5": {"exist": true, "char_ranges": [[245, 410]], "word_ranges": [[35, 61]], "text": "No se deben hacer cambios en el tratamiento hipouricemiante durante este periodo porque se altera la cadena metab\u00f3lica del \u00e1cido \u00farico y puede empeorar la situaci\u00f3n."}}} +{"id": 243, "year": 2014, "question_id_specific": 112, "full_question": "Una chica de 20 a\u00f1os acude a la consulta con un cuadro agudo de fiebre, adenopat\u00edas cervicales y rash cut\u00e1neo. Seg\u00fan refiere la paciente, hace 3 semanas tuvo una relaci\u00f3n sexual que pudo ser de riesgo para contraer el virus del VIH. Indica cu\u00e1l de las siguientes respuestas es verdadera:", "full_answer": "Pregunta sencilla si tenemos claro el proceso diagn\u00f3stico del VIH. A primera vista, podr\u00edamos descartar directamente la respuesta 2 ya que sabemos que los cuadros mononucleosicos pueden ser expresi\u00f3n de la primera fase de una infecci\u00f3n por VIH, m\u00e1xime cuando tenemos el antecedente de un contacto de riesgo. Las respuestas 3 y 4 son tambi\u00e9n f\u00e1cilmente descartables; el ELISA es una t\u00e9cnica muy sensible pero poco espec\u00edfica y SIEMPRE es necesaria la confirmaci\u00f3n con una prueba Western-blot. La respuesta 1 y la 5 se contradicen si os dais cuenta, por lo que una de las dos es la correcta. En este caso, la falsa es la 1, se trata de una respuesta muy categ\u00f3rica (\u201cdescarta\u201d) que no se adapta del todo a la realidad; como hemos dicho, el ELISA es una t\u00e9cnica muy sensible, por lo que un resultado negativo podr\u00eda dejarnos bastante tranquilos, sin embargo, todos sabemos que no hay ninguna prueba m\u00e9dica que nos descarte nada con total certeza (a no ser que su sensibilidad sea del 100%, que no es el caso) y, por otra parte, podr\u00edamos decir que en este caso tenemos una alta sospecha cl\u00ednica (el cuadro es sugestivo, y los antecedentes est\u00e1n ah\u00ed). Adem\u00e1s, se da la circunstancia de que solamente hace 3 semanas desde el contacto, por lo que nuestra paciente se encuentra probablemente en el \u201cperiodo ventana\u201d durante el que las t\u00e9cnicas serol\u00f3gicas pueden ser poco rentables. En conclusi\u00f3n, la respuesta que mejor nos cuadra ser\u00eda la 5, en el periodo ventana puede estar indicada la realizaci\u00f3n de una carga viral para el diagn\u00f3stico (positivo si hay m\u00e1s de 10.000 copias), aunque esta prueba no se haga de forma rutinaria.", "type": "ENFERMEDADES INFECCIOSAS", "options": {"1": "Una serolog\u00eda VIH-1/VIH-2 negativa realizada por medio de la t\u00e9cnica de ELISA, descarta la posibilidad de que la paciente haya sido contagiada con el virus VIH.", "2": "El proceso cl\u00ednico que padece la paciente no concuerda con el de la infecci\u00f3n aguda por VIH.", "3": "Si el test de ELISA de la paciente fuera positivo, no ser\u00eda necesario realizar nada m\u00e1s para el diagn\u00f3stico de infecci\u00f3n por VIH.", "4": "La t\u00e9cnica de ELISA tiene una alta sensibilidad para el diagn\u00f3stico de infecci\u00f3n por VIH, pero su especificidad es a\u00fan mayor.", "5": "Si el test de ELISA para diagnosticar VIH en la paciente fuera negativo, podr\u00edamos determinar por la t\u00e9cnica de PCR la carga viral en sangre."}, "correct_option": 5, "explanations": {"1": {"exist": true, "char_ranges": [[735, 1005]], "word_ranges": [[127, 175]], "text": "el ELISA es una t\u00e9cnica muy sensible, por lo que un resultado negativo podr\u00eda dejarnos bastante tranquilos, sin embargo, todos sabemos que no hay ninguna prueba m\u00e9dica que nos descarte nada con total certeza (a no ser que su sensibilidad sea del 100%, que no es el caso)"}, "2": {"exist": true, "char_ranges": [[84, 307]], "word_ranges": [[13, 48]], "text": "podr\u00edamos descartar directamente la respuesta 2 ya que sabemos que los cuadros mononucleosicos pueden ser expresi\u00f3n de la primera fase de una infecci\u00f3n por VIH, m\u00e1xime cuando tenemos el antecedente de un contacto de riesgo."}, "3": {"exist": true, "char_ranges": [[366, 491]], "word_ranges": [[57, 77]], "text": "el ELISA es una t\u00e9cnica muy sensible pero poco espec\u00edfica y SIEMPRE es necesaria la confirmaci\u00f3n con una prueba Western-blot."}, "4": {"exist": true, "char_ranges": [[366, 491]], "word_ranges": [[57, 77]], "text": "el ELISA es una t\u00e9cnica muy sensible pero poco espec\u00edfica y SIEMPRE es necesaria la confirmaci\u00f3n con una prueba Western-blot."}, "5": {"exist": true, "char_ranges": [[1391, 1623]], "word_ranges": [[237, 278]], "text": "la respuesta que mejor nos cuadra ser\u00eda la 5, en el periodo ventana puede estar indicada la realizaci\u00f3n de una carga viral para el diagn\u00f3stico (positivo si hay m\u00e1s de 10.000 copias), aunque esta prueba no se haga de forma rutinaria."}}} +{"id": 458, "year": 2018, "question_id_specific": 126, "full_question": "Hombre de 47 a\u00f1os, con antecedente de un adenoma pleomorfo parot\u00eddeo derecho, tratado con cirug\u00eda (parotidectom\u00eda extrafacial) hace 6 meses, que acude a nuestra consulta por presentar durante la masticaci\u00f3n dolor con sudoraci\u00f3n y enrojecimiento de la piel de la regi\u00f3n preauricular. \u00bfQu\u00e9 tratamiento ser\u00eda de elecci\u00f3n?", "full_answer": "Est\u00e1n preguntando por el s\u00edndrome auriculotemporal o s\u00edndrome de Frey. Es la expresi\u00f3n cl\u00ednica de una neuropat\u00eda vegetativa que tiene como causa la lesi\u00f3n mec\u00e1nica o irritativa de las fibras vegetativas del nervio auriculotemporal en su trayecto infratemporal. En nuestro medio la causa m\u00e1s frecuente es la cirug\u00eda de la gl\u00e1ndula partida con un porcentaje de aparici\u00f3n entre el 10 y el 80%. El tratamiento conservador con toxina botul\u00ednica ofrece buenos resultados.", "type": "OTORRINOLARINGOLOG\u00cdA Y CIRUG\u00cdA MAXILOFACIAL", "options": {"1": "Parotidectom\u00eda total ampliada ante la sospecha de recidiva tumoral.", "2": "Pregabalina.", "3": "Inyecci\u00f3n de toxina botul\u00ednica intrad\u00e9rmica.", "4": "Antibioterapia de amplio espectro.", "5": null}, "correct_option": 3, "explanations": {"1": {"exist": false, "char_ranges": [], "word_ranges": [], "text": ""}, "2": {"exist": false, "char_ranges": [], "word_ranges": [], "text": ""}, "3": {"exist": true, "char_ranges": [[261, 465]], "word_ranges": [[38, 72]], "text": "En nuestro medio la causa m\u00e1s frecuente es la cirug\u00eda de la gl\u00e1ndula partida con un porcentaje de aparici\u00f3n entre el 10 y el 80%. El tratamiento conservador con toxina botul\u00ednica ofrece buenos resultados."}, "4": {"exist": false, "char_ranges": [], "word_ranges": [], "text": ""}, "5": {"exist": false, "char_ranges": [], "word_ranges": [], "text": ""}}} +{"id": 123, "year": 2012, "question_id_specific": 223, "full_question": "Gestante de 27 a\u00f1os, 30 semanas de gestaci\u00f3n. Acude a Urgencias por notar desde ayer dolor en regi\u00f3n lumbar izquierda y disuria. No tiene sensaci\u00f3n febril. Refiere infecciones del tracto urinario (ITU) reiteradas. En el an\u00e1lisis de orina se observan Hb 3+, leucocitos 3+, nitritos 2+, Sedimento: 15-20 leucocitos por campo y 5-10 hemat\u00edes por campo. Ante la conveniencia de instaurar un tratamiento emp\u00edrico. \u00bfCu\u00e1l de los siguientes microorganismos es el responsable m\u00e1s frecuente en las gestantes?", "full_answer": "La respuesta correcta es la primera. La causa m\u00e1s frecuente de infecciones urinarias es Escherichia coli y en el caso de las embarazadas tambi\u00e9n.", "type": "MICROBIOLOG\u00cdA", "options": {"1": "Escherichia coli.", "2": "Enterococcus faecalis.", "3": "Streptococcus agalactiae.", "4": "Proteus mirabilis.", "5": "Satphylococcus saprophyticus."}, "correct_option": 1, "explanations": {"1": {"exist": true, "char_ranges": [[37, 145]], "word_ranges": [[6, 24]], "text": "La causa m\u00e1s frecuente de infecciones urinarias es Escherichia coli y en el caso de las embarazadas tambi\u00e9n."}, "2": {"exist": false, "char_ranges": [], "word_ranges": [], "text": ""}, "3": {"exist": false, "char_ranges": [], "word_ranges": [], "text": ""}, "4": {"exist": false, "char_ranges": [], "word_ranges": [], "text": ""}, "5": {"exist": false, "char_ranges": [], "word_ranges": [], "text": ""}}} +{"id": 522, "year": 2021, "question_id_specific": 165, "full_question": "Mujer de 66 a\u00f1os diagnosticada de diabetes mellitus tipo 2 desde hace tres meses. Tiene un IMC de 31 Kg/m2 y presenta mal control gluc\u00e9mico a pesar de realizar un programa de medidas no farmacol\u00f3gicas (alimentaci\u00f3n saludable, ejercicio). \u00bfCu\u00e1l de los siguientes f\u00e1rmacos hipoglucemiantes se asocia a aumento de peso y deber\u00edamos evitar en esta paciente?:", "full_answer": "Una pregunta algo m\u00e1s complicada que la anterior. En cuanto al peso, la metformina en menor medidas, los inhibidores de SGLT-2 y, sobre todo, los agonistas de GLP-1, se han relacionado con una disminuci\u00f3n significativa del peso en paciente con DM-2 (respuestas 1,3 y 4 incorrectas). En cambio, la pioglitazona, tal como refleja su ficha t\u00e9cnica, puede producir aumento de peso dosis-dependiente, principalmente por acumulaci\u00f3n de grasa y a\u00f1adida, en algunos casos, a la retenci\u00f3n h\u00eddrica.", "type": "ENDOCRINOLOG\u00cdA", "options": {"1": "Metformina (biguanida).", "2": "Pioglitazona (tiazolidindiona).", "3": "Canagliflozina (inhibidor del cotransportador 2 de sodio-glucosa- iSGLT2).", "4": "Liraglutida (agonista del receptor para GLP-1).", "5": null}, "correct_option": 2, "explanations": {"1": {"exist": true, "char_ranges": [[50, 282]], "word_ranges": [[8, 46]], "text": "En cuanto al peso, la metformina en menor medidas, los inhibidores de SGLT-2 y, sobre todo, los agonistas de GLP-1, se han relacionado con una disminuci\u00f3n significativa del peso en paciente con DM-2 (respuestas 1,3 y 4 incorrectas)."}, "2": {"exist": true, "char_ranges": [[283, 488]], "word_ranges": [[46, 76]], "text": "En cambio, la pioglitazona, tal como refleja su ficha t\u00e9cnica, puede producir aumento de peso dosis-dependiente, principalmente por acumulaci\u00f3n de grasa y a\u00f1adida, en algunos casos, a la retenci\u00f3n h\u00eddrica."}, "3": {"exist": true, "char_ranges": [[50, 282]], "word_ranges": [[8, 46]], "text": "En cuanto al peso, la metformina en menor medidas, los inhibidores de SGLT-2 y, sobre todo, los agonistas de GLP-1, se han relacionado con una disminuci\u00f3n significativa del peso en paciente con DM-2 (respuestas 1,3 y 4 incorrectas)."}, "4": {"exist": true, "char_ranges": [[50, 282]], "word_ranges": [[8, 46]], "text": "En cuanto al peso, la metformina en menor medidas, los inhibidores de SGLT-2 y, sobre todo, los agonistas de GLP-1, se han relacionado con una disminuci\u00f3n significativa del peso en paciente con DM-2 (respuestas 1,3 y 4 incorrectas)."}, "5": {"exist": false, "char_ranges": [], "word_ranges": [], "text": ""}}} +{"id": 209, "year": 2014, "question_id_specific": 185, "full_question": "Una mujer de 32 a\u00f1os solicita consejo preconcepcional. La paciente refiere que fue sometida a una conizaci\u00f3n cervical por una lesi\u00f3n intraepitelial de alto grado (H-SIL) y que posteriormente sufri\u00f3 tres abortos entre las 20 y 22 semanas de gestaci\u00f3n. No tiene hijos vivos. En las tres ocasiones acudi\u00f3 a Urgencias con sensaci\u00f3n de peso en hipogastrio y all\u00ed se constat\u00f3 que llegaba con una dilataci\u00f3n de 8cm y con las membranas amni\u00f3ticas prominentes. Nunca hab\u00eda sentido contracciones. \u00bfQu\u00e9 consejo le dar\u00eda para el pr\u00f3ximo embarazo?", "full_answer": "Se trata de un t\u00edpico caso de incompetencia cervical (y esta paciente tiene como factor de riesgo la conizaci\u00f3n). Esta patolog\u00eda consiste en una dilataci\u00f3n del c\u00e9rvix en ausencia de contracciones, y que requiere la realizaci\u00f3n de un cerclaje para evitar el aborto o el parto inmaduro. El cerclaje se realiza idealmente de forma electiva, programada, y no cuando la paciente llega a Urgencias \u201cin extemis\u201d (que tambi\u00e9n se hacen, pero con menor tasa de \u00e9xito\u2026 lo ideal es hacerlos en fr\u00edo).", "type": "GINECOLOG\u00cdA Y OBSTETRICIA", "options": {"1": "Le pautar\u00eda una profilaxis con atosib\u00e1n por v\u00eda oral durante todo el embarazo.", "2": "Le ofrecer\u00eda la maduraci\u00f3n pulmonar con corticoides a partir de las 19-20 semanas de gestaci\u00f3n.", "3": "Le recomendar\u00eda realizar un cerclaje cervical programado a partir de la semana 14 de gestaci\u00f3n.", "4": "Le aconsejar\u00eda no intentar m\u00e1s embarazos por el alto riesgo de recidiva.", "5": "Le recomendar\u00eda recurrir a t\u00e9cnicas de reproducci\u00f3n asistida."}, "correct_option": 3, "explanations": {"1": {"exist": false, "char_ranges": [], "word_ranges": [], "text": ""}, "2": {"exist": false, "char_ranges": [], "word_ranges": [], "text": ""}, "3": {"exist": true, "char_ranges": [[0, 284]], "word_ranges": [[0, 47]], "text": "Se trata de un t\u00edpico caso de incompetencia cervical (y esta paciente tiene como factor de riesgo la conizaci\u00f3n). Esta patolog\u00eda consiste en una dilataci\u00f3n del c\u00e9rvix en ausencia de contracciones, y que requiere la realizaci\u00f3n de un cerclaje para evitar el aborto o el parto inmaduro."}, "4": {"exist": false, "char_ranges": [], "word_ranges": [], "text": ""}, "5": {"exist": false, "char_ranges": [], "word_ranges": [], "text": ""}}} +{"id": 282, "year": 2016, "question_id_specific": 55, "full_question": "Una paciente de 70 a\u00f1os ingresa en UCl tras sufrir IAM anterior tratado mediante angioplastia coronaria y colocaci\u00f3n de stent en la arteria descendente anterior. 4 d\u00edas despu\u00e9s presenta bruscamente hipotensi\u00f3n que obliga a aporte vigoroso de volumen, inicio de drogas vasoactivas, intubaci\u00f3n orotraqueal y conexi\u00f3n a la ventilaci\u00f3n mec\u00e1nica. A la exploraci\u00f3n f\u00edsica destaca un soplo no presente previamente. Ante la sospecha de complicaci\u00f3n mec\u00e1nica del infarto,se realiza ecocardiografia transtor\u00e1cica que muestra derrame peric\u00e1rdico. Se\u00f1ale la Respuesta CORRECTA:", "full_answer": "En caso de rotura de pared libre no hay fr\u00e9mito palpable.", "type": "CARDIOLOG\u00cdA Y CIRUG\u00cdA VASCULAR", "options": {"1": "La mortalidad con tratamiento m\u00e9dico es del 20%.", "2": "En caso de rotura de pared libre hay salto oxim\u00e9trico en el ventr\u00edculo derecho en el cateterismo de Swan-Ganz.", "3": "En caso de rotura de pared libre no hay fr\u00e9mito palpable.", "4": "Las complicaciones mec\u00e1nicas suelen aparecer en el primer d\u00eda postinfarto", "5": null}, "correct_option": 3, "explanations": {"1": {"exist": false, "char_ranges": [], "word_ranges": [], "text": ""}, "2": {"exist": false, "char_ranges": [], "word_ranges": [], "text": ""}, "3": {"exist": true, "char_ranges": [[0, 57]], "word_ranges": [[0, 11]], "text": "En caso de rotura de pared libre no hay fr\u00e9mito palpable."}, "4": {"exist": false, "char_ranges": [], "word_ranges": [], "text": ""}, "5": {"exist": false, "char_ranges": [], "word_ranges": [], "text": ""}}} +{"id": 464, "year": 2019, "question_id_specific": 155, "full_question": "Un hombre de 58 a\u00f1os, a las tres semanas de un esguince grave de tobillo presenta, de forma r\u00e1pidamente progresiva, disnea de reposo, mareo y s\u00edncope. Al llegar al hospital tiene hipotensi\u00f3n (PA sist\u00f3lica 80 mmHg, diast\u00f3lica 40 mmHg) y mala perfusi\u00f3n. Es intubado y conectado a ventilaci\u00f3n mec\u00e1nica y se inicia noradrenalina. El ecocardiograma muestra signos de h\u00edpertensi\u00e9n pulmonar. En la angio-TC se observan m\u00faltiples defectos de replecci\u00f3n ocupando ambas arterias pulmonares principales. \u00bfCu\u00e1l de los siguientes tratamientos se asociar\u00eda a una mejor\u00eda hemodin\u00e1mica m\u00e1s r\u00e1pida en este caso?", "full_answer": "El paciente presenta un tromboembolismo pulmonar que, adem\u00e1s de ser bilateral, produce una afectaci\u00f3n hemodin\u00e1mica severa, una situaci\u00f3n de shock y requiere ventilaci\u00f3n mec\u00e1nica. No nos dicen que tenga ninguna contraindicaci\u00f3n para realizar fibrinolisis, as\u00ed que \u00e9sta ser\u00eda la opci\u00f3n m\u00e1s adecuada por su rapidez de administraci\u00f3n y acci\u00f3n. La heparina s\u00f3dica se reserva para casos en los que el paciente est\u00e1 estable. La tromboendarterectom\u00eda podr\u00eda realizarse de urgencia en centros seleccionados (no disponible en todos) en pacientes en los que estuviera contraindicada la fibrinolisis sist\u00e9mica, o bien en centros donde la experiencia de esta t\u00e9cnica sea constatada y se pueda realizar con rapidez. El filtro en vena cava inferior es un tratamiento que se indica en el momento agudo pacientes estables, cuando tienen contraindicada la anticoagulaci\u00f3n; o bien a posteriori, como profilaxis, en este grupo de pacientes. Las indicaciones de realizaci\u00f3n de fibrinolisis del \u00faltimo documento de consenso de la SEPAR inclu\u00edan pacientes en riesgo intermedio (podr\u00edan beneficiarse de ella) y riesgo alto. Clasifican el riesgo intermedio como PESIs \u2265 1 o PESI III-IV. En este subgrupo, parece que se benefician m\u00e1s de la fibrinolisis los pacientes con disfunci\u00f3n de ventr\u00edculo derecho, troponina o BNP superior al l\u00edmite de corte y trombosis venosa profunda. En el subgrupo de pacientes de riesgo alto (definido por hipotensi\u00f3n o criterios de shock cardiog\u00e9nico), est\u00e1 mucho m\u00e1s claro el uso de fibrinolisis sist\u00e9mica, como el caso que nos presentan en la pregunta.", "type": "CUIDADOS CR\u00cdTICOS Y URGENCIAS", "options": {"1": "Heparina s\u00f3dica en perfusi\u00f3n intravenosa.", "2": "Tromboendartectom\u00eda.", "3": "Fibrinolisis sist\u00e9mica con rt-PA (alteplase) 100 mg intravenoso.", "4": "Filtro en vena cava inferior.", "5": null}, "correct_option": 3, "explanations": {"1": {"exist": true, "char_ranges": [[340, 417]], "word_ranges": [[49, 63]], "text": "La heparina s\u00f3dica se reserva para casos en los que el paciente est\u00e1 estable."}, "2": {"exist": true, "char_ranges": [[418, 701]], "word_ranges": [[63, 104]], "text": "La tromboendarterectom\u00eda podr\u00eda realizarse de urgencia en centros seleccionados (no disponible en todos) en pacientes en los que estuviera contraindicada la fibrinolisis sist\u00e9mica, o bien en centros donde la experiencia de esta t\u00e9cnica sea constatada y se pueda realizar con rapidez."}, "3": {"exist": true, "char_ranges": [[0, 339]], "word_ranges": [[0, 49]], "text": "El paciente presenta un tromboembolismo pulmonar que, adem\u00e1s de ser bilateral, produce una afectaci\u00f3n hemodin\u00e1mica severa, una situaci\u00f3n de shock y requiere ventilaci\u00f3n mec\u00e1nica. No nos dicen que tenga ninguna contraindicaci\u00f3n para realizar fibrinolisis, as\u00ed que \u00e9sta ser\u00eda la opci\u00f3n m\u00e1s adecuada por su rapidez de administraci\u00f3n y acci\u00f3n."}, "4": {"exist": true, "char_ranges": [[702, 920]], "word_ranges": [[104, 138]], "text": "El filtro en vena cava inferior es un tratamiento que se indica en el momento agudo pacientes estables, cuando tienen contraindicada la anticoagulaci\u00f3n; o bien a posteriori, como profilaxis, en este grupo de pacientes."}, "5": {"exist": false, "char_ranges": [], "word_ranges": [], "text": ""}}} +{"id": 502, "year": 2020, "question_id_specific": 77, "full_question": "Lactante de 6 meses que acude a urgencias por dificultad respiratoria. Exploraci\u00f3n: temperatura axilar 37,2\u00b0C, frecuencia respiratoria 40 rpm, frecuencia card\u00edaca 160 lpm, tensi\u00f3n arterial 90/45 mmHg, SatO2 95 % con aire ambiente. Muestra dificultad respiratoria moderada con retracci\u00f3n intercostal y subcostal. Auscultaci\u00f3n pulmonar: roncus espiratorios diseminados, espiraci\u00f3n alargada y ligera disminuci\u00f3n en la entrada de aire en ambos campos pulmonares. Auscultaci\u00f3n cardiaca: sin soplos. Se decide mantener al paciente en observaci\u00f3n en el hospital durante unas horas. \u00bfQu\u00e9 actitud considera m\u00e1s adecuada en este momento respecto a las pruebas complementarias?:", "full_answer": "El paciente probablemente presenta un cuadro de bronquiolitis. En este momento no ser\u00eda necesario realizar ninguna prueba complementaria salvo empeoramiento cl\u00ednico.", "type": "PEDIATR\u00cdA", "options": {"1": "Solicitar gasometr\u00eda venosa, recuento leucocitario y reactantes de fase aguda.", "2": "Solicitar radiograf\u00eda de t\u00f3rax.", "3": "Solicitar gasometr\u00eda arterial y reactantes de fase aguda.", "4": "No solicitar pruebas complementarias.", "5": null}, "correct_option": 4, "explanations": {"1": {"exist": false, "char_ranges": [], "word_ranges": [], "text": ""}, "2": {"exist": false, "char_ranges": [], "word_ranges": [], "text": ""}, "3": {"exist": false, "char_ranges": [], "word_ranges": [], "text": ""}, "4": {"exist": true, "char_ranges": [[0, 165]], "word_ranges": [[0, 21]], "text": "El paciente probablemente presenta un cuadro de bronquiolitis. En este momento no ser\u00eda necesario realizar ninguna prueba complementaria salvo empeoramiento cl\u00ednico."}, "5": {"exist": false, "char_ranges": [], "word_ranges": [], "text": ""}}} +{"id": 71, "year": 2012, "question_id_specific": 50, "full_question": "Un paciente con antecedentes de cuadro febril y dolor tor\u00e1cico acude al hospital con disnea y taquipnea. En la exploraci\u00f3n f\u00edsica las cifrad de presi\u00f3n arterial est\u00e1n bajas, la presi\u00f3n venosa yugular est\u00e1 elevada con un seno X descendiente profundo y tiene pulso parad\u00f3jico. Que patolog\u00eda debe sospechar?", "full_answer": "La PA baja con Presi\u00f3n yugular alta debe hacer siempre sospechar un taponamiento cardiaco. El cuadro febril y dolor tor\u00e1cico con disnea y taquipnea deben hacer sospechar un derrame peric\u00e1rdico con compromiso hemodin\u00e1mica. El pulso parad\u00f3jico es un hallazgo t\u00edpico de taponamiento cardiaco. Por tanto la respuesta correcata es la 5.", "type": "ANESTESIOLOG\u00cdA, CUIDADOS CR\u00cdTICOS Y URGENCIAS", "options": {"1": "Cardiopat\u00eda isqu\u00e9mica.", "2": "Miocardiopat\u00eda dilatada.", "3": "Estenosis valvular a\u00f3rtica severa.", "4": "Pericarditis constrictiva.", "5": "Derrame peric\u00e1rdico con taponamiento cardiaco."}, "correct_option": 5, "explanations": {"1": {"exist": false, "char_ranges": [], "word_ranges": [], "text": ""}, "2": {"exist": false, "char_ranges": [], "word_ranges": [], "text": ""}, "3": {"exist": false, "char_ranges": [], "word_ranges": [], "text": ""}, "4": {"exist": false, "char_ranges": [], "word_ranges": [], "text": ""}, "5": {"exist": true, "char_ranges": [[0, 289]], "word_ranges": [[0, 43]], "text": "La PA baja con Presi\u00f3n yugular alta debe hacer siempre sospechar un taponamiento cardiaco. El cuadro febril y dolor tor\u00e1cico con disnea y taquipnea deben hacer sospechar un derrame peric\u00e1rdico con compromiso hemodin\u00e1mica. El pulso parad\u00f3jico es un hallazgo t\u00edpico de taponamiento cardiaco."}}} +{"id": 486, "year": 2020, "question_id_specific": 45, "full_question": "Mujer de 15 a\u00f1os que presenta un retraso en la menarquia y una talla baja. No tiene discapacidad intelectual. \u00bfCu\u00e1l de las siguientes pruebas gen\u00e9ticas se utilizar\u00eda habitualmente para el diagn\u00f3stico de esta paciente?:", "full_answer": "Esta pregunta puede entra\u00f1ar algo m\u00e1s de dificultad, pues el caso cl\u00ednico que nos presentan es muy vago. Una adolescente con retraso puberal y talla baja; nada m\u00e1s. Con esa informaci\u00f3n podr\u00eda tener varias patolog\u00edas de base gen\u00e9tica, pero esto es el MIR y adem\u00e1s nos dicen que la paciente no tiene discapacidad intelectual. En el MIR, una adolescente con retraso puberal y talla baja debe ser un Turner mientras no se demuestre lo contrario (hubo dos preguntas en el a\u00f1o 2017). Por tanto, si \u00e9sa es nuestra primera sospecha diagn\u00f3stica, la prueba HABITUAL (como dice el enunciado) para el diagn\u00f3stico debe ser un cariotipo convencional (f\u00f3rmula: 45, X0). Tambi\u00e9n se puede diagnosticar un Turner con un FISH (s\u00f3lo habr\u00eda una se\u00f1al para el X en vez de las dos que tienen habitualmente las mujeres) o con un array. Sin embargo, tanto el array, como el NGS lo reservamos habitualmente para pacientes con cuadros cl\u00ednicos no evidentes, discapacidad intelectual, o sospecha de alg\u00fan s\u00edndrome gen\u00e9tico que precise de estas pruebas concretas (por ejemplo, un array para diagn\u00f3sticar un s\u00edndrome de deleci\u00f3n 22q11 o un panel NGS para diagnosticar un s\u00edndrome de Noonan).", "type": "GEN\u00c9TICA", "options": {"1": "Secuenciaci\u00f3n masiva (NGS).", "2": "FISH.", "3": "Microarrays de ADN y/o de ARN.", "4": "Cariotipo.", "5": null}, "correct_option": 4, "explanations": {"1": {"exist": true, "char_ranges": [[825, 1033]], "word_ranges": [[141, 171]], "text": "tanto el array, como el NGS lo reservamos habitualmente para pacientes con cuadros cl\u00ednicos no evidentes, discapacidad intelectual, o sospecha de alg\u00fan s\u00edndrome gen\u00e9tico que precise de estas pruebas concretas"}, "2": {"exist": true, "char_ranges": [[656, 795]], "word_ranges": [[109, 135]], "text": "Tambi\u00e9n se puede diagnosticar un Turner con un FISH (s\u00f3lo habr\u00eda una se\u00f1al para el X en vez de las dos que tienen habitualmente las mujeres)"}, "3": {"exist": true, "char_ranges": [[825, 1033]], "word_ranges": [[141, 171]], "text": "tanto el array, como el NGS lo reservamos habitualmente para pacientes con cuadros cl\u00ednicos no evidentes, discapacidad intelectual, o sospecha de alg\u00fan s\u00edndrome gen\u00e9tico que precise de estas pruebas concretas"}, "4": {"exist": true, "char_ranges": [[335, 654]], "word_ranges": [[57, 109]], "text": "una adolescente con retraso puberal y talla baja debe ser un Turner mientras no se demuestre lo contrario (hubo dos preguntas en el a\u00f1o 2017). Por tanto, si \u00e9sa es nuestra primera sospecha diagn\u00f3stica, la prueba HABITUAL (como dice el enunciado) para el diagn\u00f3stico debe ser un cariotipo convencional (f\u00f3rmula: 45, X0)."}, "5": {"exist": false, "char_ranges": [], "word_ranges": [], "text": ""}}} +{"id": 216, "year": 2014, "question_id_specific": 121, "full_question": "Hombre de 38 a\u00f1os que consulta por disnea y hemoptisis. En los an\u00e1lisis de sangre tiene creatinina 7 mg/dl, urea 250 mg/dl y anti-MBG (anticuerpos anti- membrana basal glomerular) positivos a t\u00edtulo alto. Se realiza biopsia renal que muestra semilunas en el 75% de los glom\u00e9rulos y en la inmunofluorescencia aparece un patr\u00f3n dep\u00f3sito lineal de Ig. \u00bfCu\u00e1l de las siguientes es la respuesta correcta?", "full_answer": "La sospecha diagn\u00f3stica por los datos que nos dan (sobre todo los anticuerpos anti- MBG y la presencia de s\u00edndrome renopulmonar) es la de un s\u00edndrome de Goodpasture. El resto de datos (seminulas en la biopsia, dep\u00f3sito lineal IgG) apoya el diagn\u00f3stico. Sabiendo esto, las opciones se descartan f\u00e1cilmente: la opci\u00f3n 1 nos habla de una nefropat\u00eda IgA (no es el caso porque los dep\u00f3sitos son de IgG); la opci\u00f3n 3 tambi\u00e9n queda descartada, al no tratarse de una GMN primaria, como la membranosa, sino de una glomerulopat\u00eda secundaria; la opci\u00f3n 4 tambi\u00e9n es falsa: el tratamiento inicial se realiza con corticoides y ciclofosfamida asociados a plasmaf\u00e9resis; y la opci\u00f3n 5 es igualmente falsa, ya que el da\u00f1o no es por inmunocomplejos circulantes, sino por anticuerpos depositados en la membrana basal glomerular. Esto nos deja la opci\u00f3n 2 como verdadera: como se ha dicho anteriormente, el tratamiento se realizar\u00eda combinando corticoides, ciclofosfamida y plasmaf\u00e9resis.", "type": "NEFROLOG\u00cdA", "options": {"1": "Se trata de una Nefropat\u00eda IgA con fracaso renal agudo.", "2": "Estar\u00eda indicada la realizaci\u00f3n de plasmaf\u00e9resis.", "3": "Se trata de una glomerulonefritis membranosa.", "4": "El micofenolato mofetilo es el tratamiento inicial de elecci\u00f3n.", "5": "La afectaci\u00f3n glomerular est\u00e1 causada por la presencia de inmunocomplejos circulantes."}, "correct_option": 2, "explanations": {"1": {"exist": true, "char_ranges": [[306, 398]], "word_ranges": [[49, 68]], "text": "la opci\u00f3n 1 nos habla de una nefropat\u00eda IgA (no es el caso porque los dep\u00f3sitos son de IgG);"}, "2": {"exist": true, "char_ranges": [[0, 165]], "word_ranges": [[0, 28]], "text": "La sospecha diagn\u00f3stica por los datos que nos dan (sobre todo los anticuerpos anti- MBG y la presencia de s\u00edndrome renopulmonar) es la de un s\u00edndrome de Goodpasture."}, "3": {"exist": true, "char_ranges": [[399, 531]], "word_ranges": [[68, 89]], "text": "la opci\u00f3n 3 tambi\u00e9n queda descartada, al no tratarse de una GMN primaria, como la membranosa, sino de una glomerulopat\u00eda secundaria;"}, "4": {"exist": true, "char_ranges": [[532, 655]], "word_ranges": [[89, 107]], "text": "la opci\u00f3n 4 tambi\u00e9n es falsa: el tratamiento inicial se realiza con corticoides y ciclofosfamida asociados a plasmaf\u00e9resis;"}, "5": {"exist": true, "char_ranges": [[659, 810]], "word_ranges": [[108, 132]], "text": "la opci\u00f3n 5 es igualmente falsa, ya que el da\u00f1o no es por inmunocomplejos circulantes, sino por anticuerpos depositados en la membrana basal glomerular."}}} +{"id": 384, "year": 2016, "question_id_specific": 151, "full_question": "Una mujer se presenta en la consulta con su hija de 3 a\u00f1os porque le ha detectado ligero desarrollo mamario desde hace 3 meses sin tomar medicaci\u00f3n alguna ni objetivarse antecedentes relevantes en la historia. Efectivamente, el examen f\u00edsico muestra un estadio Tanner IV, sin crecimiento del vello p\u00fabico o axilar. Los genitales externos son normales. La ecograf\u00eda revela un peque\u00f1o \u00fatero y la radiolog\u00eda una edad \u00f3sea de 3 a\u00f1os. \u00bfQu\u00e9 actitud se deber\u00eda adoptar?", "full_answer": "Esta pregunta est\u00e1 regular planteada, demasiado desarrollo para unas exploraciones complementarias normales\u2026 Parece que nos quieren presentar una pubertad precoz (o una telarquia prematura) pero no aportan ning\u00fan dato anal\u00edtico y los datos de la ecograf\u00eda son ambiguos (habr\u00eda que dar por supuesto que con un peque\u00f1o \u00fatero se est\u00e1n refiriendo a un \u00fatero prepuberal, pero no aportan datos a cerca del tama\u00f1o ov\u00e1rico). Nos presentan el caso de una ni\u00f1a con desarrollo mamario avanzado de tres a\u00f1os de edad, en principio sin ninguna causa asociada (en principio no toma f\u00e1rmacos que puedan aumentar el nivel de estr\u00f3geno en sangre, no parece usar cremas corporales ni ingesta importante de carne de pollo). Si seguimos el esquema diagn\u00f3stico ante una telarquia prematura o sospecha de pubertad precoz, se solicita edad \u00f3sea y ecograf\u00eda abdominal (la EO no est\u00e1 adelantada como ocurre en la pubertad precoz, y damos por supuesto que con un \u00fatero peque\u00f1o se quieren referir a un \u00fatero prepuberal); seg\u00fan las exploraciones complementarias que nos dan, no parece una pubertad precoz, salvo la cl\u00ednica (Tanner IV). Estrictamente, sin datos hormonales anal\u00edticos, parece que podr\u00edamos marcar la opci\u00f3n 1, siendo necesario hacer un seguimiento estrecho de la ni\u00f1a. Si damos por hecho todos los datos anteriores, podr\u00edamos descartar la opci\u00f3n 4, que ser\u00eda el tratamiento de una pubertad precoz central. Respecto a la opci\u00f3n de mamograf\u00eda, en pediatr\u00eda se usa la ecograf\u00eda mamaria, y en este caso estar\u00eda indicada si nos dijesen que existe una asimetr\u00eda mamaria (descartamos la opci\u00f3n 3). Respecto a la biopsia mamaria \u00fanicamente estar\u00eda indicada ante signos de alarma.", "type": "PEDIATR\u00cdA", "options": {"1": "Seguimiento cada 3-4 meses, ya que se trata de una condici\u00f3n temporal que a menudo se resuelve sola.", "2": "Biopsia mamaria.", "3": "Mamograf\u00eda.", "4": "Administraci\u00f3n de an\u00e1logos GnRh.", "5": null}, "correct_option": 1, "explanations": {"1": {"exist": true, "char_ranges": [[704, 1075]], "word_ranges": [[112, 172]], "text": "Si seguimos el esquema diagn\u00f3stico ante una telarquia prematura o sospecha de pubertad precoz, se solicita edad \u00f3sea y ecograf\u00eda abdominal (la EO no est\u00e1 adelantada como ocurre en la pubertad precoz, y damos por supuesto que con un \u00fatero peque\u00f1o se quieren referir a un \u00fatero prepuberal); seg\u00fan las exploraciones complementarias que nos dan, no parece una pubertad precoz,"}, "2": {"exist": true, "char_ranges": [[1577, 1657]], "word_ranges": [[251, 263]], "text": "Respecto a la biopsia mamaria \u00fanicamente estar\u00eda indicada ante signos de alarma."}, "3": {"exist": true, "char_ranges": [[1392, 1576]], "word_ranges": [[220, 251]], "text": "Respecto a la opci\u00f3n de mamograf\u00eda, en pediatr\u00eda se usa la ecograf\u00eda mamaria, y en este caso estar\u00eda indicada si nos dijesen que existe una asimetr\u00eda mamaria (descartamos la opci\u00f3n 3)."}, "4": {"exist": true, "char_ranges": [[704, 1075]], "word_ranges": [[112, 172]], "text": "Si seguimos el esquema diagn\u00f3stico ante una telarquia prematura o sospecha de pubertad precoz, se solicita edad \u00f3sea y ecograf\u00eda abdominal (la EO no est\u00e1 adelantada como ocurre en la pubertad precoz, y damos por supuesto que con un \u00fatero peque\u00f1o se quieren referir a un \u00fatero prepuberal); seg\u00fan las exploraciones complementarias que nos dan, no parece una pubertad precoz,"}, "5": {"exist": false, "char_ranges": [], "word_ranges": [], "text": ""}}} +{"id": 238, "year": 2014, "question_id_specific": 145, "full_question": "Mujer de 40 a\u00f1os que consulta por aproximadamente 20 episodios al d\u00eda de dolor intenso, periocular izquierdo de 15 minutos de duraci\u00f3n, acompa\u00f1ado de intenso lagrimeo y rinorrea. Su exploraci\u00f3n y resonancia magn\u00e9tica son normales. Su tratamiento de elecci\u00f3n ser\u00eda:", "full_answer": "En esta pregunta nos presentan un caso para deducir un diagn\u00f3stico y luego indicar cual ser\u00eda el tratamiento de elecci\u00f3n. Tanto por las caracter\u00edsticas del caso como por las respuestas se deduce que se trata de una cefalea trig\u00e9mino-auton\u00f3mica (dolor intenso, periocular con lagrimeo y rinorrea). La idea es realizar un diagn\u00f3stico diferencial fundamentalmente entre una cefalea en racimos (cuyo tratamientos incluyen las respuestas 2, 3, 4 y 5), una hemicranea parox\u00edstica (cuyo tratamiento de elecci\u00f3n es la indometacina de la respuesta 1) y un SUNCT (Cefalea Neuralgiforme unilateral con inyecci\u00f3n conjuntival y lagrimeo). La cefalea en racimos predomina en varones cuya duraci\u00f3n puede variar entre 15-180 minutos, entre una vez cada 2 d\u00edas, hasta 8 veces al d\u00eda. La hemicranea parox\u00edstica predomina en la mujer, con episodios de dolor similares a la cefalea en racimos, pero con una duraci\u00f3n m\u00e1s breve (2-30 min), y una frecuencia mayor (5-30 episodios al d\u00eda). En cuanto al SUNCT las crisis son mucho m\u00e1s breves, de segundos de duraci\u00f3n (5-240 segundos de duraci\u00f3n) y suele ser refractaria a tratamiento. Por lo tanto el diagn\u00f3stico ser\u00eda de una hemicranea parox\u00edstica y su tratamiento de elecci\u00f3n es la indometacina (que adem\u00e1s la respuesta es un criterio diagn\u00f3stico). Con lo cual respuesta correcta 1 (Indometacina). Todos los datos ofrecidos se basan en Criterios Diagn\u00f3sticos de la International Headache Society.", "type": "NEUROLOG\u00cdA", "options": {"1": "Indometacina.", "2": "Lamotrigina.", "3": "Verapamilo.", "4": "Prednisona.", "5": "Carbonato de litio."}, "correct_option": 1, "explanations": {"1": {"exist": true, "char_ranges": [[767, 1274]], "word_ranges": [[119, 202]], "text": "La hemicranea parox\u00edstica predomina en la mujer, con episodios de dolor similares a la cefalea en racimos, pero con una duraci\u00f3n m\u00e1s breve (2-30 min), y una frecuencia mayor (5-30 episodios al d\u00eda). En cuanto al SUNCT las crisis son mucho m\u00e1s breves, de segundos de duraci\u00f3n (5-240 segundos de duraci\u00f3n) y suele ser refractaria a tratamiento. Por lo tanto el diagn\u00f3stico ser\u00eda de una hemicranea parox\u00edstica y su tratamiento de elecci\u00f3n es la indometacina (que adem\u00e1s la respuesta es un criterio diagn\u00f3stico)."}, "2": {"exist": true, "char_ranges": [[626, 766]], "word_ranges": [[94, 119]], "text": "La cefalea en racimos predomina en varones cuya duraci\u00f3n puede variar entre 15-180 minutos, entre una vez cada 2 d\u00edas, hasta 8 veces al d\u00eda."}, "3": {"exist": true, "char_ranges": [[626, 766]], "word_ranges": [[94, 119]], "text": "La cefalea en racimos predomina en varones cuya duraci\u00f3n puede variar entre 15-180 minutos, entre una vez cada 2 d\u00edas, hasta 8 veces al d\u00eda."}, "4": {"exist": true, "char_ranges": [[626, 766]], "word_ranges": [[94, 119]], "text": "La cefalea en racimos predomina en varones cuya duraci\u00f3n puede variar entre 15-180 minutos, entre una vez cada 2 d\u00edas, hasta 8 veces al d\u00eda."}, "5": {"exist": true, "char_ranges": [[626, 766]], "word_ranges": [[94, 119]], "text": "La cefalea en racimos predomina en varones cuya duraci\u00f3n puede variar entre 15-180 minutos, entre una vez cada 2 d\u00edas, hasta 8 veces al d\u00eda."}}} +{"id": 47, "year": 2011, "question_id_specific": 157, "full_question": "Ni\u00f1o de 6 a\u00f1os que viene a la consulta acompa\u00f1ado del monitor de un centro de acogida de nuestro barrio por tumoraci\u00f3n dolorosa a la palpaci\u00f3n de 3 cm de di\u00e1metro en zona occipital derecha del cuero cabelludo. Padece alopecia en dicha zona y 3 adenomegalias de consistencia bastante dura en regi\u00f3n cervical posterior derecha. \u00bfQu\u00e9 tratamiento ser\u00eda el m\u00e1s adecuado?", "full_answer": "La respuesta correcta es la 3. Si no me equivoco est\u00e1 describiendo un Querion de Celso para el cual el tratamiento de elecci\u00f3n es la griseofulvina v\u00eda oral.", "type": "PEDIATR\u00cdA", "options": {"1": "Incisi\u00f3n y drenaje.", "2": "Mupirocina t\u00f3pica.", "3": "Griseofulvina por v\u00eda oral.", "4": "Cefazolina intravenosa.", "5": "Ketoconazol t\u00f3pico."}, "correct_option": 3, "explanations": {"1": {"exist": false, "char_ranges": [], "word_ranges": [], "text": ""}, "2": {"exist": false, "char_ranges": [], "word_ranges": [], "text": ""}, "3": {"exist": true, "char_ranges": [[31, 156]], "word_ranges": [[6, 28]], "text": "Si no me equivoco est\u00e1 describiendo un Querion de Celso para el cual el tratamiento de elecci\u00f3n es la griseofulvina v\u00eda oral."}, "4": {"exist": false, "char_ranges": [], "word_ranges": [], "text": ""}, "5": {"exist": false, "char_ranges": [], "word_ranges": [], "text": ""}}} +{"id": 381, "year": 2016, "question_id_specific": 139, "full_question": "Mujer de 70 a\u00f1os con antecedente de anorexia,p\u00e9rdida de peso, molestias en la musculatura y articulaciones proximales m\u00e1s dolorimiento en la regi\u00f3n temporo-mandibular que acude al servicio de urgencias por p\u00e9rdida de visi\u00f3n unilateral (movimiento de manos), de aparici\u00f3n brusca e indolora (defecto pupilar aferente).\u00bfQu\u00e9 prueba solicitar\u00eda en primer lugar con fines diagn\u00f3sticos?", "full_answer": "Nos presentan el caso de una amaurosis monocular de perfil vascular, adem\u00e1s en una mujer mayor con antecedentes de p\u00e9rdida de peso y lo que parecen s\u00edntomas de claudicaci\u00f3n mandibular y polimialgia reum\u00e1tica, por lo que la primera causa que hay que pensar es en una arteritis de c\u00e9lulas gigantes como causante de la NOIA, por lo que de las opciones la correcta ser\u00eda la 2. La 4 nos la plantear\u00edamos si nos contaran una NOIA pero de caracter\u00edsticas no arter\u00edticas (sin toda esa sintomatolog\u00eda acompa\u00f1ante).", "type": "NEUROLOG\u00cdA", "options": {"1": "Punci\u00f3n lumbar.", "2": "Prote\u00edna C Reactiva.", "3": "Angio Resonancia Magn\u00e9tica.", "4": "Ecograf\u00eda carot\u00eddea.", "5": null}, "correct_option": 2, "explanations": {"1": {"exist": false, "char_ranges": [], "word_ranges": [], "text": ""}, "2": {"exist": true, "char_ranges": [[220, 372]], "word_ranges": [[36, 66]], "text": "la primera causa que hay que pensar es en una arteritis de c\u00e9lulas gigantes como causante de la NOIA, por lo que de las opciones la correcta ser\u00eda la 2."}, "3": {"exist": false, "char_ranges": [], "word_ranges": [], "text": ""}, "4": {"exist": true, "char_ranges": [[373, 505]], "word_ranges": [[66, 86]], "text": "La 4 nos la plantear\u00edamos si nos contaran una NOIA pero de caracter\u00edsticas no arter\u00edticas (sin toda esa sintomatolog\u00eda acompa\u00f1ante)."}, "5": {"exist": false, "char_ranges": [], "word_ranges": [], "text": ""}}} +{"id": 32, "year": 2011, "question_id_specific": 64, "full_question": "En un paciente diagnosticado de epilepsia que presenta episodios de falta de respuesta a est\u00edmulos externos, movimientos irregulares de las cuatro extremidades, ojos cerrados, emisi\u00f3n de llanto y movimientos p\u00e9lvicos, de cinco a veinte segundos de duraci\u00f3n y que no responden al tratamiento con f\u00e1rmacos antiepil\u00e9pticos. \u00bfCu\u00e1l es el estudio complementario con mayor probabilidad de aclarar el diagn\u00f3stico?", "full_answer": "Respuesta 1 correcta: El cuadro descrito es muy sugestivo de pseudocrisis con movimientos as\u00edncronos de las extremidades, movimientos p\u00e9lvicos, llanto y mala respuesta a f\u00e1rmacos antiepil\u00e9pticos.", "type": "NEUROLOG\u00cdA Y NEUROCIRUG\u00cdA", "options": {"1": "Monitorizaci\u00f3n v\u00eddeo-EEG para diagn\u00f3stico de pseudocrisis (crisis psic\u00f3genas).", "2": "Holter ECG para diagn\u00f3stico de cardiopat\u00eda arr\u00edtmica.", "3": "EEG de rutina para diagn\u00f3stico del tipo de epilepsia (generalizada o foc).", "4": "Resonancia magn\u00e9tica cerebral para detectar lesiones epilept\u00f3genas (displasia cortical, tumor, esclerosis temporal medial).", "5": "Determinaci\u00f3n de glucemia capilar para diagn\u00f3stico de hipoglucemia."}, "correct_option": 1, "explanations": {"1": {"exist": true, "char_ranges": [[22, 195]], "word_ranges": [[3, 26]], "text": "El cuadro descrito es muy sugestivo de pseudocrisis con movimientos as\u00edncronos de las extremidades, movimientos p\u00e9lvicos, llanto y mala respuesta a f\u00e1rmacos antiepil\u00e9pticos."}, "2": {"exist": false, "char_ranges": [], "word_ranges": [], "text": ""}, "3": {"exist": false, "char_ranges": [], "word_ranges": [], "text": ""}, "4": {"exist": false, "char_ranges": [], "word_ranges": [], "text": ""}, "5": {"exist": false, "char_ranges": [], "word_ranges": [], "text": ""}}} +{"id": 106, "year": 2012, "question_id_specific": 135, "full_question": "Ni\u00f1o de 2 a\u00f1os. En sus antecedentes personales destacan: 3 episodios de otitis media aguda, 1 meningitis meningoc\u00f3cica y 2 neumon\u00edas (una de l\u00f3bulo medio y otra de l\u00f3bulo superior izquierdo). Ha ingresado en 3 ocasiones por p\u00farpura trombop\u00e9nica (en tres ocasiones los anticuerpos antiplaquetas fueron negativos y en la m\u00e9dula \u00f3seas se observaban megacariocitos normales). Varios varones de la familia materna hab\u00edan fallecido en la infancia por procesos infecciosos. En la exploraci\u00f3n f\u00edsica presenta lesiones t\u00edpicas de dermatitis at\u00f3pica. En el estudio inmunol\u00f3gico destaca una leve disminuci\u00f3n de subpoblaciones de linfocitos T; elevaci\u00f3n de IgA e IgE; disminuci\u00f3n de IgM y IgG en el l\u00edmite inferior de la normalidad. \u00bfCu\u00e1l es el diagn\u00f3stico m\u00e1s probable?", "full_answer": "Para contestar correctamente a esta pregunta hay que resaltar que de entre la sintomatolog\u00eda que se expone se encuentran: \u2022 Ni\u00f1o de 2 a\u00f1os. \u2022 Infecciones ORL. \u2022 Infecciones Pulmonares. \u2022 Ingresos Hospitalarios. \u2022 PTI. \u2022 Historia Familiar de infecciones con muertes por infecciones en Varones (familia materna). \u2022 Dermatitis At\u00f3pica. Todos estos datos, en conjunto, adem\u00e1s de los anal\u00edticos, est\u00e1n describiendo una inmunodeficiencia que por la historia familiar-materna, parece ligada al X, al ser varios los varones fallecidos por una cl\u00ednica similar (el padre del paciente aport\u00f3 el cromosoma Y, la madre el X). Dentro de las inmunodeficiencias ligadas al X se encuentra el S\u00edndrome de Wiskott-Aldrich, una entidad que se describe con una tr\u00edada inicial de s\u00edntomas consistentes en sangrado (t\u00edpicos AUNQUE ausentes en el caso: sangrado abundante tras la realizaci\u00f3n de circuncisi\u00f3n, diarrea sanguinolienta), infecciones de repetici\u00f3n y eczema. Adem\u00e1s de la trombocitopenia presentan un riesgo incrementado para padecer fen\u00f3menos autoinmunes y neoplasias linfoides. A la opci\u00f3n de Hiper-IgE le faltan datos cl\u00ednicos muy caracter\u00edsticos como las alteraciones \u00f3seas y las lesiones cut\u00e1neas, que no son de dermatitis at\u00f3pica, ya que siguen un patr\u00f3n diferente (erupci\u00f3n p\u00e1pulopustular en la cara y el cuero cabelludo. A la opci\u00f3n de Hipogammaglobulinemia transitoria de la infancia y las inmunodeficiencias combinada severa y variable com\u00fan les falla, entre otras caracter\u00edsticas, la determinaci\u00f3n de inmunoglobulinas G y M, que est\u00e1n en el l\u00edmite inferior, pero dentro de la normalidad.", "type": "GEN\u00c9TICA E INMUNOLOG\u00cdA", "options": {"1": "S\u00edndrome de Wiskott-Aldrich.", "2": "S\u00edndrome de hiper IgE.", "3": "Hipogammaglobulinemia transitoria de la infancia.", "4": "Inmunodeficiencia combinada severa ligada al X.", "5": "Inmunodeficiencia variable com\u00fan."}, "correct_option": 4, "explanations": {"1": {"exist": true, "char_ranges": [[613, 945]], "word_ranges": [[95, 142]], "text": "Dentro de las inmunodeficiencias ligadas al X se encuentra el S\u00edndrome de Wiskott-Aldrich, una entidad que se describe con una tr\u00edada inicial de s\u00edntomas consistentes en sangrado (t\u00edpicos AUNQUE ausentes en el caso: sangrado abundante tras la realizaci\u00f3n de circuncisi\u00f3n, diarrea sanguinolienta), infecciones de repetici\u00f3n y eczema."}, "2": {"exist": true, "char_ranges": [[1067, 1315]], "word_ranges": [[157, 197]], "text": "A la opci\u00f3n de Hiper-IgE le faltan datos cl\u00ednicos muy caracter\u00edsticos como las alteraciones \u00f3seas y las lesiones cut\u00e1neas, que no son de dermatitis at\u00f3pica, ya que siguen un patr\u00f3n diferente (erupci\u00f3n p\u00e1pulopustular en la cara y el cuero cabelludo."}, "3": {"exist": true, "char_ranges": [[1316, 1585]], "word_ranges": [[197, 237]], "text": "A la opci\u00f3n de Hipogammaglobulinemia transitoria de la infancia y las inmunodeficiencias combinada severa y variable com\u00fan les falla, entre otras caracter\u00edsticas, la determinaci\u00f3n de inmunoglobulinas G y M, que est\u00e1n en el l\u00edmite inferior, pero dentro de la normalidad."}, "4": {"exist": true, "char_ranges": [[391, 612]], "word_ranges": [[60, 95]], "text": "est\u00e1n describiendo una inmunodeficiencia que por la historia familiar-materna, parece ligada al X, al ser varios los varones fallecidos por una cl\u00ednica similar (el padre del paciente aport\u00f3 el cromosoma Y, la madre el X)."}, "5": {"exist": false, "char_ranges": [], "word_ranges": [], "text": ""}}} +{"id": 62, "year": 2011, "question_id_specific": 119, "full_question": "Nos consultan para valorar una mujer de 83 a\u00f1os que ha ingresado en el servicio de Traumatolog\u00eda por una fractura de cadera hace 6 horas. Tiene AP de HTA, DL, demencia moderada y vive en una residencia. Su tratamiento habitual es tiazida, atorvastatina, donepezilo, Calcio y vitamina D. EF: Paciente confusa, pulso 90 lpm, frec respiratoria 20 rpm, T art 170/88, presi\u00f3n venosa yugular normal. La anal\u00edtica y la Rx de t\u00f3rax son normales y el ECG muestra un ritmo sinusal sin alteraciones isqu\u00e9micas. \u00bfCu\u00e1l de las siguientes es la actitud terap\u00e9utica m\u00e1s correcta?", "full_answer": "El cuadro confusional est\u00e1 determinado por la demencia que ya sufre la paciente, as\u00ed que por mucho que retrasemos la cirug\u00eda, no vamos a conseguir nada. La elevaci\u00f3n de la tensi\u00f3n arterial se debe, en principio, al dolor (por lo que la primera opci\u00f3n es un analg\u00e9sico) y, luego, a la situaci\u00f3n de estr\u00e9s que lleva a una paciente ya hipertensa a aumentar su tensi\u00f3n arterial, por lo que el labetalol podr\u00eda ayudarnos. La \u00faltima respuesta no es correcta porque depende del tipo de fractura, que no nos especifican en el enunciado: una pertrocant\u00e9rea, necesitar\u00eda osteos\u00edntesis cerrada y una subcapital, una pr\u00f3tesis.", "type": "ANESTESIOLOG\u00cdA Y CUIDADOS CR\u00cdTICOS", "options": {"1": "Retrasar la cirug\u00eda hasta que haya desaparecido el cuadro confusional.", "2": "Retrasar la cirug\u00eda y realizar un ecocardiograma.", "3": "Retrasar la cirug\u00eda hasta buen control de la tensi\u00f3n arterial.", "4": "Iniciar un betabloqueante e iniciar la cirug\u00eda.", "5": "Realizar osteos\u00edntesis cerrada, evitando en todo caso la implantaci\u00f3n de pr\u00f3tesis."}, "correct_option": 4, "explanations": {"1": {"exist": true, "char_ranges": [[0, 152]], "word_ranges": [[0, 26]], "text": "El cuadro confusional est\u00e1 determinado por la demencia que ya sufre la paciente, as\u00ed que por mucho que retrasemos la cirug\u00eda, no vamos a conseguir nada."}, "2": {"exist": false, "char_ranges": [], "word_ranges": [], "text": ""}, "3": {"exist": true, "char_ranges": [[153, 416]], "word_ranges": [[26, 73]], "text": "La elevaci\u00f3n de la tensi\u00f3n arterial se debe, en principio, al dolor (por lo que la primera opci\u00f3n es un analg\u00e9sico) y, luego, a la situaci\u00f3n de estr\u00e9s que lleva a una paciente ya hipertensa a aumentar su tensi\u00f3n arterial, por lo que el labetalol podr\u00eda ayudarnos."}, "4": {"exist": false, "char_ranges": [], "word_ranges": [], "text": ""}, "5": {"exist": true, "char_ranges": [[417, 614]], "word_ranges": [[73, 102]], "text": "La \u00faltima respuesta no es correcta porque depende del tipo de fractura, que no nos especifican en el enunciado: una pertrocant\u00e9rea, necesitar\u00eda osteos\u00edntesis cerrada y una subcapital, una pr\u00f3tesis."}}} +{"id": 592, "year": 2022, "question_id_specific": 76, "full_question": "Mujer de 40 a\u00f1os que consulta porque desde hace un mes se nota un bulto en el cuadrante superoexterno de la mama derecha. Aporta un informe de una mamograf\u00eda que describe una lesi\u00f3n BIRADS 3. \u00bfCu\u00e1l es la actitud a seguir?:", "full_answer": "BI-RADS Breast Imaging Reporting and Data System. BI-RADS 3 se define con la respuesta 3.", "type": "ONCOLOG\u00cdA", "options": {"1": "Tranquilizarle, puesto que ya se ha hecho una prueba de imagen y se ha descartado malignidad.", "2": "Esa clasificaci\u00f3n implica probablemente cirug\u00eda dado que la probabilidad de c\u00e1ncer es mayor del 10 %. Se lo explica y le deriva de forma preferente a la Unidad de Mama.", "3": "Se trata de un hallazgo probablemente benigno, dado que hay menos de un 2 % de probabilidad de c\u00e1ncer. Le explica que requiere de un seguimiento cada 6-12 meses hasta los 24 meses o una biopsia.", "4": "Los hallazgos son de baja sospecha de c\u00e1ncer (entre un 2 y un 10 %) pero es necesario hacer una biopsia.", "5": null}, "correct_option": 3, "explanations": {"1": {"exist": false, "char_ranges": [], "word_ranges": [], "text": ""}, "2": {"exist": false, "char_ranges": [], "word_ranges": [], "text": ""}, "3": {"exist": true, "char_ranges": [[0, 89]], "word_ranges": [[0, 15]], "text": "BI-RADS Breast Imaging Reporting and Data System. BI-RADS 3 se define con la respuesta 3."}, "4": {"exist": false, "char_ranges": [], "word_ranges": [], "text": ""}, "5": {"exist": false, "char_ranges": [], "word_ranges": [], "text": ""}}} +{"id": 26, "year": 2011, "question_id_specific": 117, "full_question": "Un paciente inmunodeficiente que presenta en la radiograf\u00eda de t\u00f3rax/TC una neumon\u00eda con el signo del halo menisco o contorno semilunar sugiere infecci\u00f3n por:", "full_answer": "El Signo del halo es caracter\u00edstico de la aspergilosis pulmonar, y m\u00e1s en un paciente inmuodeprimido. Pero no es patognom\u00f3nico, tambi\u00e9n se ha asociado a TBC, algunas neoplasias y la granulomatosis de Wegener.", "type": "INFECCIOSAS", "options": {"1": "Staphylococcus aureus.", "2": "Streptococcus pneumoniae.", "3": "Candida albicans.", "4": "Pseudomonas aeruginosa.", "5": "Aspergillus fumigatus."}, "correct_option": 5, "explanations": {"1": {"exist": false, "char_ranges": [], "word_ranges": [], "text": ""}, "2": {"exist": false, "char_ranges": [], "word_ranges": [], "text": ""}, "3": {"exist": false, "char_ranges": [], "word_ranges": [], "text": ""}, "4": {"exist": false, "char_ranges": [], "word_ranges": [], "text": ""}, "5": {"exist": true, "char_ranges": [[0, 101]], "word_ranges": [[0, 16]], "text": "El Signo del halo es caracter\u00edstico de la aspergilosis pulmonar, y m\u00e1s en un paciente inmuodeprimido."}}} +{"id": 435, "year": 2018, "question_id_specific": 121, "full_question": "Hombre de 45 a\u00f1os que consulta por tos productiva, dolor pleur\u00edtico en costado derecho y fiebre de 48 h de evoluci\u00f3n. Se aprecia una saturaci\u00f3n basal de O2 del 88% y estertores en base derecha. La RX de t\u00f3rax muestra una consolidaci\u00f3n basal derecha. Tiene antecedentes de infecci\u00f3n por VIH bien controlada con antirretrovirales linfocitos CD4 550 ce/uL y carga viral de VlH indetectable). \u00bfCual de los tratamientos antimicrobianos emp\u00edricos que se se\u00f1alan a continuaci\u00f3n le parece mas adecuado?", "full_answer": "La primera opci\u00f3n parece ser la correcta, por la baja probabilidad de infecci\u00f3n por P. jirovecii en paciente con m\u00e1s de 500 linfocitos CD4 y carga viral indetectable. Lo m\u00e1s probable es que sea infecci\u00f3n por neumococo que cubrimos con Ceftriaxona y con azitromicina cubrimos las llamadas \u201cat\u00edpicas\u201d. Meropenem es un antibi\u00f3tico de espectro demasiado amplio, que tendr\u00eda opci\u00f3n en una neumon\u00eda intrahospitalaria por la P. aeruginosa, agente bacteriano que tambi\u00e9n puede causar neumon\u00eda a pacientes VIH, pero no es lo frecuente.", "type": "ENFERMEDADES INFECCIOSAS Y MICROBIOLOG\u00cdA", "options": {"1": "Cefiriaxona 2 g y azitromicina 500 mg cada 24 horas.", "2": "Cefiriaxona 2 g, azitrornicina 500 mg cada 24 horas y trimetoprim-sulfametoxazol 5 mg/kg/8 h (basado en dosis de trimetoprim).", "3": "Metil-prednisolona 40 mgd\u00eda, cefiriaxona 2 g IV 124 h y trimetoprim-sulfametoxazol 5 mg/kg/8 h (basado en dosis de trimetoprim).", "4": "Meropenem I g/8 h y vancomicina I g/l2 h.", "5": null}, "correct_option": 1, "explanations": {"1": {"exist": true, "char_ranges": [[167, 299]], "word_ranges": [[28, 48]], "text": "Lo m\u00e1s probable es que sea infecci\u00f3n por neumococo que cubrimos con Ceftriaxona y con azitromicina cubrimos las llamadas \u201cat\u00edpicas\u201d."}, "2": {"exist": false, "char_ranges": [], "word_ranges": [], "text": ""}, "3": {"exist": false, "char_ranges": [], "word_ranges": [], "text": ""}, "4": {"exist": true, "char_ranges": [[300, 526]], "word_ranges": [[48, 82]], "text": "Meropenem es un antibi\u00f3tico de espectro demasiado amplio, que tendr\u00eda opci\u00f3n en una neumon\u00eda intrahospitalaria por la P. aeruginosa, agente bacteriano que tambi\u00e9n puede causar neumon\u00eda a pacientes VIH, pero no es lo frecuente."}, "5": {"exist": false, "char_ranges": [], "word_ranges": [], "text": ""}}} +{"id": 474, "year": 2020, "question_id_specific": 142, "full_question": "Mujer de 70 a\u00f1os, diab\u00e9tica e hipertensa que sufre una ca\u00edda en su domicilio, presentando una herida de 9 cm que comunica con un foco de fractura de la tibia derecha. Radiogr\u00e1ficamente se observa una fractura oblicua corta de tercio medio-distal de tibia. Se opera de urgencias mediante limpieza (Friederich) y colocaci\u00f3n de un clavo endomedular acerrojado. A los 11 meses presenta una pseudoartrosis atr\u00f3fica de tibia con supuraci\u00f3n en la zona de la herida. \u00bfCu\u00e1l ser\u00e1 su mejor opci\u00f3n terap\u00e9utica inmediata?:", "full_answer": "Lo primero es tratar la infecci\u00f3n y para ello debemos retirar todo el material de osteos\u00edntesis, desbridar y dar antibioterapia. La fijaci\u00f3n en este caso es mejor externa para evitar todo el material alrededor de la zona afectada por lo que 4 es correcta y 1 y 2 no. La 3 se plantear\u00eda despu\u00e9s de vencido el cuadro infeccioso. TRATAMIENTO \u2022 Tratamiento antibi\u00f3tico supresivo: indicado en pacientes de tipo C de Ciemy, consiste en antibioterapia oral prolongada durante al menos 6 meses para \u00abenfriar\u00bb la reagudizaci\u00f3n del cuadro cl\u00ednico. \u2022 Tratamiento curativo: incluye una primera cirug\u00eda con desbridamiento agresivo, de tipo tumoral, de todos los tejidos afectos, irrigaci\u00f3n profusa, estabilizaci\u00f3n con fijador externo si la estabilidad est\u00e1 comprometida da y eventual relleno de las cavidades con sustancias liberadoras de antibi\u00f3ticos. Tras per\u00edodos prolongados de antibioterapia seleccionada seg\u00fan los antibiogramas de los cultivos, y una vez que se tiene la certeza de haber curado la infecci\u00f3n, se han de plantear t\u00e9cnicas especiales para la reconstrucci\u00f3n del defecto \u00f3seo y para conseguir una adecuada cobertura de partes blandas. Estamos ante una infecci\u00f3n asociada a implante (clavo intramedular) y pseudoartrosis de la fractura de tibia (ausencia de uni\u00f3 tras 11 meses). La infecci\u00f3n se entiende por la cl\u00ednica (supuraci\u00f3n, ausencia de uni\u00f3n) y el antecedente de diabetes y fractura abierta. El manejo se solapa con el del manejo de una osteomielitis cr\u00f3nica. La antibioterapia aislada (opci\u00f3n 2) est\u00e1 indicada s\u00f3lo en pacientes con grave comorbilidad en los cuales el tratamiento quir\u00fargico ser\u00eda m\u00e1s agresivo que continuar con la enfermedad. La dinamizaci\u00f3n del clavo asociado a antibioterapia de amplio espectro (opci\u00f3n 1) tampoco, porque (a) no ha demostrado beneficio en pseudoartrosis establecidas y (b) tenemos el mismo tema que la opci\u00f3n 1, no eliminamos biofilm. La opci\u00f3n 3, discutida, se plantear\u00eda s\u00f3lo si no hubiera infecci\u00f3n asociada.", "type": "CIRUG\u00cdA ORTOP\u00c9DICA Y TRAUMATOLOG\u00cdA", "options": {"1": "Triple antibioterapia (grampositivos, gramnegativos y anaerobios) y limpieza de la herida quir\u00fargica, retirando los cerrojos distales para favorecer la consolidaci\u00f3n \u00f3sea.", "2": "Actitud expectante y tratamiento antibi\u00f3tico con quinolonas.", "3": "Aporte de injerto aut\u00f3logo y de factores de crecimiento (BMP 2 y 7) para estimular el proceso de consolidaci\u00f3n \u00f3sea, que est\u00e1 retardando.", "4": "Retirada del clavo, desbridamiento, colocaci\u00f3n de fijador externo y antibioterapia ajustada a los resultados de los cultivos.", "5": null}, "correct_option": 4, "explanations": {"1": {"exist": true, "char_ranges": [[129, 266]], "word_ranges": [[20, 49]], "text": "La fijaci\u00f3n en este caso es mejor externa para evitar todo el material alrededor de la zona afectada por lo que 4 es correcta y 1 y 2 no."}, "2": {"exist": true, "char_ranges": [[129, 266]], "word_ranges": [[20, 49]], "text": "La fijaci\u00f3n en este caso es mejor externa para evitar todo el material alrededor de la zona afectada por lo que 4 es correcta y 1 y 2 no."}, "3": {"exist": true, "char_ranges": [[267, 326]], "word_ranges": [[49, 59]], "text": "La 3 se plantear\u00eda despu\u00e9s de vencido el cuadro infeccioso."}, "4": {"exist": true, "char_ranges": [[129, 266]], "word_ranges": [[20, 49]], "text": "La fijaci\u00f3n en este caso es mejor externa para evitar todo el material alrededor de la zona afectada por lo que 4 es correcta y 1 y 2 no."}, "5": {"exist": false, "char_ranges": [], "word_ranges": [], "text": ""}}} +{"id": 17, "year": 2011, "question_id_specific": 137, "full_question": "Una paciente de 14 a\u00f1os de edad en buen estado general presenta desde hace 4 dias una erupci\u00f3n cut\u00e1nea generalizada muy pruriginosa formada por placas eritemato-edematosas de entre 2 y 15 cm de di\u00e1metro sin descamaci\u00f3n con tendencia a adquirir una morfolog\u00eda anular que individualmente desaparecen en menos de 24 horas. Las mucosas est\u00e1n respetadas. Su primera impresi\u00f3n diagn\u00f3stica ser\u00eda:", "full_answer": "Urticaria Aguda: se caracteriza por lesiones eritemato-edematosas, evanescentes pruriginosas, que duran menos de 24 horas, sin descamaci\u00f3n. El estado general suele estar conservado. En los ni\u00f1os es m\u00e1s frecuente el patr\u00f3n anular. Tanto en la rubeola como en la toxicodermia, el estado general no esta conservado. El Shock Estafiloc\u00f3cico, presenta afectaci\u00f3n del estado general y adem\u00e1s las lesiones son ampollosas. Las lesiones de la Escabiosis son interdigitales preferentemente, en forma de papulocostras que pueden seguir trayectos lineales.", "type": "DERMATOLOG\u00cdA", "options": {"1": "Urticaria.", "2": "Rubeola.", "3": "Toxicodermia.", "4": "Shock t\u00f3xico estafiloc\u00f3cico.", "5": "Escabiosis."}, "correct_option": 1, "explanations": {"1": {"exist": true, "char_ranges": [[0, 312]], "word_ranges": [[0, 46]], "text": "Urticaria Aguda: se caracteriza por lesiones eritemato-edematosas, evanescentes pruriginosas, que duran menos de 24 horas, sin descamaci\u00f3n. El estado general suele estar conservado. En los ni\u00f1os es m\u00e1s frecuente el patr\u00f3n anular. Tanto en la rubeola como en la toxicodermia, el estado general no esta conservado."}, "2": {"exist": false, "char_ranges": [], "word_ranges": [], "text": ""}, "3": {"exist": false, "char_ranges": [], "word_ranges": [], "text": ""}, "4": {"exist": false, "char_ranges": [], "word_ranges": [], "text": ""}, "5": {"exist": false, "char_ranges": [], "word_ranges": [], "text": ""}}} +{"id": 425, "year": 2018, "question_id_specific": 92, "full_question": "Mujer de 34 a\u00f1os que ingresa para estudio de poliuria y polidipsia. En las primeras 24 horas de ingreso se constata una diuresis de 8,2 litros y se obtiene una anal\u00edtica que muestra una glicemia de 96 mg/dL, natremia de 148 mEq/L y osmolalidad plasmatica de 309 mOsm/kg con osmolalidad urinaria de 89 mOsmlkg. \u00bfQu\u00e9 prueba diagn\u00f3stica debe realizarse a continuaci\u00f3n?", "full_answer": "Nos encontramos ante poliuria. Inicialmente descartamos diabetes mellitus (nuestra paciente tiene una glucemia normal, de 96mgr/dl). Los datos anal\u00edticos nos orientan a diabetes ins\u00edpida (osm en plasma alta con osm urinaria baja). Ahora debemos diferenciar entre diabetes ins\u00edpida central (falta de ADH) o nefrog\u00e9nica (la ADH no ejerce su acci\u00f3n a nivel renal). Ello se consigue mediante el test de vasopresina (administraci\u00f3n intravenosa de ADH y medir de nuevo la osmolaridad urinaria). Por tanto, la respuesta correcta es la opci\u00f3n 3.", "type": "ENDOCRINOLOG\u00cdA", "options": {"1": "Test de infusi\u00f3n de suero salino hipertonico para determinaci\u00f3n seriada de hormona antidiur\u00e9tica.", "2": "Test de deshidrataci\u00f3n (test de Miller).", "3": "Administraci\u00f3n de desmopresina con control seriado de osmolalidad en orina.", "4": "Determinaci\u00f3n de hormona antidiur\u00e9tica en plasma.", "5": null}, "correct_option": 3, "explanations": {"1": {"exist": false, "char_ranges": [], "word_ranges": [], "text": ""}, "2": {"exist": false, "char_ranges": [], "word_ranges": [], "text": ""}, "3": {"exist": true, "char_ranges": [[133, 537]], "word_ranges": [[16, 81]], "text": "Los datos anal\u00edticos nos orientan a diabetes ins\u00edpida (osm en plasma alta con osm urinaria baja). Ahora debemos diferenciar entre diabetes ins\u00edpida central (falta de ADH) o nefrog\u00e9nica (la ADH no ejerce su acci\u00f3n a nivel renal). Ello se consigue mediante el test de vasopresina (administraci\u00f3n intravenosa de ADH y medir de nuevo la osmolaridad urinaria). Por tanto, la respuesta correcta es la opci\u00f3n 3."}, "4": {"exist": false, "char_ranges": [], "word_ranges": [], "text": ""}, "5": {"exist": false, "char_ranges": [], "word_ranges": [], "text": ""}}} +{"id": 8, "year": 2011, "question_id_specific": 232, "full_question": "Mujer de 52 a\u00f1os que consulta copor haber notado durante la semana previa coloraci\u00f3n amarillenta de conjuntivas. No refiere conductas sexuales de riesgo ni antecedente epidemiol\u00f3gicos de riesgo de hepatitis v\u00edricas. No consume alcohol ni f\u00e1rmacos hepatot\u00f3xicos. Relata historia de un a\u00f1o de evoluci\u00f3n de prurito generalizado, astenia, sequedad de boca y ausencia de lagrimeo de causa no filiada. Resto de la anamnesis sin datos patol\u00f3gicos. En la exploraci\u00f3n f\u00edsica se observan lesiones de rascado, ictericia conjuntival y hepatomegalia no dolorosa. Aporta anal\u00edtica realizada en su empresa con los siguientes resultados patol\u00f3gicos: Bilirrubina 3 mg/dl, FA 400 UI/ VSG 40mm 1 hora. Indique cual ser\u00eda la recomendaci\u00f3n m\u00e1s acertada para establecer el diagn\u00f3stico etiol\u00f3gico del cuadro que presenta la paciente:", "full_answer": "Cuadro de libro de Cirrosis Biliar Primaria, de esos que no se presentan en la vida real. El diagn\u00f3stico estar\u00eda pr\u00e1cticamente hecho con los AMA (1). Con la 2 descartar\u00edamos hemocromatosis, Con la 3 la enfermedad de Wilson. Con la 4 podr\u00edamos descartar enfermedades raras tipo malformaciones de v\u00eda biliar o enf. De Caroli y con la 5 las hepatitis v\u00edricas.", "type": "DIGESTIVO", "options": {"1": "Anticuerpos antimitocondriales.", "2": "Estudio del metabolismo del Fe.", "3": "Estudio del metabolismo del cobre.", "4": "Resonancia magn\u00e9tica hep\u00e1tica.", "5": "Serolog\u00eda de virus B y C."}, "correct_option": 1, "explanations": {"1": {"exist": true, "char_ranges": [[90, 149]], "word_ranges": [[17, 26]], "text": "El diagn\u00f3stico estar\u00eda pr\u00e1cticamente hecho con los AMA (1)."}, "2": {"exist": true, "char_ranges": [[150, 188]], "word_ranges": [[26, 31]], "text": "Con la 2 descartar\u00edamos hemocromatosis,"}, "3": {"exist": true, "char_ranges": [[190, 223]], "word_ranges": [[31, 38]], "text": "Con la 3 la enfermedad de Wilson."}, "4": {"exist": true, "char_ranges": [[224, 322]], "word_ranges": [[38, 54]], "text": "Con la 4 podr\u00edamos descartar enfermedades raras tipo malformaciones de v\u00eda biliar o enf. De Caroli"}, "5": {"exist": true, "char_ranges": [[325, 356]], "word_ranges": [[55, 61]], "text": "con la 5 las hepatitis v\u00edricas."}}} +{"id": 621, "year": 2022, "question_id_specific": 60, "full_question": "Paciente de 20 a\u00f1os que acude a urgencias tras sufrir un accidente de bicicleta con traumatismo facial. Se realiza TC craneal que muestra una fractura del tercio medio facial que afecta a la regi\u00f3n \u00f3rbito-malar. Una de las complicaciones m\u00e1s frecuentes de este tipo de fracturas es:", "full_answer": "Las fracturas del tercio medio facial en la regi\u00f3n orbito-malar pueden afectar al suelo y, en menor medida, a la pared lateral de la \u00f3rbita. Asumiendo por tanto que nos preguntan cu\u00e1l es una de las complicaciones m\u00e1s frecuentes del suelo de la \u00f3rbita, deben destacarse siempre dos complicaciones que pueden indicar tratamiento quir\u00fargico, incluso urgente: la diplop\u00eda, por luxaci\u00f3n del m\u00fasculo recto inferior al seno maxilar infrayacente (e incluso atrapamiento del mismo); y el enoftalmos, que puede condicionar a medio y largo plazo otras complicaciones asociadas, como la pseudo-ptosis palpebral superior por p\u00e9rdida de volumen orbitario (opci\u00f3n 4 correcta). La anquilosis temporo-mandibular no se contempla porque, aunque relativamente cercana a la cavidad orbitaria, no forma parte del complejo \u00f3rbito-malar (opci\u00f3n 1 descartada). La afectaci\u00f3n del maxilar puede dar lugar a maloclusi\u00f3n dental, pero normalmente ocurre en fracturas de localizaci\u00f3n m\u00e1s inferior a la cavidad orbitaria (opci\u00f3n 2 descartada). Las fracturas naso-etmoidales se contemplan dentro de las fracturas del tercio medio facial, pero los huesos propios de la nariz se localizan m\u00e1s anteriormente al reborde orbitario medial, y por consiguiente fuera de la \u00f3rbita (opci\u00f3n 3 descartada).", "type": "OFTALMOLOG\u00cdA (ECT\u00d3PICO)", "options": {"1": "Anquilosis t\u00e9mporo-mandibular.", "2": "Maloclusi\u00f3n dental.", "3": "Pseudoartrosis naso-etmoidal.", "4": "Enoftalmos.", "5": null}, "correct_option": 4, "explanations": {"1": {"exist": true, "char_ranges": [[662, 835]], "word_ranges": [[100, 123]], "text": "La anquilosis temporo-mandibular no se contempla porque, aunque relativamente cercana a la cavidad orbitaria, no forma parte del complejo \u00f3rbito-malar (opci\u00f3n 1 descartada)."}, "2": {"exist": true, "char_ranges": [[836, 1011]], "word_ranges": [[123, 149]], "text": "La afectaci\u00f3n del maxilar puede dar lugar a maloclusi\u00f3n dental, pero normalmente ocurre en fracturas de localizaci\u00f3n m\u00e1s inferior a la cavidad orbitaria (opci\u00f3n 2 descartada)."}, "3": {"exist": true, "char_ranges": [[1012, 1261]], "word_ranges": [[149, 187]], "text": "Las fracturas naso-etmoidales se contemplan dentro de las fracturas del tercio medio facial, pero los huesos propios de la nariz se localizan m\u00e1s anteriormente al reborde orbitario medial, y por consiguiente fuera de la \u00f3rbita (opci\u00f3n 3 descartada)."}, "4": {"exist": true, "char_ranges": [[141, 661]], "word_ranges": [[25, 100]], "text": "Asumiendo por tanto que nos preguntan cu\u00e1l es una de las complicaciones m\u00e1s frecuentes del suelo de la \u00f3rbita, deben destacarse siempre dos complicaciones que pueden indicar tratamiento quir\u00fargico, incluso urgente: la diplop\u00eda, por luxaci\u00f3n del m\u00fasculo recto inferior al seno maxilar infrayacente (e incluso atrapamiento del mismo); y el enoftalmos, que puede condicionar a medio y largo plazo otras complicaciones asociadas, como la pseudo-ptosis palpebral superior por p\u00e9rdida de volumen orbitario (opci\u00f3n 4 correcta)."}, "5": {"exist": false, "char_ranges": [], "word_ranges": [], "text": ""}}} +{"id": 470, "year": 2020, "question_id_specific": 138, "full_question": "Mujer de 73 a\u00f1os con antecedentes de obesidad, diabetes mellitus tipo 2, hipertensi\u00f3n arterial y dislipemia. Consulta por dolor insoportable en la rodilla derecha de 5 d\u00edas de evoluci\u00f3n, sin trauma previo. Exploraci\u00f3n: rodilla globulosa, varo moderado, extensi\u00f3n y flexi\u00f3n limitadas por dolor, dolor difuso medial. En la radiograf\u00eda se aprecian osteofitos y pinzamiento leve de la interl\u00ednea medial. \u00bfCu\u00e1l ser\u00eda su manejo inicial?:", "full_answer": "Nos est\u00e1n describiendo una crisis aguda de dolor en un paciente con gonartrosis. En esta situaci\u00f3n lo primero es resolver la crisis de dolor y plantear un tratamiento adecuado conservador para dicha artrosis (1 correcta). No se plantea de entrada una artroplastia de rodilla por lo que 2 es falsa. No nos est\u00e1n contando una cl\u00ednica infecciosa para sospechar de una artritis que justificara un desbridamiento y lavado por lo que 3 es falsa. En un paciente con gonartrosis va a existir siempre meniscopat\u00eda, forma parte de los cambios degenerativos. El quiste de Baker solo nos interesar\u00eda en una crisis de dolor severa en el diagn\u00f3stico diferencial con una trombosis profunda y se valora con una ecodoppler, en el cuadro de gonartrosis no tiene ning\u00fan valor detectar un quiste de Baker. Las tendinitis se diagnostican mediante la exploraci\u00f3n no con RMN. Por todo ello la 4 es falsa.", "type": "CIRUG\u00cdA ORTOP\u00c9DICA Y TRAUMATOLOG\u00cdA", "options": {"1": "Explicaci\u00f3n del diagn\u00f3stico, reposo relativo, paracetamol 1g/8h m\u00e1s metamizol 500 mg/ 8 h naproxeno de rescate.", "2": "Derivaci\u00f3n preferente a consultas externas de Traumatolog\u00eda para valoraci\u00f3n de pr\u00f3tesis total cementada.", "3": "Derivaci\u00f3n preferente a consultas externas de Traumatolog\u00eda para valorar desbridamiento artrosc\u00f3pico.", "4": "Solicitud de resonancia magn\u00e9tica preferente para evaluaci\u00f3n de meniscopat\u00eda,, quiste de Baker y/o tendinitis.", "5": null}, "correct_option": 1, "explanations": {"1": {"exist": true, "char_ranges": [[0, 221]], "word_ranges": [[0, 35]], "text": "Nos est\u00e1n describiendo una crisis aguda de dolor en un paciente con gonartrosis. En esta situaci\u00f3n lo primero es resolver la crisis de dolor y plantear un tratamiento adecuado conservador para dicha artrosis (1 correcta)."}, "2": {"exist": true, "char_ranges": [[222, 297]], "word_ranges": [[35, 50]], "text": "No se plantea de entrada una artroplastia de rodilla por lo que 2 es falsa."}, "3": {"exist": true, "char_ranges": [[298, 439]], "word_ranges": [[50, 74]], "text": "No nos est\u00e1n contando una cl\u00ednica infecciosa para sospechar de una artritis que justificara un desbridamiento y lavado por lo que 3 es falsa."}, "4": {"exist": true, "char_ranges": [[548, 881]], "word_ranges": [[90, 148]], "text": "El quiste de Baker solo nos interesar\u00eda en una crisis de dolor severa en el diagn\u00f3stico diferencial con una trombosis profunda y se valora con una ecodoppler, en el cuadro de gonartrosis no tiene ning\u00fan valor detectar un quiste de Baker. Las tendinitis se diagnostican mediante la exploraci\u00f3n no con RMN. Por todo ello la 4 es falsa."}, "5": {"exist": false, "char_ranges": [], "word_ranges": [], "text": ""}}} +{"id": 298, "year": 2016, "question_id_specific": 103, "full_question": "Hombre de 25 a\u00f1os sin antecedentes de inter\u00e9s que acude a urgencias con fiebre, cefalea, mialgias, n\u00e1useas, v\u00f3mitos, dolor abdominal, ictericia e inyecci\u00f3n conjuntival, 2 semanas despu\u00e9s de haber viajado a Thailandia para participar en una regata en agua dulce. \u00bfCu\u00e1l es el diagn\u00f3stico m\u00e1s probable?:", "full_answer": "El antecedente de contacto con agua dulce en un \u00e1rea end\u00e9mica orienta a leptospirosis, y en la cl\u00ednica encontramos tambi\u00e9n ictericia e inyecci\u00f3n conjuntival que nos orienta tambi\u00e9n a leptospirosis y no se relaciona tan caracter\u00edsticamente con los otros tres pat\u00f3genos.", "type": "ENFERMEDADES INFECCIOSAS", "options": {"1": "Malaria.", "2": "Esquistosomiasis.", "3": "Leptospirosis.", "4": "Rabia.", "5": null}, "correct_option": 3, "explanations": {"1": {"exist": true, "char_ranges": [[0, 268]], "word_ranges": [[0, 41]], "text": "El antecedente de contacto con agua dulce en un \u00e1rea end\u00e9mica orienta a leptospirosis, y en la cl\u00ednica encontramos tambi\u00e9n ictericia e inyecci\u00f3n conjuntival que nos orienta tambi\u00e9n a leptospirosis y no se relaciona tan caracter\u00edsticamente con los otros tres pat\u00f3genos."}, "2": {"exist": true, "char_ranges": [[0, 268]], "word_ranges": [[0, 41]], "text": "El antecedente de contacto con agua dulce en un \u00e1rea end\u00e9mica orienta a leptospirosis, y en la cl\u00ednica encontramos tambi\u00e9n ictericia e inyecci\u00f3n conjuntival que nos orienta tambi\u00e9n a leptospirosis y no se relaciona tan caracter\u00edsticamente con los otros tres pat\u00f3genos."}, "3": {"exist": true, "char_ranges": [[0, 268]], "word_ranges": [[0, 41]], "text": "El antecedente de contacto con agua dulce en un \u00e1rea end\u00e9mica orienta a leptospirosis, y en la cl\u00ednica encontramos tambi\u00e9n ictericia e inyecci\u00f3n conjuntival que nos orienta tambi\u00e9n a leptospirosis y no se relaciona tan caracter\u00edsticamente con los otros tres pat\u00f3genos."}, "4": {"exist": true, "char_ranges": [[0, 268]], "word_ranges": [[0, 41]], "text": "El antecedente de contacto con agua dulce en un \u00e1rea end\u00e9mica orienta a leptospirosis, y en la cl\u00ednica encontramos tambi\u00e9n ictericia e inyecci\u00f3n conjuntival que nos orienta tambi\u00e9n a leptospirosis y no se relaciona tan caracter\u00edsticamente con los otros tres pat\u00f3genos."}, "5": {"exist": false, "char_ranges": [], "word_ranges": [], "text": ""}}} +{"id": 365, "year": 2016, "question_id_specific": 100, "full_question": "Un paciente de 64 a\u00f1os de edad, agricultor, ex\u00ad fumador (5 a\u00f1os), EPOC y afecto de artritis reumatoide en tratamiento corticoideo. Consulta en urgencias por presentar cefalea intensa de 2 d\u00edas de evoluci\u00f3n con desviaci\u00f3n de comisura bucal. Como antecedente, relata que tras cuadro gripal hace un mes, persiste tos, expectoraci\u00f3n purulenta y ocasionalmente hemoptoica, febr\u00edcula, anorexia, astenia y p\u00e9rdida de peso. A su llegada se aprecia la existencia de fiebre de 38.2\u00baC, abscesos cut\u00e1neos m\u00faltiples en manos, espalda y nalgas (algunos con trayectos fistulosos) y par\u00e1lisis facial central derecha, infiltrados apicales con peque\u00f1o derrame pleural asociado en la radiograf\u00eda de t\u00f3rax y leucocitosis con neutrofilia. Entre los siguientes diagn\u00f3sticos de sospecha considerar\u00eda M\u00c1S probable:", "full_answer": "La nocardia tipicamente puede afectar a inmunosuprimidos, en especial con alteraci\u00f3n de la inmunidad celular como es la producida por los esteroides, y puede cursar con afectaci\u00f3n pulmonar, abscesos cerebrales y cut\u00e1neos. La Tuberculosis puede afectar a pulm\u00f3n y cerebro pero no abscesos cut\u00e1neos. Aspergillus tampoco afecta a piel.", "type": "NEUMOLOG\u00cdA Y CIRUG\u00cdA TOR\u00c1CICA", "options": {"1": "Neoplasia pulmonar con met\u00e1stasis cerebrales.", "2": "Tuberculosis diseminada.", "3": "Nocardiosis.", "4": "Aspergilosis.", "5": null}, "correct_option": 4, "explanations": {"1": {"exist": false, "char_ranges": [], "word_ranges": [], "text": ""}, "2": {"exist": true, "char_ranges": [[222, 296]], "word_ranges": [[32, 44]], "text": "La Tuberculosis puede afectar a pulm\u00f3n y cerebro pero no abscesos cut\u00e1neos."}, "3": {"exist": true, "char_ranges": [[0, 221]], "word_ranges": [[0, 32]], "text": "La nocardia tipicamente puede afectar a inmunosuprimidos, en especial con alteraci\u00f3n de la inmunidad celular como es la producida por los esteroides, y puede cursar con afectaci\u00f3n pulmonar, abscesos cerebrales y cut\u00e1neos."}, "4": {"exist": true, "char_ranges": [[298, 332]], "word_ranges": [[44, 49]], "text": "Aspergillus tampoco afecta a piel."}, "5": {"exist": false, "char_ranges": [], "word_ranges": [], "text": ""}}} +{"id": 392, "year": 2016, "question_id_specific": 223, "full_question": "Mujer de 68 a\u00f1os, con antecedentes de 2 episodios depresivos mayores a lo largo de su vida, que consulta por s\u00edntomas de tristeza, decaimiento, anhedonia, astenia y anorexia compatibles con un nuevo episodio depresivo. Se pautan 10 mg de escitalopram y se eval\u00faa 2 semanas despu\u00e9s. En esta revisi\u00f3n la paciente refiere estar muy bien, se despierta temprano muy hiperactiva y con \u2018muchas ganas de hacer cosas\u2019, dice tener mucha energ\u00eda y est\u00e1 m\u00e1s habladora de lo que en ella es habitual. No refiere estar irritable y es capaz de dormir 6 horas continuadamente. Ante esta situaci\u00f3n, \u00bfqu\u00e9 pensar\u00eda que tiene la paciente?", "full_answer": "En esta pregunta cabr\u00eda sospechar la posibilidad de un viraje afectivo ante la introducci\u00f3n del f\u00e1rmaco antidepresivo, por lo que refiere la paciente, pero si nos fijamos hay un dato claro que \u201cdescarta\u201d el viraje hipo/man\u00edaco y es la capacidad de mantener el sue\u00f1o. Dos semanas es poco tiempo para que el f\u00e1rmaco haya hecho efecto pleno sobre el estado del \u00e1nimo, pero s\u00ed puede aparecer una activaci\u00f3n inicial que los pacientes a veces no llevan muy bien (algo que no parece ser el caso). En esta paciente est\u00e1 indicado mantener el tratamiento y reevaluar en no m\u00e1s de un mes.", "type": "PSIQUIATR\u00cdA", "options": {"1": "Trastorno bipolar tipo I.", "2": "Hipoman\u00eda inducida por f\u00e1rmacos.", "3": "Respuesta normal al escitalopram.", "4": "Demencia frontal.", "5": null}, "correct_option": 3, "explanations": {"1": {"exist": false, "char_ranges": [], "word_ranges": [], "text": ""}, "2": {"exist": true, "char_ranges": [[171, 266]], "word_ranges": [[27, 44]], "text": "hay un dato claro que \u201cdescarta\u201d el viraje hipo/man\u00edaco y es la capacidad de mantener el sue\u00f1o."}, "3": {"exist": false, "char_ranges": [], "word_ranges": [], "text": ""}, "4": {"exist": false, "char_ranges": [], "word_ranges": [], "text": ""}, "5": {"exist": false, "char_ranges": [], "word_ranges": [], "text": ""}}} +{"id": 285, "year": 2016, "question_id_specific": 59, "full_question": "Mujer de 73 a\u00f1os de edad que ingresa por cuadro de disnea progresiva hasta hacerse de reposo, ortopnea y aumento de 4 kg de peso. En la exploraci\u00f3n fisica se aprecia presi\u00f3n arterial de 150/84 mm Hg, frecuencia cardiaca 100 latidos/minuto, aumento de la presi\u00f3n venosa yugular, crepitantes en ambas bases y edemas maleolares. Tratamiento habitual: enalapril 5 mg cada 12 horas, furosemida 80 mg al d\u00eda. \u00bfCu\u00e1l es el tratamiento m\u00e1s adecuado en este momento?", "full_answer": "Aumentar dosis de enalapril seg\u00fan tolerancia y administrar turosemida por v\u00eda intravenosa.", "type": "CARDIOLOG\u00cdA Y CIRUG\u00cdA VASCULAR", "options": {"1": "Administrar fiirosemida por v\u00eda intravenosa.", "2": "Aumentar dosis de enalapril seg\u00fan tolerancia y administrar furosemida por v\u00eda intravenosa.", "3": "Iniciar un beta-bloqueante.", "4": "A\u00f1adir tratamiento con amlodipino.", "5": null}, "correct_option": 2, "explanations": {"1": {"exist": false, "char_ranges": [], "word_ranges": [], "text": ""}, "2": {"exist": true, "char_ranges": [[0, 90]], "word_ranges": [[0, 12]], "text": "Aumentar dosis de enalapril seg\u00fan tolerancia y administrar turosemida por v\u00eda intravenosa."}, "3": {"exist": false, "char_ranges": [], "word_ranges": [], "text": ""}, "4": {"exist": false, "char_ranges": [], "word_ranges": [], "text": ""}, "5": {"exist": false, "char_ranges": [], "word_ranges": [], "text": ""}}} +{"id": 301, "year": 2016, "question_id_specific": 232, "full_question": "Una mujer de 24 a\u00f1os de edad consulta al haber apreciado adenopat\u00edas inguinales. En el interrogatorio no se recoge la presencia de ninguna molestia local ni datos sugerentes de infecci\u00f3n de transmisi\u00f3n sexual. En la exploraci\u00f3n se aprecian dos adenopat\u00edas, una en cada ingle, de 1 cm de di\u00e1metro mayor, blandas, m\u00f3viles, no dolorosas. No se aprecia ninguna lesi\u00f3n cut\u00e1nea en miembros inferiores, ano o perin\u00e9. \u00bfQu\u00e9 prueba considera imprescindible?", "full_answer": "En el adulto sano pueden presentarse ganglios inguinales palpables de hasta 2 cent\u00edmetros que pueden considerarse dentro de la normalidad. No est\u00e1 justificado el estudio complementario de estos ganglios linf\u00e1ticos normales.", "type": "ENFERMEDADES INFECCIOSAS", "options": {"1": "Una serolog\u00eda de l\u00faes puesto que lo m\u00e1s probable es que se trate de una infecci\u00f3n por Treponema pallidum.", "2": "Una exploraci\u00f3n ginecol\u00f3gica a fin de descartar un c\u00e1ncer de ovario.", "3": "Por las caracter\u00edsticas cl\u00ednicas parece tratarse de unos ganglios normales y no deben hacerse exploraciones complementarias.", "4": "Debe realizarse una prueba de Paul-Bunnell a \ufb01n de descartar una mononucleosis infecciosa.", "5": null}, "correct_option": 3, "explanations": {"1": {"exist": false, "char_ranges": [], "word_ranges": [], "text": ""}, "2": {"exist": false, "char_ranges": [], "word_ranges": [], "text": ""}, "3": {"exist": true, "char_ranges": [[0, 223]], "word_ranges": [[0, 31]], "text": "En el adulto sano pueden presentarse ganglios inguinales palpables de hasta 2 cent\u00edmetros que pueden considerarse dentro de la normalidad. No est\u00e1 justificado el estudio complementario de estos ganglios linf\u00e1ticos normales."}, "4": {"exist": false, "char_ranges": [], "word_ranges": [], "text": ""}, "5": {"exist": false, "char_ranges": [], "word_ranges": [], "text": ""}}} +{"id": 23, "year": 2011, "question_id_specific": 113, "full_question": "Mujer de 71 a\u00f1os, con antecedentes de artritis reumatoide en tto con sulfasalazina, prednisona y etanercept. Acude a URG por cl\u00ednica de 72 horas compatible con Herpes Z\u00f3ster facial afectando a a hemicara derecha, pabell\u00f3n auricular, respetando frente y quemosis conjuntival. \u00bfCu\u00e1l ser\u00eda el tratamiento adecuado?", "full_answer": "Creo que esta pregunta no est\u00e1 claramente en el temario de Infecciosas, y puede que se solape con OFT y DERMA, pero seg\u00fan tengo entendido, en una paciente inmunodeprimida y adem\u00e1s con datos de afectaci\u00f3n ocular, estar\u00eda indicado el ingreso para tratamiento intravenoso por el alto riesgo de complicaciones posibles.", "type": "INFECCIOSAS", "options": {"1": "Tto sintom\u00e1tico del dolor \u00fanicamente.", "2": "Tto t\u00f3pico con aciclovir.", "3": "Tto ambulatorio con aciclovir, valaciclovir o famciclovir oral.", "4": "Ingreso hospitalario y tto con aciclovir o famciclovir iv.", "5": "Ig parenteral y vacunaci\u00f3n."}, "correct_option": 4, "explanations": {"1": {"exist": false, "char_ranges": [], "word_ranges": [], "text": ""}, "2": {"exist": false, "char_ranges": [], "word_ranges": [], "text": ""}, "3": {"exist": false, "char_ranges": [], "word_ranges": [], "text": ""}, "4": {"exist": true, "char_ranges": [[139, 315]], "word_ranges": [[25, 50]], "text": "en una paciente inmunodeprimida y adem\u00e1s con datos de afectaci\u00f3n ocular, estar\u00eda indicado el ingreso para tratamiento intravenoso por el alto riesgo de complicaciones posibles."}, "5": {"exist": false, "char_ranges": [], "word_ranges": [], "text": ""}}} +{"id": 564, "year": 2022, "question_id_specific": 126, "full_question": "Var\u00f3n de 58 a\u00f1os con antecedentes de hipertensi\u00f3n arterial de 6 a\u00f1os de evoluci\u00f3n, que consulta por mal control de las cifras de presi\u00f3n arterial a pesar de recibir tratamiento con un inhibidor de la enzima convertidora en angiotensina, un diur\u00e9tico y un calcioantagonista. En consulta presenta cifras de 149/100 mmHg. Anal\u00edtica: creatinina 1,2 mg/dl, potasio 2,2 mEq/l y alcalosis metab\u00f3lica compensada; resto del estudio bioqu\u00edmico, hemograma, coagulaci\u00f3n y sedimento urinario normal. Se\u00f1ale la afirmaci\u00f3n correcta:", "full_answer": "Nos presentan un paciente con hipertensi\u00f3n arterial resistente, dando por hecho que tiene un hiperaldosteronismo primario por la alcalosis metab\u00f3lica hipopotas\u00e9mica. Cuando est\u00e1 bioqu\u00edmicamente confirmado el diagn\u00f3stico, la siguiente prueba a realizar es una TC para determinar el subtipo y descartar la presencia de un carcinoma suprarrenal (opci\u00f3n 3 correcta). Por tanto, hay que esperar a la prueba de imagen para confirmar la etiolog\u00eda (opci\u00f3n 1 incorrecta). La causa m\u00e1s frecuente es el adenoma productor de aldosterona (opci\u00f3n 2 incorrecta). La espironolactona es el tratamiento m\u00e9dico de elecci\u00f3n (opci\u00f3n 4 incorrecta).", "type": "NEFROLOG\u00cdA", "options": {"1": "El origen de la hipertensi\u00f3n en este caso es la secreci\u00f3n excesiva de aldosterona causada por una hiperfunci\u00f3n aut\u00f3noma de la m\u00e9dula suprarrenal.", "2": "En la mayor\u00eda de los casos el sustrato anat\u00f3mico es una hiperplasia bilateral de la corteza suprarrenal.", "3": "La TC forma parte del estudio diagn\u00f3stico en caso de haber una confirmaci\u00f3n bioqu\u00edmica.", "4": "La espironolactona est\u00e1 contraindicada en el manejo de esta patolog\u00eda.", "5": null}, "correct_option": 3, "explanations": {"1": {"exist": true, "char_ranges": [[363, 462]], "word_ranges": [[50, 67]], "text": "Por tanto, hay que esperar a la prueba de imagen para confirmar la etiolog\u00eda (opci\u00f3n 1 incorrecta)."}, "2": {"exist": true, "char_ranges": [[463, 547]], "word_ranges": [[67, 80]], "text": "La causa m\u00e1s frecuente es el adenoma productor de aldosterona (opci\u00f3n 2 incorrecta)."}, "3": {"exist": true, "char_ranges": [[0, 362]], "word_ranges": [[0, 50]], "text": "Nos presentan un paciente con hipertensi\u00f3n arterial resistente, dando por hecho que tiene un hiperaldosteronismo primario por la alcalosis metab\u00f3lica hipopotas\u00e9mica. Cuando est\u00e1 bioqu\u00edmicamente confirmado el diagn\u00f3stico, la siguiente prueba a realizar es una TC para determinar el subtipo y descartar la presencia de un carcinoma suprarrenal (opci\u00f3n 3 correcta)."}, "4": {"exist": true, "char_ranges": [[548, 626]], "word_ranges": [[80, 91]], "text": "La espironolactona es el tratamiento m\u00e9dico de elecci\u00f3n (opci\u00f3n 4 incorrecta)."}, "5": {"exist": false, "char_ranges": [], "word_ranges": [], "text": ""}}} +{"id": 292, "year": 2016, "question_id_specific": 213, "full_question": "Gustavo acude a Urgencias con lesiones cut\u00e1neas y malestar general de varios d\u00edas de evoluci\u00f3n. Tiene lesiones psoriasiformes en tronco con afectaci\u00f3n de palmas y plantas. Tambi\u00e9n presenta una inflamaci\u00f3n articular asim\u00e9trica no supurativa y enrojecimiento ocular bilateral as\u00ed como erosiones en glande. En la anamnesis posterior Gustavo reconoce un contacto sexual de riesgo 20 d\u00edas antes. \u00bfCu\u00e1l es su diagn\u00f3stico?", "full_answer": "Est\u00e1 describiendo un s\u00edndrome de Reiter de libro: queratodermia palmo-plantar, artritis y manifestaciones oculares, junto a seguramente una uretritis por clamidia, quiz\u00e1 asintom\u00e1tica (no explican exudado uretral). Naturalmente, adem\u00e1s el paciente puede tener una infecci\u00f3n por VIH o incluso una s\u00edfilis secundaria (adem\u00e1s del Reiter).", "type": "DERMATOLOG\u00cdA, VENEREOLOG\u00cdA Y CIRUG\u00cdA PL\u00c1STICA", "options": {"1": "Infecci\u00f3n por VIH.", "2": "S\u00edfilis secundaria.", "3": "S\u00edndrome de Reiter.", "4": "Eritema multiforme.", "5": null}, "correct_option": 3, "explanations": {"1": {"exist": false, "char_ranges": [], "word_ranges": [], "text": ""}, "2": {"exist": false, "char_ranges": [], "word_ranges": [], "text": ""}, "3": {"exist": true, "char_ranges": [[0, 213]], "word_ranges": [[0, 27]], "text": "Est\u00e1 describiendo un s\u00edndrome de Reiter de libro: queratodermia palmo-plantar, artritis y manifestaciones oculares, junto a seguramente una uretritis por clamidia, quiz\u00e1 asintom\u00e1tica (no explican exudado uretral)."}, "4": {"exist": false, "char_ranges": [], "word_ranges": [], "text": ""}, "5": {"exist": false, "char_ranges": [], "word_ranges": [], "text": ""}}} +{"id": 477, "year": 2020, "question_id_specific": 128, "full_question": "Hombre de 35 a\u00f1os, ingresado por traumatismo tor\u00e1cico grave con m\u00faltiples fracturas costales. Tras responder favorablemente al tratamiento con analg\u00e9sicos y ox\u00edgeno, comienza a presentar hipoxemia grave. Se\u00f1ale cu\u00e1l es la causa m\u00e1s probable de este deterioro:", "full_answer": "La contusi\u00f3n pulmonar es la lesi\u00f3n m\u00e1s grave y que condiciona peor pron\u00f3stico en un traumatismo tor\u00e1cico. Tambi\u00e9n es la lesi\u00f3n que ocasiona m\u00e1s precozmente hipoxemia. En contra de lo que podemos pensar, el t\u00f3rax inestable por fracturas costales m\u00faltiples (volet costal) ocasionar\u00eda una hipoventilaci\u00f3n progresiva con atelectasias del par\u00e9nquima pulmonar, que ir\u00eda progresando por una parte en hipercapnia y acidosis respiratoria, y por otra, en hipoxemia m\u00e1s tard\u00eda por infecci\u00f3n asociada a las atelectasias. La infecci\u00f3n respiratoria por aspiraci\u00f3n tambi\u00e9n se presentar\u00eda de forma m\u00e1s tard\u00eda, y en no aparece en todos los casos de traumatismo tor\u00e1cico grave, s\u00f3lo si ha habido disminuci\u00f3n de nivel de consciencia (TCE asociado, intubaci\u00f3n no controlada con broncoaspiraci\u00f3n\u2026). La hipovolemia postraum\u00e1tica (en este caso, al haber fracturas costales m\u00faltiples podr\u00eda deberse a un hemot\u00f3rax) se producir\u00eda de forma m\u00e1s precoz y asociar\u00eda en primer lugar inestabilidad hemodin\u00e1mica.", "type": "CUIDADOS CR\u00cdTICOS", "options": {"1": "La inestabilidad de la pared tor\u00e1cica por las fracturas m\u00faltiples.", "2": "La infecci\u00f3n respiratoria por aspiraci\u00f3n.", "3": "La alteraci\u00f3n del intercambio gaseoso por la contusi\u00f3n pulmonar.", "4": "La hipovolemia postraum\u00e1tica.", "5": null}, "correct_option": 3, "explanations": {"1": {"exist": true, "char_ranges": [[203, 508]], "word_ranges": [[33, 75]], "text": "el t\u00f3rax inestable por fracturas costales m\u00faltiples (volet costal) ocasionar\u00eda una hipoventilaci\u00f3n progresiva con atelectasias del par\u00e9nquima pulmonar, que ir\u00eda progresando por una parte en hipercapnia y acidosis respiratoria, y por otra, en hipoxemia m\u00e1s tard\u00eda por infecci\u00f3n asociada a las atelectasias."}, "2": {"exist": true, "char_ranges": [[509, 777]], "word_ranges": [[75, 115]], "text": "La infecci\u00f3n respiratoria por aspiraci\u00f3n tambi\u00e9n se presentar\u00eda de forma m\u00e1s tard\u00eda, y en no aparece en todos los casos de traumatismo tor\u00e1cico grave, s\u00f3lo si ha habido disminuci\u00f3n de nivel de consciencia (TCE asociado, intubaci\u00f3n no controlada con broncoaspiraci\u00f3n\u2026)."}, "3": {"exist": true, "char_ranges": [[0, 166]], "word_ranges": [[0, 26]], "text": "La contusi\u00f3n pulmonar es la lesi\u00f3n m\u00e1s grave y que condiciona peor pron\u00f3stico en un traumatismo tor\u00e1cico. Tambi\u00e9n es la lesi\u00f3n que ocasiona m\u00e1s precozmente hipoxemia."}, "4": {"exist": true, "char_ranges": [[778, 980]], "word_ranges": [[115, 144]], "text": "La hipovolemia postraum\u00e1tica (en este caso, al haber fracturas costales m\u00faltiples podr\u00eda deberse a un hemot\u00f3rax) se producir\u00eda de forma m\u00e1s precoz y asociar\u00eda en primer lugar inestabilidad hemodin\u00e1mica."}, "5": {"exist": false, "char_ranges": [], "word_ranges": [], "text": ""}}} +{"id": 572, "year": 2022, "question_id_specific": 195, "full_question": "Mujer de 66 a\u00f1os con diabetes mellitus tipo 2. Valorando su funci\u00f3n renal presenta un estadio G3a/A1. \u00bfA qu\u00e9 valores corresponde este estadio, el m\u00e1s frecuente en pacientes con nefropat\u00eda diab\u00e9tica?:", "full_answer": "Pregunta f\u00e1cil. El estadio G3a corresponde a un filtrado entre 45-59 ml/min. El estadio A1 corresponde a una albuminuria inferior a 30 mg/ml. Por tanto la opci\u00f3n correcta es la 1.", "type": "NEFROLOG\u00cdA", "options": {"1": "Filtrado glomerular 45-59 ml/min/1,73 m\u00b2 y albuminuria <30 mg/ml.", "2": "Filtrado glomerular 30-44 ml/min/1,73 m\u00b2 y albuminuria <30 mg/ml.", "3": "Filtrado glomerular 45-59 ml/min/1,73 m\u00b2 y albuminuria 30-300 mg/ml.", "4": "Filtrado glomerular 15-29 ml/min/1,73 m\u00b2 y albuminuria <30 mg/ml.", "5": null}, "correct_option": 1, "explanations": {"1": {"exist": true, "char_ranges": [[16, 179]], "word_ranges": [[2, 31]], "text": "El estadio G3a corresponde a un filtrado entre 45-59 ml/min. El estadio A1 corresponde a una albuminuria inferior a 30 mg/ml. Por tanto la opci\u00f3n correcta es la 1."}, "2": {"exist": false, "char_ranges": [], "word_ranges": [], "text": ""}, "3": {"exist": false, "char_ranges": [], "word_ranges": [], "text": ""}, "4": {"exist": false, "char_ranges": [], "word_ranges": [], "text": ""}, "5": {"exist": false, "char_ranges": [], "word_ranges": [], "text": ""}}} +{"id": 180, "year": 2013, "question_id_specific": 232, "full_question": "Un paciente de 29 a\u00f1os acude a su consulta con diagn\u00f3stico de aplasia medular severa. \u00bfCu\u00e1l es el tratamiento de elecci\u00f3n?", "full_answer": "Transfundir a este paciente y hartarlo de antibi\u00f3ticos es darle pan para hoy y hambre para ma\u00f1ana. Queremos algo m\u00e1s duradero. Los andr\u00f3genos y las transfusiones de plaquetas tampoco nos arreglan el problema. Nos quedan las otras tres opciones. Un transplante aut\u00f3logo no es razonable, ya que la m\u00e9dula \u00f3sea de un paciente con aplasia medular es menor del 25%, de ah\u00ed poco se puede sacar. Por tanto, nos inclinamos por el transplante alog\u00e9nico de m\u00e9dula \u00f3sea si tiene un hermano HLA id\u00e9ntico, ya que \u00e9se es el tratamiento de elecci\u00f3n seg\u00fan marca el protocolo de la Sociedad Espa\u00f1ola de Hematolog\u00eda y Hemoterapia para pacientes con menos de 40 a\u00f1os y aplasia medular severa. Respuesta correcta, la 3.", "type": "HEMATOLOG\u00cdA", "options": {"1": "Transfusiones peri\u00f3dicas y antibi\u00f3ticos.", "2": "Andr\u00f3genos y transfusiones de plaquetas.", "3": "Transplante alog\u00e9nico de m\u00e9dula \u00f3sea si hermano HLA id\u00e9ntico.", "4": "Transplante aut\u00f3logo de m\u00e9dula \u00f3sea para evitar rechazo.", "5": "Ciclosporina A y globulina antitimoc\u00edtica."}, "correct_option": 3, "explanations": {"1": {"exist": true, "char_ranges": [[0, 98]], "word_ranges": [[0, 17]], "text": "Transfundir a este paciente y hartarlo de antibi\u00f3ticos es darle pan para hoy y hambre para ma\u00f1ana."}, "2": {"exist": true, "char_ranges": [[127, 208]], "word_ranges": [[21, 33]], "text": "Los andr\u00f3genos y las transfusiones de plaquetas tampoco nos arreglan el problema."}, "3": {"exist": true, "char_ranges": [[400, 673]], "word_ranges": [[68, 114]], "text": "nos inclinamos por el transplante alog\u00e9nico de m\u00e9dula \u00f3sea si tiene un hermano HLA id\u00e9ntico, ya que \u00e9se es el tratamiento de elecci\u00f3n seg\u00fan marca el protocolo de la Sociedad Espa\u00f1ola de Hematolog\u00eda y Hemoterapia para pacientes con menos de 40 a\u00f1os y aplasia medular severa."}, "4": {"exist": true, "char_ranges": [[245, 388]], "word_ranges": [[39, 66]], "text": "Un transplante aut\u00f3logo no es razonable, ya que la m\u00e9dula \u00f3sea de un paciente con aplasia medular es menor del 25%, de ah\u00ed poco se puede sacar."}, "5": {"exist": false, "char_ranges": [], "word_ranges": [], "text": ""}}} +{"id": 270, "year": 2015, "question_id_specific": 135, "full_question": "Joven que acude a urgencias por quemadura por llama de segundo grado del 10% de la superficie corporal, afectando al brazo derecho de forma extensa y circular. No se halla pulso arterial en la mano medido por doppler. \u00bfCu\u00e1l es el tratamiento de elecci\u00f3n?", "full_answer": "La pregunta tiene trampa: habla de una quemadura del 10% de la superficie corporal y el tratamiento de elecci\u00f3n para quemaduras de segundo grado menores del 20% del BSA debe ser t\u00f3pico y vigilar. Sin embargo hay un signo de alarma: no tiene pulso arterial y la quemadura es circunferencial al miembro. Esto supone una emergencia que, de dejarse evolucionar, comprometer\u00e1 la viabilidad del miembro afecto y terminar en amputaci\u00f3n: hay que realizar una escarotom\u00eda de urgencia con la finalidad aliviar la presi\u00f3n del tercer espacio sobre el tronco vascular arterial.", "type": "DERMATOLOG\u00cdA Y CIRUG\u00cdA PL\u00c1STICA", "options": {"1": "Curas con sulfadiacina arg\u00e9ntica oclusivas y evaluaci\u00f3n de la profundidad a la semana.", "2": "Drenajes linf\u00e1ticos y valorar un by-pass vascular.", "3": "Escarotomia o incisiones de descompresi\u00f3n de urgencia.", "4": "Conducta expectante.", "5": "Amputaci\u00f3n de la extremidad."}, "correct_option": 3, "explanations": {"1": {"exist": false, "char_ranges": [], "word_ranges": [], "text": ""}, "2": {"exist": false, "char_ranges": [], "word_ranges": [], "text": ""}, "3": {"exist": true, "char_ranges": [[208, 564]], "word_ranges": [[36, 91]], "text": "hay un signo de alarma: no tiene pulso arterial y la quemadura es circunferencial al miembro. Esto supone una emergencia que, de dejarse evolucionar, comprometer\u00e1 la viabilidad del miembro afecto y terminar en amputaci\u00f3n: hay que realizar una escarotom\u00eda de urgencia con la finalidad aliviar la presi\u00f3n del tercer espacio sobre el tronco vascular arterial."}, "4": {"exist": false, "char_ranges": [], "word_ranges": [], "text": ""}, "5": {"exist": false, "char_ranges": [], "word_ranges": [], "text": ""}}} +{"id": 368, "year": 2016, "question_id_specific": 121, "full_question": "Encontr\u00e1ndose de guardia m\u00e9dica en el Servicio de Urgencias de su hospital, tiene que atender a un paciente de 64 a\u00f1os de edad con insuficiencia respiratoria aguda. Su estado cl\u00ednico es cr\u00edtico, con baja saturaci\u00f3n de ox\u00edgeno e inestabilidad hemodin\u00e1mica. Se realiza Rx urgente de t\u00f3rax que muestra atelectasia de 2/3 del pulm\u00f3n derecho. Se procede a intubaci\u00f3n orotraqueal y ventilaci\u00f3n en modo asistido, con Fi02 de 1,0. En una gasometr\u00eda arterial practicada posterior\u00ad mente obtenemos estos valores: pH 7,23; Pa02 60 mmHg y PaC02 30 mmHg. \u00bfCu\u00e1l es la causa de la hipoxemia?:", "full_answer": "La FiO2 de 1 y la hiperventilaci\u00f3n con la ventilaci\u00f3n mec\u00e1nica, demostrada por la baja PCO2, no consiguen remontar suficientemente la PO2, debido a un trastorno en la V/Q, con shunt probable en la zona de la atelectasia pulmonar, probablemente aguda, dado que el pulm\u00f3n no ha podido crear mecanismos compensatorios de limitar la perfusi\u00f3n en las zonas mal ventiladas.", "type": "NEUMOLOG\u00cdA Y CIRUG\u00cdA TOR\u00c1CICA", "options": {"1": "Cortocircuito.", "2": "Hipoventilaci\u00f3n.", "3": "Baja presi\u00f3n de 02 inspirado.", "4": "Enfermedad neuromuscular.", "5": null}, "correct_option": 1, "explanations": {"1": {"exist": true, "char_ranges": [[0, 367]], "word_ranges": [[0, 60]], "text": "La FiO2 de 1 y la hiperventilaci\u00f3n con la ventilaci\u00f3n mec\u00e1nica, demostrada por la baja PCO2, no consiguen remontar suficientemente la PO2, debido a un trastorno en la V/Q, con shunt probable en la zona de la atelectasia pulmonar, probablemente aguda, dado que el pulm\u00f3n no ha podido crear mecanismos compensatorios de limitar la perfusi\u00f3n en las zonas mal ventiladas."}, "2": {"exist": false, "char_ranges": [], "word_ranges": [], "text": ""}, "3": {"exist": false, "char_ranges": [], "word_ranges": [], "text": ""}, "4": {"exist": false, "char_ranges": [], "word_ranges": [], "text": ""}, "5": {"exist": false, "char_ranges": [], "word_ranges": [], "text": ""}}} +{"id": 167, "year": 2013, "question_id_specific": 85, "full_question": "Si en un paciente con insuficiencia cardiaca cr\u00f3nica detectamos unas ondas v prominentes en el pulso venoso yugular y en la auscultaci\u00f3n cardiaca se ausculta un soplo holosist\u00f3lico en el \u00e1rea del ap\u00e9ndice xifoides que se acent\u00faa con la inspiraci\u00f3n profunda. \u00bfCu\u00e1l es la valvulopat\u00eda responsable de esta exploraci\u00f3n f\u00edsica?", "full_answer": "Semiolog\u00eda b\u00e1sica. No obstante, razonamiento de emergencia: soplo sist\u00f3lico, algo que en s\u00edstole ten\u00eda que estar cerrado y no lo est\u00e1, o ten\u00eda que abrirse, y no lo hace: opciones 1, 3 y 5. Y si vemos repercusi\u00f3n en el pulso yugular, es que debe estar en cavidades derechas. Aparte de que la onda v aparece durante la s\u00edstole, mientras las aur\u00edculas se llenan: si el flujo del VD sube a la AD, lo que ocurrir\u00e1 es que la onda v ser\u00e1 grand\u00edsima\u2026", "type": "CARDIOLOG\u00cdA Y CIRUG\u00cdA CARDIOVASCULAR", "options": {"1": "Insuficiencia mitral.", "2": "Insuficiencia pulmonar.", "3": "Insuficiencia tric\u00faspide.", "4": "Insuficiencia a\u00f3rtica.", "5": "Estenosis a\u00f3rtica."}, "correct_option": 3, "explanations": {"1": {"exist": false, "char_ranges": [], "word_ranges": [], "text": ""}, "2": {"exist": false, "char_ranges": [], "word_ranges": [], "text": ""}, "3": {"exist": true, "char_ranges": [[191, 442]], "word_ranges": [[35, 83]], "text": "si vemos repercusi\u00f3n en el pulso yugular, es que debe estar en cavidades derechas. Aparte de que la onda v aparece durante la s\u00edstole, mientras las aur\u00edculas se llenan: si el flujo del VD sube a la AD, lo que ocurrir\u00e1 es que la onda v ser\u00e1 grand\u00edsima\u2026"}, "4": {"exist": false, "char_ranges": [], "word_ranges": [], "text": ""}, "5": {"exist": false, "char_ranges": [], "word_ranges": [], "text": ""}}} +{"id": 154, "year": 2012, "question_id_specific": 75, "full_question": "Una paciente de 42 a\u00f1os de edad refiere dolor de caracter\u00edsticas inflamatorias y tumefacci\u00f3n en ambas mu\u00f1ecas, 2\u00aa y 3\u00aa metacarpofal\u00e1ngicas e interfal\u00e1ngicas proximales de forma bilateral y tobillo izquierdo de 4 meses de evoluci\u00f3n acompa\u00f1ada de rigidez matutina de m\u00e1s de una hora de duraci\u00f3n. Esn la radiograf\u00eda de manos se objetiva una erosi\u00f3n en la ap\u00f3fisis estiloides del c\u00fabito en el carpo derecho. En la anal\u00edtica destaca una Hb: 10 g/dL con VSG de 45 mm en 1\u00aa hora, PCR 16 mg/L, factor reumatoide 160 UI/ML. Tras 6 meses de tratamiento con indometacina y metotrexato la paciente persiste con dolor y tumefacci\u00f3n de ambos carpos, rigidez matutina de 30 minutos de duraci\u00f3n y una anal\u00edtica donde destaca una VSG 30 mm en 1\u00aa hora y una PCR 9 mg/dL. Respecto a la actitud a tomar, cu\u00e1l de las siguientes es verdadera:", "full_answer": "Se trata de una paciente afecta de artritis reumatoide. En mi opini\u00f3n, como en estos seis meses ha mostrado una clara mejor\u00eda cl\u00ednica y biol\u00f3gica (dismininuci\u00f3n de reactantes de fase aguda), mantendr\u00eda la actitud terap\u00e9utica tomada y esperar\u00eda a una nueva valoraci\u00f3n. Eso s\u00ed, considero que esta pregunta podr\u00eda tener otra respuesta v\u00e1lida, la 3. Depende un poco de la actitud de cada reumat\u00f3logo. Si lo que se busca es una remisi\u00f3n de la enfermedad lo antes posible, se podr\u00eda optar por valorar asociar un anti-TNF alfa...", "type": "REUMATOLOG\u00cdA", "options": {"1": "Suspender el tratamiento pautado por la falta de respuesta e iniciar prednisona a dosis altas para el control de los s\u00edntomas exclusivamente.", "2": "Mantener la actitud terap\u00e9utica tomada dado que s\u00f3lo llevamos 6 meses y habr\u00eda que esperar un m\u00ednimo de 9 meses para valorar respuesta terap\u00e9utica.", "3": "Si no existe contraindicaci\u00f3n m\u00e9dica, valorar asociar al tratamiento un anti-TNF alfa.", "4": "Iniciar lo antes posible un segundo f\u00e1rmaco modificador de la enfermedad dado que no se podr\u00eda iniciar tratamiento con terapia biol\u00f3gica s\u00f3lo tras metotrexato.", "5": "Valorar iniciar tratamiento con terapia anti-CD20 asociada al metotrexato."}, "correct_option": 2, "explanations": {"1": {"exist": false, "char_ranges": [], "word_ranges": [], "text": ""}, "2": {"exist": true, "char_ranges": [[71, 267]], "word_ranges": [[12, 42]], "text": "como en estos seis meses ha mostrado una clara mejor\u00eda cl\u00ednica y biol\u00f3gica (dismininuci\u00f3n de reactantes de fase aguda), mantendr\u00eda la actitud terap\u00e9utica tomada y esperar\u00eda a una nueva valoraci\u00f3n."}, "3": {"exist": true, "char_ranges": [[397, 522]], "word_ranges": [[64, 87]], "text": "Si lo que se busca es una remisi\u00f3n de la enfermedad lo antes posible, se podr\u00eda optar por valorar asociar un anti-TNF alfa..."}, "4": {"exist": false, "char_ranges": [], "word_ranges": [], "text": ""}, "5": {"exist": false, "char_ranges": [], "word_ranges": [], "text": ""}}} +{"id": 176, "year": 2013, "question_id_specific": 57, "full_question": "Mujer de 56 a\u00f1os de edad, con antecedentes de esquizofrenia bien controlada, sin h\u00e1bitos t\u00f3xicos. Ingresada por neumon\u00eda en l\u00f3bulo medio con un peque\u00f1o derrame pleural metaneum\u00f3nico asociado y en tratamiento con levofloxacino 500 mg/24h . Presenta buena evoluci\u00f3n cl\u00ednica salvo por persistencia de febr\u00edcula y leucocitosis al sexto d\u00eda de tratam\u00edento. No se dispone de estudios microbiol\u00f3gicos. La conducta m\u00e1s adecuada es:", "full_answer": "Mientras persista la fiebre en una neumon\u00eda en evoluci\u00f3n, existe el riesgo de empiematizaci\u00f3n, m\u00e1xime si ya ha presentado un peque\u00f1o derrame al inicio del cuadro. El tratamiento antibi\u00f3tico en principio es correcto y antes de plantear cambios de tratamiento, debe de evaluarse la posibilidad de empiematizaci\u00f3n.", "type": "NEUMOLOG\u00cdA", "options": {"1": "La evoluci\u00f3n es normal, debe mantenerse tratamiento hasta completar los 10 d\u00edas.", "2": "Se considera un fracaso terap\u00e9utico y debe modificar el tratamiento antibi\u00f3tico.", "3": "Realizar toracocentesis para descartar empiema.", "4": "Asociar corticoides a dosis de 0.5 mg/Kg/d\u00eda al tratamiento antibi\u00f3tico.", "5": "Realizar broncoscopia con biopsia, aspiraci\u00f3n y lavado broncoalveolar."}, "correct_option": 3, "explanations": {"1": {"exist": false, "char_ranges": [], "word_ranges": [], "text": ""}, "2": {"exist": false, "char_ranges": [], "word_ranges": [], "text": ""}, "3": {"exist": true, "char_ranges": [[163, 311]], "word_ranges": [[26, 47]], "text": "El tratamiento antibi\u00f3tico en principio es correcto y antes de plantear cambios de tratamiento, debe de evaluarse la posibilidad de empiematizaci\u00f3n."}, "4": {"exist": false, "char_ranges": [], "word_ranges": [], "text": ""}, "5": {"exist": false, "char_ranges": [], "word_ranges": [], "text": ""}}} +{"id": 130, "year": 2012, "question_id_specific": 82, "full_question": "Paciente mujer de 18 a\u00f1os, con hisotria de ausencias entre los 6 y 9 a\u00f1os, crisis t\u00f3nico-cl\u00f3nicas generalizadas de reciente comienzo y saltos violentos de miembros superiores al desayunar. La cl\u00ednica empeora con salidas nocturnas de fines de semana. Un EEG muestra descargar de polipuntas agudas a 6 ciclos/segundo. El diagn\u00f3stico m\u00e1s probable es:", "full_answer": "Paciente en edad adolescente con mioclon\u00edas (\u201csaltos violentos\u201d) a la hora del desayuno, va a guiarnos casi siempre hacia la Epilepsia miocl\u00f3nica juvenil (respuesta 5 correcta), entidad muy bien caracterizada. Entre otras caracter\u00edsticas est\u00e1 la historia previa de crisis o ausencias, el empeoramiento con las salidas nocturnas, y el EEG con descargas de polipuntas generalizadas a una frecuencia mayor que el de las ausencias. Del resto de respuestas, el Lennox Gastaut se desarrolla a una edad temprana con otra sintomatolog\u00eda, la 4 es lo mismo que ausencias, y la 3 suele cursar con crisis parciales complejas y de cualquier manera, el EEG es claramente distinto a un foco temporal.", "type": "NEUROLOG\u00cdA Y NEUROCIRUG\u00cdA", "options": {"1": "Gran mal epil\u00e9ptico.", "2": "S\u00edndrome de Lennox-Gastaut.", "3": "Epilepsia sintom\u00e1tica por esclerosis temporal mesial.", "4": "Peque\u00f1o mal at\u00edpico.", "5": "Epilepsia miocl\u00f3nica juvenil."}, "correct_option": 5, "explanations": {"1": {"exist": false, "char_ranges": [], "word_ranges": [], "text": ""}, "2": {"exist": true, "char_ranges": [[453, 528]], "word_ranges": [[68, 80]], "text": "el Lennox Gastaut se desarrolla a una edad temprana con otra sintomatolog\u00eda,"}, "3": {"exist": true, "char_ranges": [[565, 612]], "word_ranges": [[88, 96]], "text": "la 3 suele cursar con crisis parciales complejas"}, "4": {"exist": true, "char_ranges": [[531, 560]], "word_ranges": [[80, 87]], "text": "la 4 es lo mismo que ausencias,"}, "5": {"exist": true, "char_ranges": [[0, 176]], "word_ranges": [[0, 26]], "text": "Paciente en edad adolescente con mioclon\u00edas (\u201csaltos violentos\u201d) a la hora del desayuno, va a guiarnos casi siempre hacia la Epilepsia miocl\u00f3nica juvenil (respuesta 5 correcta),"}}} +{"id": 276, "year": 2016, "question_id_specific": 75, "full_question": "Indique la situaci\u00f3n cl\u00ednica que, en relaci\u00f3n con la infecci\u00f3n por virus de la hepatitis B, presenta un paciente de 5 a\u00f1os procedente de Nigeria, con exploraci\u00f3n f\u00edsica normal y con la siguiente serolog\u00eda frente a hepatitis B: HBsAg + / ANTI-HBs \u2013 / HbeAg \u2013 / ANTI-HBe + / ANTI-HBc IgM \u2013 / ANTI-HBc IgG + / DNA VHB +:", "full_answer": "Que tenga anticuerpos contra el core implica contacto natural, y que sea IgG, que no es agudo. La ausencia del ant\u00edgeno e descarta la replicaci\u00f3n activa. Y la persistencia del ant\u00edgeno de superficie (HBsAg) y del ADN del virus indica que sigue presente. En conjunto, indican que se trata de un portador asintom\u00e1tico. Los portadores del mutante pre-core habitualmente tienen periodos sintom\u00e1ticos recurrentes y mayor inflamaci\u00f3n hep\u00e1tica. No habr\u00eda datos suficientes para descartarlo por completo, pero no est\u00e1 entre las respuestas e indican la exploraci\u00f3n f\u00edsica completamente normal.", "type": "APARATO DIGESTIVO", "options": {"1": "Infecci\u00f3n aguda.", "2": "Infecci\u00f3n cr\u00f3nica.", "3": "Paciente vacunado.", "4": "Portador asintom\u00e1tico.", "5": null}, "correct_option": 4, "explanations": {"1": {"exist": false, "char_ranges": [], "word_ranges": [], "text": ""}, "2": {"exist": false, "char_ranges": [], "word_ranges": [], "text": ""}, "3": {"exist": false, "char_ranges": [], "word_ranges": [], "text": ""}, "4": {"exist": true, "char_ranges": [[0, 316]], "word_ranges": [[0, 53]], "text": "Que tenga anticuerpos contra el core implica contacto natural, y que sea IgG, que no es agudo. La ausencia del ant\u00edgeno e descarta la replicaci\u00f3n activa. Y la persistencia del ant\u00edgeno de superficie (HBsAg) y del ADN del virus indica que sigue presente. En conjunto, indican que se trata de un portador asintom\u00e1tico."}, "5": {"exist": false, "char_ranges": [], "word_ranges": [], "text": ""}}} +{"id": 183, "year": 2013, "question_id_specific": 64, "full_question": "Un paciente con soporte nutricional enteral, presenta a las 72 horas de iniciar la nutrici\u00f3n enteral, una anal\u00edtica que muestra una hipofosforemia e hipopotasemia, con cl\u00ednica de insuficiencia cardiaca. El paciente es diagnosticado de s\u00edndrome de realimentaci\u00f3n. Indique cu\u00e1l de los siguientes factores NO se considera de riesgo para que un paciente presente este cuadro:", "full_answer": "El s\u00edndrome de realimentaci\u00f3n ocurre en pacientes con desnutrici\u00f3n previa expuestos a tratamiento nutricional bien oral, enteral o parenteral. Todas las respuestas son causa de desnutrici\u00f3n menos la 3.", "type": "ENDOCRINOLOG\u00cdA", "options": {"1": "Malnutrici\u00f3n cal\u00f3rica previa.", "2": "Anorexia nerviosa.", "3": "Obesidad no m\u00f3rbida.", "4": "Ancianos.", "5": "V\u00f3mitos y diarrea prolongados."}, "correct_option": 3, "explanations": {"1": {"exist": true, "char_ranges": [[0, 201]], "word_ranges": [[0, 29]], "text": "El s\u00edndrome de realimentaci\u00f3n ocurre en pacientes con desnutrici\u00f3n previa expuestos a tratamiento nutricional bien oral, enteral o parenteral. Todas las respuestas son causa de desnutrici\u00f3n menos la 3."}, "2": {"exist": true, "char_ranges": [[0, 201]], "word_ranges": [[0, 29]], "text": "El s\u00edndrome de realimentaci\u00f3n ocurre en pacientes con desnutrici\u00f3n previa expuestos a tratamiento nutricional bien oral, enteral o parenteral. Todas las respuestas son causa de desnutrici\u00f3n menos la 3."}, "3": {"exist": true, "char_ranges": [[0, 201]], "word_ranges": [[0, 29]], "text": "El s\u00edndrome de realimentaci\u00f3n ocurre en pacientes con desnutrici\u00f3n previa expuestos a tratamiento nutricional bien oral, enteral o parenteral. Todas las respuestas son causa de desnutrici\u00f3n menos la 3."}, "4": {"exist": true, "char_ranges": [[0, 201]], "word_ranges": [[0, 29]], "text": "El s\u00edndrome de realimentaci\u00f3n ocurre en pacientes con desnutrici\u00f3n previa expuestos a tratamiento nutricional bien oral, enteral o parenteral. Todas las respuestas son causa de desnutrici\u00f3n menos la 3."}, "5": {"exist": true, "char_ranges": [[0, 201]], "word_ranges": [[0, 29]], "text": "El s\u00edndrome de realimentaci\u00f3n ocurre en pacientes con desnutrici\u00f3n previa expuestos a tratamiento nutricional bien oral, enteral o parenteral. Todas las respuestas son causa de desnutrici\u00f3n menos la 3."}}} +{"id": 115, "year": 2012, "question_id_specific": 98, "full_question": "Mujer de 32 a\u00f1os con par\u00e1lisis cerebral del parto que acude a Urgencias por cuadro de orinas oscuras de unos d\u00edas de evoluci\u00f3n en relaci\u00f3n con episodio de fiebre alta y tos seca. Al ingreso se objetiva en hemograma 16900 leucocitos/mm3 (85% S, 11% L, 4% M), hemoglobina de 6.3 gr/dl; VCM 109 fl, 360000 plaquetas/mm3. En la bioqu\u00edmica LDH 2408; bilirrubina 6.8 mg/dl, (bilirrubina no conjugada 6.1 mg/dl), GOT y GPT normales. En el estudio morfol\u00f3gico de sangre se observa anisocitosis macroc\u00edtica con frecuentes formas esferocitadas y policromatofilia sin blastos. El estudio de anticuerpos irregulares es positivo en forma de panaglutinina, dificultando la prueba cruzada. \u00bfCu\u00e1l ser\u00eda su sospecha y el tratamiento m\u00e1s adecuado?", "full_answer": "Un enunciado largo lleno de datos bastante liosos. Podemos atacar esa pregunta de dos maneras: fij\u00e1ndonos en los datos claves y yendo del tir\u00f3n al diagn\u00f3stico que consideramos o descartando una por una. La coluria, la LDH, la presencia de panaglutinina y el estudio del frotis de sangre perif\u00e9rica \u00abhuelen\u00bb a anemia hemol\u00edtica. La respuesta 1 no puede ser porque una aplasia medular no explica la coluria, la elevaci\u00f3n de LDH, ni en el estudio de anticuerpos irregulares es positivo en forma de panaglutinina; una aplasia es una insuficiencia medular que se caracteriza por una desaparici\u00f3n total o parcial de progenitores hemopoy\u00e9ticos. Adem\u00e1s, no se observa una pancitopenia, que es lo que nos inclinar\u00eda m\u00e1s por esta patolog\u00eda. La respuesta 2 tampoco es posible, ya que una esferocitosis no justifica la presencia de panaglutinina. La respuesta 3 es la que considero correcta: una anemia hemol\u00edtica autoinmune s\u00ed justificar\u00eda los datos que nos dan: elevaci\u00f3n de LDH y bilirrubinemia por destrucci\u00f3n de los hemat\u00edes, policromatofilia, esferocitosis y anisocitosis porque la m\u00e9dula est\u00e1 trabajando a marchas forzadas para intentar compensar la anemia, que es regenerativa. El estudio de anticuerpos irregulares y la presencia de panaglutinina tambi\u00e9n es un dato a favor de esta respuesta, ya que al unirse un anticuerpo al hemat\u00ede propicia su lisis y destrucci\u00f3n. La chica presenta tos y fiebre, lo que concuerda con una infecci\u00f3n respiratoria. Y el tratamiento inicial son los corticoides. Lo confirma Sans Sabrafens en su libro \u00abHematolog\u00eda cl\u00ednica\u00bb [1]. La respuesta 4 da lugar a bastantes dudas; no es posible porque una anemia perniciosa no justifica la presencia de panaglutinina aunque s\u00ed justificar\u00eda la elevaci\u00f3n de LDH y bilirrubina; adem\u00e1s, es una hem\u00f3lisis intramedular, arregenerativa, no hay reticulocitosis ni lanzamiento a la sangre de formas inmaduras en un intento de compensar y arreglar la situaci\u00f3n. La respuesta 5 no es cierta porque no se observan blastos en sangre ni explica la presencia de panaglutinina.", "type": "HEMATOLOG\u00cdA", "options": {"1": "Aplasia medular e inmunoterapia con timoglobulina y ciclosporina.", "2": "Esferocitosis hereditaria y esplenectom\u00eda.", "3": "Anemia hemol\u00edtica autoinmune asociada a infecci\u00f3n respiratoria y corticoides.", "4": "Anemia perniciosa e inyecciones peri\u00f3dicas de vitamina B12.", "5": "Leucemia aguda y quimioterapia."}, "correct_option": 3, "explanations": {"1": {"exist": true, "char_ranges": [[328, 508]], "word_ranges": [[53, 84]], "text": "La respuesta 1 no puede ser porque una aplasia medular no explica la coluria, la elevaci\u00f3n de LDH, ni en el estudio de anticuerpos irregulares es positivo en forma de panaglutinina;"}, "2": {"exist": true, "char_ranges": [[731, 834]], "word_ranges": [[118, 134]], "text": "La respuesta 2 tampoco es posible, ya que una esferocitosis no justifica la presencia de panaglutinina."}, "3": {"exist": true, "char_ranges": [[835, 1173]], "word_ranges": [[134, 183]], "text": "La respuesta 3 es la que considero correcta: una anemia hemol\u00edtica autoinmune s\u00ed justificar\u00eda los datos que nos dan: elevaci\u00f3n de LDH y bilirrubinemia por destrucci\u00f3n de los hemat\u00edes, policromatofilia, esferocitosis y anisocitosis porque la m\u00e9dula est\u00e1 trabajando a marchas forzadas para intentar compensar la anemia, que es regenerativa."}, "4": {"exist": true, "char_ranges": [[1558, 1921]], "word_ranges": [[245, 301]], "text": "La respuesta 4 da lugar a bastantes dudas; no es posible porque una anemia perniciosa no justifica la presencia de panaglutinina aunque s\u00ed justificar\u00eda la elevaci\u00f3n de LDH y bilirrubina; adem\u00e1s, es una hem\u00f3lisis intramedular, arregenerativa, no hay reticulocitosis ni lanzamiento a la sangre de formas inmaduras en un intento de compensar y arreglar la situaci\u00f3n."}, "5": {"exist": true, "char_ranges": [[1922, 2031]], "word_ranges": [[301, 320]], "text": "La respuesta 5 no es cierta porque no se observan blastos en sangre ni explica la presencia de panaglutinina."}}} +{"id": 75, "year": 2012, "question_id_specific": 65, "full_question": "Un paciente ingresado por pancreatitis aguda comienza con taquipnea, taquicardia, sudoraci\u00f3n y cianosis progresiva. La PaO2 es de 55 mm Hg (cociente PaO2/FiO2<200). LA RxT muestra infiltrados pulmonares alveolares bilaterales. La presi\u00f3n de enclavamiento capilar pulmonar es normal. La oxigenoterapia no mejora la situaci\u00f3n. Que diagn\u00f3stico es el mas probable:", "full_answer": "Pregunta con respuesta directa. La pregunta cuenta el caso de un paciente con una pancreatitis aguda que est\u00e1 desarrollando un cuadro de insuficiencia respiratoria aguda. En el enunciado se enumeran los criterios de SDRA (s\u00edndrome de distr\u00e9s respiratorio agudo): PaO2/fiO2 < 200 + infiltrados pulmonares bilaterales + PCP<18. Por lo que el diagn\u00f3stico es distr\u00e9s respiratorio y por tanto la respuesta correcta es la opci\u00f3n 5.", "type": "ANESTESIOLOG\u00cdA, CUIDADOS CR\u00cdTICOS Y URGENCIAS", "options": {"1": "Neumon\u00eda nosocomial.", "2": "Insuficiencia cardiaca.", "3": "Linfangitis carcinomatosa.", "4": "Tromboembolismo pulmonar.", "5": "Distress respiratorio."}, "correct_option": 5, "explanations": {"1": {"exist": false, "char_ranges": [], "word_ranges": [], "text": ""}, "2": {"exist": false, "char_ranges": [], "word_ranges": [], "text": ""}, "3": {"exist": false, "char_ranges": [], "word_ranges": [], "text": ""}, "4": {"exist": false, "char_ranges": [], "word_ranges": [], "text": ""}, "5": {"exist": true, "char_ranges": [[171, 425]], "word_ranges": [[25, 66]], "text": "En el enunciado se enumeran los criterios de SDRA (s\u00edndrome de distr\u00e9s respiratorio agudo): PaO2/fiO2 < 200 + infiltrados pulmonares bilaterales + PCP<18. Por lo que el diagn\u00f3stico es distr\u00e9s respiratorio y por tanto la respuesta correcta es la opci\u00f3n 5."}}} +{"id": 191, "year": 2013, "question_id_specific": 125, "full_question": "Lactante de 13 meses que acude a urgencias por fiebre de hasta 39\u00baC de 48h de evoluci\u00f3n sin otra sintomatolog\u00eda asociada. Exploraci\u00f3n por \u00f3rganos y aparatos sin hallazgos significativos, destacando buen estado general. Usted le iba entregar el alta domiciliaria pero el adjunto de Pediatr\u00eda que est\u00e1 de guardia le pide sistem\u00e1tico de orina y un urocultivo por sondaje. En la orina destaca leucocituria ++, hematuria + y nitritos ++ y en el Gram de orina se observan bacilos Gram negativos. En la anal\u00edtica de sangre no existe leucocitosis y la prote\u00edna C reactiva es de 50 mg/l. El adjunto le dice ahora que no es preciso que el ni\u00f1o ingrese y que le paute un antibi\u00f3tico oral. Se\u00f1ale el tratamiento antibi\u00f3tico emp\u00edrico menos adecuado en este caso:", "full_answer": "El resto de f\u00e1rmacos son una buena elecci\u00f3n en las infecciones de orina por bacilos gran negativos (el m\u00e1s probable: E. coli). La presencia de leucocitos+ nitritos en a orina es muy sugestiva de ITU. Y si el Gram de orina detecta bacterias G- probablemente se trate de una ITU por E Coli. Muchos E Coli son resistentes a amoxicilina.", "type": "PEDIATR\u00cdA", "options": {"1": "Amoxicilina.", "2": "Amoxicilina-clavul\u00e1nico.", "3": "Cefuroxima-axetilo.", "4": "Cotrimoxazol.", "5": "Cefixima."}, "correct_option": 1, "explanations": {"1": {"exist": true, "char_ranges": [[127, 333]], "word_ranges": [[22, 60]], "text": "La presencia de leucocitos+ nitritos en a orina es muy sugestiva de ITU. Y si el Gram de orina detecta bacterias G- probablemente se trate de una ITU por E Coli. Muchos E Coli son resistentes a amoxicilina."}, "2": {"exist": true, "char_ranges": [[0, 98]], "word_ranges": [[0, 17]], "text": "El resto de f\u00e1rmacos son una buena elecci\u00f3n en las infecciones de orina por bacilos gran negativos"}, "3": {"exist": true, "char_ranges": [[0, 98]], "word_ranges": [[0, 17]], "text": "El resto de f\u00e1rmacos son una buena elecci\u00f3n en las infecciones de orina por bacilos gran negativos"}, "4": {"exist": true, "char_ranges": [[0, 98]], "word_ranges": [[0, 17]], "text": "El resto de f\u00e1rmacos son una buena elecci\u00f3n en las infecciones de orina por bacilos gran negativos"}, "5": {"exist": true, "char_ranges": [[0, 98]], "word_ranges": [[0, 17]], "text": "El resto de f\u00e1rmacos son una buena elecci\u00f3n en las infecciones de orina por bacilos gran negativos"}}} +{"id": 323, "year": 2016, "question_id_specific": 150, "full_question": "Ni\u00f1o de 12 meses de edad, que en los ex\u00e1menes de salud practicados desde el nacimiento presenta test\u00edculo derecho en conducto inguinal que no es posible descender hasta el escroto. Se\u00f1ale la respuesta CORRECTA:", "full_answer": "Se trata de una criptorquidia. Hay que tratarlo antes de cumplir 2 a\u00f1os, y actualmente el tratamiento hormonal est\u00e1 en desuso. El tratamiento de primera elecci\u00f3n es la orquidopexia.", "type": "UROLOG\u00cdA", "options": {"1": "El diagn\u00f3stico m\u00e1s probable es el de test\u00edculo retr\u00e1ctil.", "2": "Se ha de esperar hasta los dos a\u00f1os de edad a que ocurra descenso espont\u00e1neo del mismo.", "3": "La gonadotropina cori\u00f3nica humana es el tratamiento de primera elecci\u00f3n.", "4": "La indicaci\u00f3n de orquidopexia no debe diferirse.", "5": null}, "correct_option": 4, "explanations": {"1": {"exist": false, "char_ranges": [], "word_ranges": [], "text": ""}, "2": {"exist": false, "char_ranges": [], "word_ranges": [], "text": ""}, "3": {"exist": false, "char_ranges": [], "word_ranges": [], "text": ""}, "4": {"exist": true, "char_ranges": [[0, 181]], "word_ranges": [[0, 29]], "text": "Se trata de una criptorquidia. Hay que tratarlo antes de cumplir 2 a\u00f1os, y actualmente el tratamiento hormonal est\u00e1 en desuso. El tratamiento de primera elecci\u00f3n es la orquidopexia."}, "5": {"exist": false, "char_ranges": [], "word_ranges": [], "text": ""}}} +{"id": 311, "year": 2016, "question_id_specific": 217, "full_question": "Mujer de 84 a\u00f1os que acude por p\u00e9rdida de visi\u00f3n en el ojo izquierdo de 4 d\u00edas de evoluci\u00f3n acompa\u00f1ada de metamorfopsias. En la m\u00e1cula se aprecian abundantes exudados duros, dos peque\u00f1as hemorragias profundas y un desprendimiento de la retina neurosensorial localizado. En el ojo contralateral presenta abundantes drusas blandas. Ante este cuadro, \u00bfCu\u00e1l de los siguientes diagn\u00f3sticos le parece m\u00e1s probable?", "full_answer": "Ninguna de las otras tres opciones da esa cl\u00ednica (el DPV suele ser asintom\u00e1tico, la OACR produce p\u00e9rdida brusca y total o casi total de la visi\u00f3n con la \u201cmancha rojo cereza\u201d t\u00edpica en el fondo de ojo, y una NOIA tanto arter\u00edtica como no arter\u00edtica producir\u00eda edema de papila habitualmente con hemorragias peripapilares pero no maculares, y ninguna de estas 3 opciones tiene relaci\u00f3n con drusas en el ojo contralateral). La DMAE en su forma exudativa se presenta tanto con esa cl\u00ednica (p\u00e9rdida de visi\u00f3n y metamorfopsias) como el fondo de ojo caracter\u00edstico descrito en la pregunta, adem\u00e1s de que habitualmente se observan drusas en el ojo contralateral.", "type": "OFTALMOLOG\u00cdA", "options": {"1": "Desprendimiento posterior de v\u00edtreo agudo.", "2": "Degeneraci\u00f3n macular asociada a la edad (DMAE) exudativa.", "3": "Obstrucci\u00f3n de arteria central de la retina.", "4": "Neuropat\u00eda \u00f3ptica isqu\u00e9mica anterior no arter\u00edtica.", "5": null}, "correct_option": 2, "explanations": {"1": {"exist": true, "char_ranges": [[341, 418]], "word_ranges": [[58, 71]], "text": "ninguna de estas 3 opciones tiene relaci\u00f3n con drusas en el ojo contralateral)."}, "2": {"exist": true, "char_ranges": [[421, 654]], "word_ranges": [[71, 109]], "text": "La DMAE en su forma exudativa se presenta tanto con esa cl\u00ednica (p\u00e9rdida de visi\u00f3n y metamorfopsias) como el fondo de ojo caracter\u00edstico descrito en la pregunta, adem\u00e1s de que habitualmente se observan drusas en el ojo contralateral."}, "3": {"exist": true, "char_ranges": [[341, 418]], "word_ranges": [[58, 71]], "text": "ninguna de estas 3 opciones tiene relaci\u00f3n con drusas en el ojo contralateral)."}, "4": {"exist": true, "char_ranges": [[341, 418]], "word_ranges": [[58, 71]], "text": "ninguna de estas 3 opciones tiene relaci\u00f3n con drusas en el ojo contralateral)."}, "5": {"exist": false, "char_ranges": [], "word_ranges": [], "text": ""}}} +{"id": 137, "year": 2012, "question_id_specific": 164, "full_question": "Ante un traumatismo directo sobre el lado derecho de la cara tras el cual el paciente presenta hematoma palpebral unilateral, diplopia en la mirada vertical y dificultad para la apertura de la boca, \u00bfcu\u00e1l de las siguientes afirmaciones es cierta?", "full_answer": "La diplopia en la mirada vertical es muy sugerente de que las paredes \u00f3seas de la \u00f3rbita est\u00e1n afectadas. La fractura de Lefort tipo I no afecta al contenido orbitario. Conociendo esta informaci\u00f3n, la \u00fanica respuesta correcta es la 2. Si dud\u00e1bamos acerca de si la fractura de Lefort tipo I afectaba la \u00f3rbita o no, simplemente mirad las respuestas. Quien puso la pregunta quiso asegurarse de que la \u00fanica respuesta v\u00e1lida fuera la n\u00famero 2; por eso a\u00f1adi\u00f3 la coletilla de \u00abpero por si acaso lo comprobamos con una TAC\u00bb.", "type": "OTORRINOLARINGOLOG\u00cdA Y CIRUG\u00cdA MAXILOFACIAL", "options": {"1": "Estamos con gran probabilidad ante una fractura de tercio medio facial tipo Lefort I.", "2": "Se trata probablemente de una fractura \u00f3rbitomalar unilateral. El diagn\u00f3stico se comprobar\u00eda idealmente mediante TAC (tomograf\u00eda axial computerizada).", "3": "Se trata de una fractura de la base del cr\u00e1neo a nivel del agujero carot\u00eddeo.", "4": "El diagn\u00f3stico probable es la fractura con dislocaci\u00f3n del c\u00f3ndilo mandibular.", "5": "Probablemente se asocia una fractura mandibular con una fractura del tercio medio facial tipo Lefort I."}, "correct_option": 2, "explanations": {"1": {"exist": true, "char_ranges": [[106, 168]], "word_ranges": [[19, 30]], "text": "La fractura de Lefort tipo I no afecta al contenido orbitario."}, "2": {"exist": true, "char_ranges": [[0, 105]], "word_ranges": [[0, 19]], "text": "La diplopia en la mirada vertical es muy sugerente de que las paredes \u00f3seas de la \u00f3rbita est\u00e1n afectadas."}, "3": {"exist": false, "char_ranges": [], "word_ranges": [], "text": ""}, "4": {"exist": false, "char_ranges": [], "word_ranges": [], "text": ""}, "5": {"exist": false, "char_ranges": [], "word_ranges": [], "text": ""}}} +{"id": 309, "year": 2016, "question_id_specific": 215, "full_question": "Un hombre con miop\u00eda magna, de 47 a\u00f1os, intervenido de cataratas hace 2 a\u00f1os acude a urgencias refiriendo una p\u00e9rdida profunda e indolora de la visi\u00f3n en su ojo derecho. \u00bfCu\u00e1l de los siguientes diagn\u00f3sticos puede provocar esta sintomatolog\u00eda?", "full_answer": "Esta pregunta podr\u00eda ser controvertida. La opci\u00f3n 1 queda descartada rotundamente porque una endoftalmitis cursa de forma t\u00edpica con mucho dolor. La opci\u00f3n 3 de forma h\u00fameda de degeneraci\u00f3n macular asociada a la edad tambi\u00e9n queda descartada, porque habitualmente se da en pacientes de edad avanzada, aunque lo que s\u00ed puede presentar un paciente con miop\u00eda magna es una degeneraci\u00f3n macular mi\u00f3pica (con una cl\u00ednica similar a la DMAE, pero se trata de una patolog\u00eda diferente). Un desprendimiento posterior de v\u00edtreo (DPV) es habitualmente poco sintom\u00e1tico, produciendo miodesopsias pero no p\u00e9rdida de visi\u00f3n ni dolor; sin embargo, algunos DPV son de tipo hemorr\u00e1gico, al traccionar de los vasos, produciendo una hemorragia v\u00edtrea que s\u00ed cursar\u00eda con p\u00e9rdida de visi\u00f3n. De todas formas, por el antecedente de miop\u00eda magna y cirug\u00eda intraocular previa, el primer diagn\u00f3stico a descartar ser\u00eda un desprendimiento de retina, ya que son dos factores de riesgo para esta patolog\u00eda. T\u00e9cnicamente, tanto la opci\u00f3n 2 como la 4 (un DPV agudo hemorr\u00e1gico) podr\u00edan provocar estos s\u00edntomas, pero la opci\u00f3n m\u00e1s correcta es la 2 (y estoy segura de que es la que el examinador quiere que se conteste).", "type": "OFTALMOLOG\u00cdA", "options": {"1": "Endoftalmitis postquir\u00fargica.", "2": "Desprendimiento de retina.", "3": "Degeneraci\u00f3n macular asociada a la edad, forma h\u00fameda.", "4": "Desprendimiento posterior de v\u00edtreo.", "5": null}, "correct_option": 2, "explanations": {"1": {"exist": true, "char_ranges": [[40, 145]], "word_ranges": [[5, 21]], "text": "La opci\u00f3n 1 queda descartada rotundamente porque una endoftalmitis cursa de forma t\u00edpica con mucho dolor."}, "2": {"exist": true, "char_ranges": [[977, 1186]], "word_ranges": [[152, 190]], "text": "T\u00e9cnicamente, tanto la opci\u00f3n 2 como la 4 (un DPV agudo hemorr\u00e1gico) podr\u00edan provocar estos s\u00edntomas, pero la opci\u00f3n m\u00e1s correcta es la 2 (y estoy segura de que es la que el examinador quiere que se conteste)."}, "3": {"exist": true, "char_ranges": [[146, 299]], "word_ranges": [[21, 46]], "text": "La opci\u00f3n 3 de forma h\u00fameda de degeneraci\u00f3n macular asociada a la edad tambi\u00e9n queda descartada, porque habitualmente se da en pacientes de edad avanzada,"}, "4": {"exist": true, "char_ranges": [[977, 1186]], "word_ranges": [[152, 190]], "text": "T\u00e9cnicamente, tanto la opci\u00f3n 2 como la 4 (un DPV agudo hemorr\u00e1gico) podr\u00edan provocar estos s\u00edntomas, pero la opci\u00f3n m\u00e1s correcta es la 2 (y estoy segura de que es la que el examinador quiere que se conteste)."}, "5": {"exist": false, "char_ranges": [], "word_ranges": [], "text": ""}}} +{"id": 279, "year": 2016, "question_id_specific": 32, "full_question": "Mujer de 59 a\u00f1os que presenta diarrea cr\u00f3nica acuosa de 4 meses de evoluci\u00f3n. En la endoscopia, la mucosa no mostraba aspectos relevantes. En concreto, no se observaron \u00falceras o \u00e1reas friables. Se realiz\u00f3 biopsia del colon transverso. En el estudio histopatol\u00f3gico se reconoci\u00f3 un \u00e1rea engrosada por debajo del epitelio de revestimiento superficial, que era m\u00e1s evidente mediante t\u00e9cnica de tricr\u00f3mico de Masson y que conllevaba atrofia y denudaci\u00f3n epitelial. Tambi\u00e9n se apreci\u00f3 un claro incremento en la densidad de linfocitos intraepiteliales. \u00bfEl diagn\u00f3stico de la lesi\u00f3n intestinal es?", "full_answer": "En este caso, la presencia de engrosamiento de la capa de col\u00e1geno subepitelial, m\u00e1s evidente con el tricr\u00f3mico de Masson (tinci\u00f3n especial que permite diferenciar las fibras de col\u00e1geno), es patognom\u00f3nica de la colitis col\u00e1gena. La atrofia junto con la denudaci\u00f3n epitelial son caracter\u00edsticas de esta enfermedad, la cual no suele causar alteraciones macrosc\u00f3picas de la mucosa. Un incremento de la densidad de linfocitos intraepiteliales y la ausencia de menci\u00f3n de alteraciones arquitecturales de las criptas (caracter\u00edsticas de la colitis ulcerosa y la enfermedad de Crohn) nos apoyan en este diagn\u00f3stico.", "type": "ANATOM\u00cdA PATOL\u00d3GICA", "options": {"1": "Colitis ulcerosa cronificada.", "2": "Colitis pseudomembranosa.", "3": "Colitis col\u00e1gena.", "4": "Enfermedad de Crohn fibrosante.", "5": null}, "correct_option": 3, "explanations": {"1": {"exist": false, "char_ranges": [], "word_ranges": [], "text": ""}, "2": {"exist": false, "char_ranges": [], "word_ranges": [], "text": ""}, "3": {"exist": true, "char_ranges": [[14, 229]], "word_ranges": [[3, 35]], "text": "la presencia de engrosamiento de la capa de col\u00e1geno subepitelial, m\u00e1s evidente con el tricr\u00f3mico de Masson (tinci\u00f3n especial que permite diferenciar las fibras de col\u00e1geno), es patognom\u00f3nica de la colitis col\u00e1gena."}, "4": {"exist": false, "char_ranges": [], "word_ranges": [], "text": ""}, "5": {"exist": false, "char_ranges": [], "word_ranges": [], "text": ""}}} +{"id": 437, "year": 2018, "question_id_specific": 118, "full_question": "Luis es un joven de 25 a\u00f1os que sufri\u00f3 esplenectomia tras accidente de bicicleta hace 1 a\u00f1o. Tiene un perro que le mordi\u00f3 hace 24 horas y le ha producido una peque\u00f1a herida en la mano derecha. Acude a su centro de salud (situada a 3 horas del hospital mas cercano) por fiebre de 39\u00baC, dolor en la herida y malestar general. A la exploraci\u00f3n PA 100/60 mm Hg, FC 110 latidos por minuto, ligera inflamaci\u00f3n en la herida sin pus. \u00bfCual de las siguientes actuaciones esta mas indicada en este momento?", "full_answer": "Al considerar un paciente \u201cespecial\u201d a un esplenectomizado, este recibir\u00eda tratamiento antibi\u00f3tico lo antes posible aunque la herida no aparezca infectada. Hay que tener que el perro es suyo (se deber\u00eda preguntar si el animal est\u00e1 correctamente vacunado) y si el paciente est\u00e1 correctamente vacunado (5 dosis de t\u00e9tanos, no requerir\u00eda antitet\u00e1nica); realizar una notificaci\u00f3n de Agresi\u00f3n por Animal, para observaci\u00f3n de este y tomar medidas en caso de que esto sea necesario.", "type": "EPIDEMIOLOG\u00cdA Y MEDICINA PREVENTIVA", "options": {"1": "Enviar al hospital para vacunar de rabia y t\u00e9tanos y mantener en observaci\u00f3n.", "2": "Limpiar la herida y administrar ganunaglobulina inespec\u00edfica intramuscular.", "3": "Dar 400 mg de moxifloxacino oral y enviar al hospital.", "4": "Dar clindamicina 600 mg oral cada 8 horas y observaci\u00f3n.", "5": null}, "correct_option": 3, "explanations": {"1": {"exist": false, "char_ranges": [], "word_ranges": [], "text": ""}, "2": {"exist": false, "char_ranges": [], "word_ranges": [], "text": ""}, "3": {"exist": true, "char_ranges": [[0, 155]], "word_ranges": [[0, 21]], "text": "Al considerar un paciente \u201cespecial\u201d a un esplenectomizado, este recibir\u00eda tratamiento antibi\u00f3tico lo antes posible aunque la herida no aparezca infectada."}, "4": {"exist": false, "char_ranges": [], "word_ranges": [], "text": ""}, "5": {"exist": false, "char_ranges": [], "word_ranges": [], "text": ""}}} +{"id": 112, "year": 2012, "question_id_specific": 154, "full_question": "Paciente de 67 a\u00f1os con antecedente de menopausia a los 55 a\u00f1os, 3 gestaciones con 3 partos eut\u00f3cicos, diabetis tipo 2 de 6 a\u00f1os de evoluci\u00f3n, tratamiento con nifedipono por HTA. Consulta por cuadro de metrorr\u00e0gia escassa intermitente de 2 meses de evoluci\u00f3n. En la exploraci\u00f3n ginecol\u00f3gica se aprecian unos genitales externos sin lesiones, un c\u00e9rvix de aspecto atr\u00f3fico, un \u00fatero y anejos normales a la palpaci\u00f3n y un estudio citol\u00f3gico de c\u00e9rvix normal. En el estudio ecogr\u00e1fico transvaginal se aprecia un endometrio hiperecog\u00e9nico de 7mm. \u00bfCu\u00e1l de las siguientes pruebas es la m\u00e1s indicada y con mayor sensibilidad para establecer un diagnostico?", "full_answer": "La respuesta correcta es la 3. El estudio que nos aporta mayor informaci\u00f3n sobre la patolog\u00eda endometrial es la histeroscopia, que nos permite realizar una biopsia dirigida de endometrio. La citologia endometrial se realiza a ciegas y tiene gran cantidad de falsos negativos. La resonancia nos ayudar\u00e1 a estadiar un c\u00e1ncer de endometrio.", "type": "GINECOLOG\u00cdA Y OBSTETRICIA", "options": {"1": "Conizaci\u00f3n de c\u00e8rvix.", "2": "Citolog\u00eda endometrial.", "3": "Histeroscopia y biopsia de endometrio.", "4": "Resonancia nuclear magn\u00e9tica de pelvis.", "5": "Exploraci\u00f3n bajo anestesia de aparato genital y bi\u00f2psia de c\u00e9rvix y endometrio."}, "correct_option": 3, "explanations": {"1": {"exist": false, "char_ranges": [], "word_ranges": [], "text": ""}, "2": {"exist": true, "char_ranges": [[188, 275]], "word_ranges": [[29, 43]], "text": "La citologia endometrial se realiza a ciegas y tiene gran cantidad de falsos negativos."}, "3": {"exist": true, "char_ranges": [[31, 187]], "word_ranges": [[6, 29]], "text": "El estudio que nos aporta mayor informaci\u00f3n sobre la patolog\u00eda endometrial es la histeroscopia, que nos permite realizar una biopsia dirigida de endometrio."}, "4": {"exist": true, "char_ranges": [[276, 337]], "word_ranges": [[43, 53]], "text": "La resonancia nos ayudar\u00e1 a estadiar un c\u00e1ncer de endometrio."}, "5": {"exist": false, "char_ranges": [], "word_ranges": [], "text": ""}}} +{"id": 374, "year": 2016, "question_id_specific": 101, "full_question": "Hombre de 52 a\u00f1os que acude al servicio de Urgencias por cefalea y fiebre (37,8\u00b0C) de 2 dias de evoluci\u00f3n. En las \u00faltimas horas asocia adem\u00e1s diticultad para la nominaci\u00f3n y comprensi\u00f3n. En la exploraci\u00f3n realizada no se observa rigidez de nuca siendo lo m\u00e1s llamativo la presencia de una afasia mixta, El fondo de ojo es normal. La TC craneal muestra una tenue hipodensidad en l\u00f3bulo temporal izquierdo sin efecto de masa y sin captaci\u00f3n de contraste. \u00bfCu\u00e1l de las siguientes afirmaciones es la correcta?:", "full_answer": "El cuadro que nos describen es t\u00edpico de una meningoencefalitis v\u00edrica, m\u00e1s concretamente por VHZ, con importante tropismo por el l\u00f3bulo temporal, causando febr\u00edcula y en muchos casos afasia, por lo que la opci\u00f3n correcta ser\u00eda la 2, ya que el l\u00edquido cefalorraqu\u00eddeo normalmente muestra una pleocitosis linfocitaria moderada, con glucorraquia normal. No son correctas el resto por: La primera porque no es una meningitis bacteriana que cursar\u00eda con m\u00e1s fiebre, la tercera porque la encefalitis l\u00edmbica normalmente cursa de forma afebril con cambios de comportamiento y la \u00faltima porque la imagen que nos describen no es un absceso, bien delimitado.", "type": "NEUROLOG\u00cdA", "options": {"1": "Una meningitis bacteriana es la primera impresi\u00f3n diagn\u00f3stica y hay que iniciar cuanto antes tratamiento con cefalosporina de 3a generaci\u00f3n.", "2": "Lo m\u00e1s probable es que el LCR de este paciente nos muestre una pleocitosis de predominio linfocitario con glucorraquia normal.", "3": "Sospechar\u00edamos una encefalitis l\u00edmbica.", "4": "Se trata de un absceso cerebral en fase inicial.", "5": null}, "correct_option": 2, "explanations": {"1": {"exist": true, "char_ranges": [[383, 460]], "word_ranges": [[58, 71]], "text": "La primera porque no es una meningitis bacteriana que cursar\u00eda con m\u00e1s fiebre,"}, "2": {"exist": true, "char_ranges": [[0, 351]], "word_ranges": [[0, 52]], "text": "El cuadro que nos describen es t\u00edpico de una meningoencefalitis v\u00edrica, m\u00e1s concretamente por VHZ, con importante tropismo por el l\u00f3bulo temporal, causando febr\u00edcula y en muchos casos afasia, por lo que la opci\u00f3n correcta ser\u00eda la 2, ya que el l\u00edquido cefalorraqu\u00eddeo normalmente muestra una pleocitosis linfocitaria moderada, con glucorraquia normal."}, "3": {"exist": true, "char_ranges": [[463, 567]], "word_ranges": [[71, 86]], "text": "la tercera porque la encefalitis l\u00edmbica normalmente cursa de forma afebril con cambios de comportamiento"}, "4": {"exist": true, "char_ranges": [[571, 649]], "word_ranges": [[87, 101]], "text": "la \u00faltima porque la imagen que nos describen no es un absceso, bien delimitado."}, "5": {"exist": false, "char_ranges": [], "word_ranges": [], "text": ""}}} +{"id": 116, "year": 2012, "question_id_specific": 101, "full_question": "Mujer de 25 a\u00f1os de edad que desea quedarse embarazada y quiere saber qu\u00e9 tratamiento debe realizar durante el eventual embarazo, ya que es portadora del factor V Leiden en heterocigosis. Nunca ha tenido ning\u00fan fen\u00f3meno tromb\u00f3tico. Se realiz\u00f3 la determinaci\u00f3n del mencionado factor como estudio familiar tras un episodio de embolia de pulm\u00f3n en un hermano. \u00bfQu\u00e9 tratamiento se debe aconsejar?", "full_answer": "Al leer las respuestas, lo primero que tenemos claro es que NO se debe administrar acenocumarol a una mujer embarazada por riesgo de teratogenicidad. Al que lo haga durante la residencia se le debe castigar con 100 fonendazos en la espalda. Descartamos respuesta 5. Lo segundo: si se pone tratamiento, debe ser durante todo el embarazo, el status protromb\u00f3tico no es s\u00f3lo durante el puerperio, as\u00ed que respuesta 2 descartada. \u00bfAconsejar a una mujer que no se quede embarazada habiendo tratamientos que aseguren un baj\u00edsimo riesgo de trombosis y/o afectaci\u00f3n fetal? Respuesta 1 descartada. Nos quedamos con la posibilidad de no dar ning\u00fan tratamiento por ser de bajo riesgo o administrar aspirina a bajas dosis. Es cierto que el factor V Leiden en heterocigosis se cataloga como de bajo riesgo, que no ha presentado episodio tromb\u00f3tico previo ni hay combinaci\u00f3n con otra trombofilia; si miramos la gu\u00eda CHEST, VIII edici\u00f3n y repasam0s lo que se dijo en el \u00faltimo congreso de la Sociedad Espa\u00f1ola de Ginecolog\u00eda y Obstetricia, vemos que la aspirina se emplea en caso de s\u00edndrome fosfol\u00edpido y que en el caso de esta mujer, sin trombosis previa, sin abortos previos (se necesitan al menos dos) y adem\u00e1s siendo heterocigota, no es necesario el tratamiento. Respuesta correcta, la 3.", "type": "HEMATOLOG\u00cdA", "options": {"1": "Dado que la gestaci\u00f3n es un estado protromb\u00f3tico, existir\u00eda un alto riesgo de tromboembolia venosa, por lo que se debe desaconsejar el embarazo.", "2": "Se debe realizar tratamiento con heparina de bajo peso molecular a dosis profil\u00e1cticas en el puerperio inmediato, siendo opcional realizar igualmente seguimiento durante el embarazo.", "3": "El factor V Leiden en heterocigosis es una trombofilia de bajo riesgo y no hay necesidad de ning\u00fan tratamiento en el embarazo y puerperio.", "4": "Se debe aconsejar aspirina a bajas dosis durante embarazo y puerperio.", "5": "Se debe realizar tratamiento con f\u00e1rmacos antivitamina K (acenocumarol) durante el embarazo."}, "correct_option": 3, "explanations": {"1": {"exist": true, "char_ranges": [[426, 564]], "word_ranges": [[70, 91]], "text": "\u00bfAconsejar a una mujer que no se quede embarazada habiendo tratamientos que aseguren un baj\u00edsimo riesgo de trombosis y/o afectaci\u00f3n fetal?"}, "2": {"exist": true, "char_ranges": [[278, 392]], "word_ranges": [[46, 65]], "text": "si se pone tratamiento, debe ser durante todo el embarazo, el status protromb\u00f3tico no es s\u00f3lo durante el puerperio,"}, "3": {"exist": true, "char_ranges": [[1025, 1252]], "word_ranges": [[168, 207]], "text": "vemos que la aspirina se emplea en caso de s\u00edndrome fosfol\u00edpido y que en el caso de esta mujer, sin trombosis previa, sin abortos previos (se necesitan al menos dos) y adem\u00e1s siendo heterocigota, no es necesario el tratamiento."}, "4": {"exist": true, "char_ranges": [[1025, 1252]], "word_ranges": [[168, 207]], "text": "vemos que la aspirina se emplea en caso de s\u00edndrome fosfol\u00edpido y que en el caso de esta mujer, sin trombosis previa, sin abortos previos (se necesitan al menos dos) y adem\u00e1s siendo heterocigota, no es necesario el tratamiento."}, "5": {"exist": true, "char_ranges": [[60, 149]], "word_ranges": [[11, 24]], "text": "NO se debe administrar acenocumarol a una mujer embarazada por riesgo de teratogenicidad."}}} +{"id": 473, "year": 2020, "question_id_specific": 141, "full_question": "El diagn\u00f3stico m\u00e1s probable de un paciente de 74 a\u00f1os que desde hace dos meses comienza con dolor lumbar irradiado a miembros inferiores, claudicaci\u00f3n neur\u00f3gena y limitaci\u00f3n a la extensi\u00f3n el tronco es:", "full_answer": "Nos est\u00e1n mostrando un cuadro de estenosis de canal lumbar (Respuesta 4 correcta). El paciente muestra el cuadro cl\u00ednico caracter\u00edstico: Dolor lumbar que se alivia al flexionar el tronco hacia delante, dolor radicular con afectaci\u00f3n de miembros inferiores y claudicaci\u00f3n neur\u00f3gena. Una lumbalgia en un paciente mayor de 60 a\u00f1os acompa\u00f1ada de debilidad en piernas en la marcha, que tiene que interrumpirla a una determinada distancia, llamada claudicaci\u00f3n nerviosa, es el cuadro t\u00edpico de la estenosis. El paciente se alivia flexionando la columna hacia delante porque ampl\u00eda el canal lumbar y empeora con la extensi\u00f3n\u2026", "type": "CIRUG\u00cdA ORTOP\u00c9DICA Y TRAUMATOLOG\u00cdA", "options": {"1": "Hernia discal de L4-L5.", "2": "Fractura vertebral lumbar.", "3": "Inestabilidad vertebral L5-S1.", "4": "Estenosis de canal lumbar.", "5": null}, "correct_option": 4, "explanations": {"1": {"exist": false, "char_ranges": [], "word_ranges": [], "text": ""}, "2": {"exist": false, "char_ranges": [], "word_ranges": [], "text": ""}, "3": {"exist": false, "char_ranges": [], "word_ranges": [], "text": ""}, "4": {"exist": true, "char_ranges": [[0, 281]], "word_ranges": [[0, 41]], "text": "Nos est\u00e1n mostrando un cuadro de estenosis de canal lumbar (Respuesta 4 correcta). El paciente muestra el cuadro cl\u00ednico caracter\u00edstico: Dolor lumbar que se alivia al flexionar el tronco hacia delante, dolor radicular con afectaci\u00f3n de miembros inferiores y claudicaci\u00f3n neur\u00f3gena."}, "5": {"exist": false, "char_ranges": [], "word_ranges": [], "text": ""}}} +{"id": 34, "year": 2011, "question_id_specific": 66, "full_question": "Mujer de 51 a\u00f1os que acude a urgencias por disminuci\u00f3n brusca de agudeza visual, cefalea intensa, nauseas y v\u00f3mitos. Hipotensa y afebril. Presenta oftalmoparesia derecha por afectaci\u00f3n de III par craneal. Una TC craneal evidencia una masa en la regi\u00f3n selar hiperdensa con erosi\u00f3n de las ap\u00f3fisis clinoides anteriores. \u00bfCu\u00e1l es la actitud de seguir?", "full_answer": "La apoplej\u00eda hipofisaria es un s\u00edndrome caracterizado por la aparici\u00f3n brusca de cefalea acompa\u00f1ada de alteraciones de la motilidad ocular y un grado variable de insuficiencia hipofisaria. La causa suele ser un r\u00e1pido crecimiento de una masa hipofisaria secundaria a un evento vascular (infarto o hemorragia). Los s\u00edntomas generales pueden incluir desde v\u00f3mitos o n\u00e1useas a irritaci\u00f3n men\u00edngea. La compresi\u00f3n sobre el quiasma y el nervio \u00f3ptico puede causar diferentes d\u00e9ficits campim\u00e9tricos (generalmente hemianopsia bitemporal mayor en los cuadrantes superiores) y disminuci\u00f3n de agudeza visual, incluso ceguera. Si la masa crece hacia los laterales de la silla turca puede comprimir los nervios oculomotores, siendo el nervio ocular motor com\u00fan el m\u00e1s frecuentemente afectado, por su trayecto m\u00e1s medial en el seno cavernoso. La prueba de elecci\u00f3n para el diagn\u00f3stico de la apoplej\u00eda hipofisaria es la RMN, ya que el TC puede no distinguir la regi\u00f3n con la suficiente claridad como para distinguir cambios degeretativos o qu\u00edsticos de sangrados previos. El hallazgo en el TC de cr\u00e1neo de erosi\u00f3n de las ap\u00f3fisis clinoides anteriores puede ir a favor de una lesi\u00f3n cr\u00f3nica, que a ra\u00edz de un fen\u00f3meno vascular agudo ha provocado la sintomatolog\u00eda. El manejo de estos pacientes pasa por un control estricto de las alteraciones hidroelectrol\u00edticas y correcci\u00f3n de los d\u00e9ficits hormonales. En situaciones inestables como en el caso que se describe, con importante alteraci\u00f3n visual, la intervenci\u00f3n quir\u00fargica suele ser de elecci\u00f3n, con descompresi\u00f3n transesfenoidal.", "type": "NEUROLOG\u00cdA Y NEUROCIRUG\u00cdA", "options": {"1": "Sospechar\u00eda meningitis qu\u00edmica derivada de la rotura de un tumor epidermoide e iniciar\u00eda de inmediato tratamiento con corticoesteroides.", "2": "Indicar\u00eda la realizaci\u00f3n de una angiograf\u00eda cerebral para descartar un aneurisma, ya que lo m\u00e1s probable es que estemos ante un caso de hemorragia subaracnoidea y la masa que se evidencia en la TC sea un aneurisma trombosado paraselar.", "3": "Ingreso en UCI y tratamiento del shock que padece la paciente y una vez estabilizada practicar resonancia cerebral para cirug\u00eda programada.", "4": "Determinaci\u00f3n de bioqu\u00edmica y hemograma urgentes, inicio de terapia con corticoesteroides a dosis elevadas y cirug\u00eda transesfenoidal urgente.", "5": "Punci\u00f3n lumbar para descartar meningitis bacteriana previo inicio de antibioterapia emp\u00edrica. Una vez estabilizada la paciente estudio de la masa selar."}, "correct_option": 5, "explanations": {"1": {"exist": false, "char_ranges": [], "word_ranges": [], "text": ""}, "2": {"exist": false, "char_ranges": [], "word_ranges": [], "text": ""}, "3": {"exist": false, "char_ranges": [], "word_ranges": [], "text": ""}, "4": {"exist": false, "char_ranges": [], "word_ranges": [], "text": ""}, "5": {"exist": true, "char_ranges": [[1388, 1565]], "word_ranges": [[213, 237]], "text": "En situaciones inestables como en el caso que se describe, con importante alteraci\u00f3n visual, la intervenci\u00f3n quir\u00fargica suele ser de elecci\u00f3n, con descompresi\u00f3n transesfenoidal."}}} +{"id": 343, "year": 2016, "question_id_specific": 158, "full_question": "Una mujer de 24 a\u00f1os, primigesta, sufre un aborto espont\u00e1neo a las 7 semanas de gestaci\u00f3n. El estudio anatomopatol\u00f3gico de los restos abortivos indica enfermedad molar. Le debemos informar que:", "full_answer": "La respuesta es la 2. Seg\u00fan la SEGO (Sociedad Espa\u00f1ola de Ginecolog\u00eda y Obstetricia) en su protocolo de 2005 \u00abEnfermedad trofobl\u00e1stica gestacional\u00bb, el riesgo de malignizaci\u00f3n, es decir, de desarrollar una neoplasia trofobl\u00e1stica gestacional va de un 5 a un 20% seg\u00fan sea una mola parcial \u00f3 una mola completa, respectivamente. Seg\u00fan este protocolo: \u00abLas pacientes ser\u00e1n controladas semanalmente con la dosificaci\u00f3n de la hcg hasta que se convierta indetectable, por tres veces consecutivas. Despu\u00e9s el control ser\u00e1 mensual, durante seis meses y luego cada dos meses otros seis meses m\u00e1s. Bas\u00e1ndose en los datos disponibles, se recomienda un seguimiento de tres a seis meses en el caso de la mola parcial y de 12 meses en el caso de la mola completa.\u00bb Por ello, los controles posteriores son estrictamente necesarios.El riesgo de una nueva gestaci\u00f3n molar, aunque aumentada respecto a la poblaci\u00f3n normal, no es de un 50%.", "type": "GINECOLOG\u00cdA Y OBSTETRICIA", "options": {"1": "El riesgo de una nueva gestaci\u00f3n molar en un futuro embarazo es del 50%.", "2": "No debe quedar embarazada hasta realizar controles peri\u00f3dicos y haber pasado un a\u00f1os con niveles de BHCG negativos.", "3": "No es necesario realizar controles posteriores si la evacuaci\u00f3n del tejido trofobl\u00e1stico fue completa.", "4": "Es necesario que se realice controles peri\u00f3dicos ya que en el 40% de los casos desarrollar\u00e1 una neoplasia trofobl\u00e1stica gestacional.", "5": null}, "correct_option": 2, "explanations": {"1": {"exist": false, "char_ranges": [], "word_ranges": [], "text": ""}, "2": {"exist": true, "char_ranges": [[149, 587]], "word_ranges": [[22, 91]], "text": "el riesgo de malignizaci\u00f3n, es decir, de desarrollar una neoplasia trofobl\u00e1stica gestacional va de un 5 a un 20% seg\u00fan sea una mola parcial \u00f3 una mola completa, respectivamente. Seg\u00fan este protocolo: \u00abLas pacientes ser\u00e1n controladas semanalmente con la dosificaci\u00f3n de la hcg hasta que se convierta indetectable, por tres veces consecutivas. Despu\u00e9s el control ser\u00e1 mensual, durante seis meses y luego cada dos meses otros seis meses m\u00e1s."}, "3": {"exist": false, "char_ranges": [], "word_ranges": [], "text": ""}, "4": {"exist": false, "char_ranges": [], "word_ranges": [], "text": ""}, "5": {"exist": false, "char_ranges": [], "word_ranges": [], "text": ""}}} +{"id": 210, "year": 2014, "question_id_specific": 187, "full_question": "Una mujer de 27 a\u00f1os, embarazada de 10 semanas y con asma al\u00e9rgico grave persistente. En la actualidad est\u00e1 adecuadamente controlada con budesonida inhalada diaria y salbutamol inhalado a demanda de rescate. Acude a su consulta preocupada por los posibles efectos teratog\u00e9nicos de su medicaci\u00f3n antiasm\u00e1tica. \u00bfCu\u00e1l de las siguientes ser\u00eda la actitud corecta?", "full_answer": "\u2022 Budesonida inh: Evaluar beneficio/riesgo. Los resultados de un extenso estudio epidemiol\u00f3gico prospectivo y de una amplia experiencia post-autorizaci\u00f3n indican que la budesonida inhalada durante el embarazo no tiene efectos adversos en la salud del feto y del reci\u00e9n nacido. Categoria B. \u2022 Salbutamol inh: Usar s\u00f3lo si beneficio para la madre supera los riesgos para el feto. Categoria C.", "type": "GINECOLOG\u00cdA Y OBSTETRICIA", "options": {"1": "Dado que el asma mejora durante el embarazo en la mayor\u00eda de pacientes lo mejor para la paciente y el feto es suspender el tratamiento antiasm\u00e1tico.", "2": "Suspender la budesonida por haberse relacionado con un riesgo aumentado de malformaciones fetales y remplazarla por un antileucotrieno oral (montelukast).", "3": "Retirar el tratamiento actual y reemplazarlo por prednisona oral a la menor dosis posible.", "4": "Mantener el tratamiento acual y tranquilizar a la paciente acerca de sus efecos secundarios y de la necesidad de un adecuado control del asma durante la gestaci\u00f3n.", "5": "Reemplazar la budesonida por un anticuerpo monoclonal anti-IgE (omalizumab) por su mayor seguridad en el embarazo al no ser un f\u00e1rmaco."}, "correct_option": 4, "explanations": {"1": {"exist": false, "char_ranges": [], "word_ranges": [], "text": ""}, "2": {"exist": false, "char_ranges": [], "word_ranges": [], "text": ""}, "3": {"exist": false, "char_ranges": [], "word_ranges": [], "text": ""}, "4": {"exist": true, "char_ranges": [[44, 276]], "word_ranges": [[5, 40]], "text": "Los resultados de un extenso estudio epidemiol\u00f3gico prospectivo y de una amplia experiencia post-autorizaci\u00f3n indican que la budesonida inhalada durante el embarazo no tiene efectos adversos en la salud del feto y del reci\u00e9n nacido."}, "5": {"exist": false, "char_ranges": [], "word_ranges": [], "text": ""}}} +{"id": 187, "year": 2013, "question_id_specific": 68, "full_question": "Mujer de 52 a\u00f1os, natural de un pueblo de la Costa Brava que al ponerse crema en el cuello nota un bulto en la zona anterior, por lo dem\u00e1s asintom\u00e1tico; acude a su m\u00e9dico de cabecera que confirma la presencia de una masa firme de 2 cm. de di\u00e1metro m\u00e1ximo, liso, que asciende con la degluci\u00f3n. No adenopat\u00edas palpables. \u00bfQu\u00e9 pruebas solicitar\u00eda de entrada?", "full_answer": "Hace referencia a un n\u00f3dulo tiroideo, y la prueba indicada es una PAAF (punci\u00f3n con aguja fina). He le\u00eddo la respuesta que ha escrito Emilio sobre la impugnaci\u00f3n; la pregunta est\u00e1 en el bloque de endocrino y parece que intenta dejar claro que se trata de un n\u00f3dulo. La Ecograf\u00eda facilita la PAAF pero no es obligatoria, un n\u00f3dulo tiroideo de 2 cm puede punzarse sin eco. Es importante tener una anal\u00edtica de funci\u00f3n tiroidea previo a la punci\u00f3n; pero como habla de una paciente asintom\u00e1tica, parece claro que no est\u00e1 hipertiroidea. No creo se impugne y creo que es una pregunta que la mayor\u00eda habr\u00e1 respondido bien.", "type": "ENDOCRINOLOG\u00cdA", "options": {"1": "Una determinaci\u00f3n de tiroglobulina en sangre.", "2": "Una TC cervical.", "3": "Una determinaci\u00f3n de anticuerpos antitiroideos (antitiroglubulina y antiperoxidasa) circulante.", "4": "Una punci\u00f3n con aguja fina.", "5": "Una determinaci\u00f3n de T3 libre."}, "correct_option": 4, "explanations": {"1": {"exist": false, "char_ranges": [], "word_ranges": [], "text": ""}, "2": {"exist": false, "char_ranges": [], "word_ranges": [], "text": ""}, "3": {"exist": false, "char_ranges": [], "word_ranges": [], "text": ""}, "4": {"exist": true, "char_ranges": [[0, 96]], "word_ranges": [[0, 17]], "text": "Hace referencia a un n\u00f3dulo tiroideo, y la prueba indicada es una PAAF (punci\u00f3n con aguja fina)."}, "5": {"exist": false, "char_ranges": [], "word_ranges": [], "text": ""}}} +{"id": 406, "year": 2016, "question_id_specific": 33, "full_question": "Mujer de 67 a\u00f1os diagnosticada de un carcinoma ductal infiltrante de mama y sin historia familiar de neoplasia. \u00bfQu\u00e9 estudios adicionales deben realizarse en el tumor por sus implicaciones cl\u00ednico-terap\u00e9uticas?:", "full_answer": "Actualmente la clasificaci\u00f3n molecular del c\u00e1ncer de mama se basa en el estudio de los receptores hormonales, el HER2 y el \u00edndice de proliferaci\u00f3n celular del tumor (Ki67). Tenemos cuatro tipos fundamentales: Luminal (A: RRHH+, HER2- y Ki67 bajo y B: RRHH+, HER2+/ \u2013 y Ki67 alto), HER2 + (RRHH -, HER2 + y Ki67 alto) y Basal Like \u00f3 triple negativo (RRHH -, HER2 \u2013 y Ki67 alto).", "type": "ONCOLOG\u00cdA (ECT\u00d3PICO)", "options": {"1": "Estudio fenot\u00edpico completo mediante citometr\u00eda de flujo.", "2": "Estudio de receptores hormonales y de HER2.", "3": "Estudio de receptores hormonales, ecadherina y estudio de familiares de primer grado.", "4": "Estudio de BRCA l-2 y estudio de familiares de primer grado.", "5": null}, "correct_option": 2, "explanations": {"1": {"exist": false, "char_ranges": [], "word_ranges": [], "text": ""}, "2": {"exist": true, "char_ranges": [[0, 172]], "word_ranges": [[0, 28]], "text": "Actualmente la clasificaci\u00f3n molecular del c\u00e1ncer de mama se basa en el estudio de los receptores hormonales, el HER2 y el \u00edndice de proliferaci\u00f3n celular del tumor (Ki67)."}, "3": {"exist": false, "char_ranges": [], "word_ranges": [], "text": ""}, "4": {"exist": false, "char_ranges": [], "word_ranges": [], "text": ""}, "5": {"exist": false, "char_ranges": [], "word_ranges": [], "text": ""}}} +{"id": 526, "year": 2021, "question_id_specific": 54, "full_question": "Mujer de 86 a\u00f1os de edad en quien se ha detectado una fibrilaci\u00f3n auricular no valvular. Tiene una puntuaci\u00f3n CHADS2 de 3 puntos. En la bibliograf\u00eda, pacientes similares en tratamiento con warfarina tienen un riesgo de ictus del 2,2 % frente al 5,2 % en los pacientes sin warfarina. \u00bfCu\u00e1l ser\u00eda el n\u00famero necesario a tratar (NNT) para prevenir un ictus embol\u00edgeno con el tratamiento anticoagulante?:", "full_answer": "El NNT (n\u00famero necesario a tratar), significa cu\u00e1ntos pacientes necesitar\u00edamos tratar con una intervenci\u00f3n para obtener un beneficio; en esta pregunta cu\u00e1ntos pacientes necesitamos tratar con warfarina para prevenir un ictus. Para calcularlo empleamos la siguiente f\u00f3rmula: NNT= 1/RRA (RRA: reducci\u00f3n del riesgo absoluto). RRA= Io \u2013 Ie (Io:Incidencia en no expuestos a warfarina; Ie: incidencia en expuestos a warfarina). RRA: 5,2%- 2,2%= 3= 0,03. NNT: 1/0,03= 33,3.", "type": "MEDICINA PREVENTIVA", "options": {"1": "3.", "2": "19,2.", "3": "33,3.", "4": "49,5.", "5": null}, "correct_option": 3, "explanations": {"1": {"exist": false, "char_ranges": [], "word_ranges": [], "text": ""}, "2": {"exist": false, "char_ranges": [], "word_ranges": [], "text": ""}, "3": {"exist": true, "char_ranges": [[0, 466]], "word_ranges": [[0, 68]], "text": "El NNT (n\u00famero necesario a tratar), significa cu\u00e1ntos pacientes necesitar\u00edamos tratar con una intervenci\u00f3n para obtener un beneficio; en esta pregunta cu\u00e1ntos pacientes necesitamos tratar con warfarina para prevenir un ictus. Para calcularlo empleamos la siguiente f\u00f3rmula: NNT= 1/RRA (RRA: reducci\u00f3n del riesgo absoluto). RRA= Io \u2013 Ie (Io:Incidencia en no expuestos a warfarina; Ie: incidencia en expuestos a warfarina). RRA: 5,2%- 2,2%= 3= 0,03. NNT: 1/0,03= 33,3."}, "4": {"exist": false, "char_ranges": [], "word_ranges": [], "text": ""}, "5": {"exist": false, "char_ranges": [], "word_ranges": [], "text": ""}}} +{"id": 6, "year": 2011, "question_id_specific": 45, "full_question": "Hombre de 30 a\u00f1os con antecedente familiar de padre fallecido a los 38 a\u00f1os por c\u00e1ncer de colon. Se practica una colonoscopia que muestra cientos de adenomas a lo largo de todo el colon. \u00bfCu\u00e1l de las afirmaciones son falsas?", "full_answer": "Paciente con Poliposis adenomatosa familiar. Todas las afirmaciones son correctas excepto la 2. El tratamiento debe ser quir\u00fargico cuando se observa la poliposis. A los 40 a\u00f1os es el punto habitual de incidencia de c\u00e1ncer.", "type": "DIGESTIVO", "options": {"1": "El paciente padece una poliposis adenomatosa familiar.", "2": "El manejo m\u00e1s apropiado es sulindac colonoscopia de seguimiento anuales y colectom\u00eda al cumplir los 40.", "3": "De no someterse a tratamiento quir\u00fargico casi con toda seguridad el paciente desarrollar\u00e1 c\u00e1ncer colorrectal.", "4": "Deben estudiarse familiares de primer grado.", "5": "Los hijos del paciente tienen un 50% de riesgo de padecer la misma enfermedad."}, "correct_option": 2, "explanations": {"1": {"exist": true, "char_ranges": [[0, 222]], "word_ranges": [[0, 35]], "text": "Paciente con Poliposis adenomatosa familiar. Todas las afirmaciones son correctas excepto la 2. El tratamiento debe ser quir\u00fargico cuando se observa la poliposis. A los 40 a\u00f1os es el punto habitual de incidencia de c\u00e1ncer."}, "2": {"exist": true, "char_ranges": [[0, 222]], "word_ranges": [[0, 35]], "text": "Paciente con Poliposis adenomatosa familiar. Todas las afirmaciones son correctas excepto la 2. El tratamiento debe ser quir\u00fargico cuando se observa la poliposis. A los 40 a\u00f1os es el punto habitual de incidencia de c\u00e1ncer."}, "3": {"exist": true, "char_ranges": [[0, 222]], "word_ranges": [[0, 35]], "text": "Paciente con Poliposis adenomatosa familiar. Todas las afirmaciones son correctas excepto la 2. El tratamiento debe ser quir\u00fargico cuando se observa la poliposis. A los 40 a\u00f1os es el punto habitual de incidencia de c\u00e1ncer."}, "4": {"exist": true, "char_ranges": [[0, 222]], "word_ranges": [[0, 35]], "text": "Paciente con Poliposis adenomatosa familiar. Todas las afirmaciones son correctas excepto la 2. El tratamiento debe ser quir\u00fargico cuando se observa la poliposis. A los 40 a\u00f1os es el punto habitual de incidencia de c\u00e1ncer."}, "5": {"exist": true, "char_ranges": [[0, 222]], "word_ranges": [[0, 35]], "text": "Paciente con Poliposis adenomatosa familiar. Todas las afirmaciones son correctas excepto la 2. El tratamiento debe ser quir\u00fargico cuando se observa la poliposis. A los 40 a\u00f1os es el punto habitual de incidencia de c\u00e1ncer."}}} +{"id": 573, "year": 2022, "question_id_specific": 92, "full_question": "Mujer de 75 a\u00f1os tra\u00edda a consulta por su familia porque le encuentran desde hace meses deprimida y con fallos de memoria. Les preocupa que pueda tener una enfermedad de Alzheimer. La paciente refiere que no cree que le pase nada y que est\u00e1 como siempre. \u00bfEn qu\u00e9 orden se presentan generalmente los siguientes s\u00edntomas en la progresi\u00f3n de la enfermedad de Alzheimer?:", "full_answer": "Esta pregunta la contesto pensando m\u00e1s en lo que quieren preguntar que en lo que realmente preguntan. Existe heterogenicidad en el desarrollo de los s\u00edntomas de la enfermedad de Alzheimer. Existen variantes conductuales, pero en el MIR suelen querer que pens\u00e9is en otro tipo de demencias ante s\u00edntomas conductuales. De hecho, la aparici\u00f3n de estos en fases precoces nos ha de poner en alerta y abrir el abanico de posibilidades diagn\u00f3sticas. Por ello descartamos la 2 y la 4. Entre la 1 y la 3 elijo la 3 por el mismo motivo y por el hincapi\u00e9 al declive de la independencia funcional que se produce en estos pacientes.", "type": "NEUROLOG\u00cdA", "options": {"1": "Cambios de humor, s\u00edntomas conductuales, d\u00e9ficit cognitivo.", "2": "S\u00edntomas conductuales, s\u00edntomas motores, declive de la independencia funcional.", "3": "Cambios de humor, d\u00e9ficit cognitivo, declive de la independencia funcional.", "4": "S\u00edntomas conductuales, cambios de humor, s\u00edntomas motores.", "5": null}, "correct_option": 3, "explanations": {"1": {"exist": true, "char_ranges": [[476, 618]], "word_ranges": [[79, 108]], "text": "Entre la 1 y la 3 elijo la 3 por el mismo motivo y por el hincapi\u00e9 al declive de la independencia funcional que se produce en estos pacientes."}, "2": {"exist": true, "char_ranges": [[189, 475]], "word_ranges": [[30, 79]], "text": "Existen variantes conductuales, pero en el MIR suelen querer que pens\u00e9is en otro tipo de demencias ante s\u00edntomas conductuales. De hecho, la aparici\u00f3n de estos en fases precoces nos ha de poner en alerta y abrir el abanico de posibilidades diagn\u00f3sticas. Por ello descartamos la 2 y la 4."}, "3": {"exist": true, "char_ranges": [[476, 618]], "word_ranges": [[79, 108]], "text": "Entre la 1 y la 3 elijo la 3 por el mismo motivo y por el hincapi\u00e9 al declive de la independencia funcional que se produce en estos pacientes."}, "4": {"exist": true, "char_ranges": [[189, 475]], "word_ranges": [[30, 79]], "text": "Existen variantes conductuales, pero en el MIR suelen querer que pens\u00e9is en otro tipo de demencias ante s\u00edntomas conductuales. De hecho, la aparici\u00f3n de estos en fases precoces nos ha de poner en alerta y abrir el abanico de posibilidades diagn\u00f3sticas. Por ello descartamos la 2 y la 4."}, "5": {"exist": false, "char_ranges": [], "word_ranges": [], "text": ""}}} +{"id": 164, "year": 2013, "question_id_specific": 123, "full_question": "Un paciente de 80 a\u00f1os con historia de hipertensi\u00f3n y en tratamiento con enalapril y espironolactona acude al hospital por astenia y debilidad muscular severa. La presi\u00f3n arterial es de 110/70 mmHg. En el ECG destacan ondas T picudas y elevadas, extras\u00edstoles ventriculares y QT corto. \u00bfCu\u00e1l es el diagn\u00f3stico m\u00e1s probable?", "full_answer": "La respuesta correcta es: 2. Hiperpotasemia. Les ha dado por el potasio en este examen\u2026 La espironolactona, como diur\u00e9tico ahorrador de potasio, inhibe la acci\u00f3n de la aldosterona a nivel renal, siendo responsable de una disminuci\u00f3n de la excreci\u00f3n de potasio. La hiperpotasemia produce trastornos de la repolarizaci\u00f3n que se traducen en la instalaci\u00f3n de una T grande, sim\u00e9trica, de base estrecha, visible en las derivaciones precordiales. El espacio QT acortado. Estas anomal\u00edas aparecen con una kaliemia de alrededor de 5,5 a 6 mmol/l. A partir de 6,5 mmol/l, las modificaciones electrocardiogr\u00e1ficas son constantes, y est\u00e1n dominadas por los trastornos de la conducci\u00f3n (haciendo posibles las extras\u00edstoles ventriculares).", "type": "NEFROLOG\u00cdA", "options": {"1": "Hipercalcemia.", "2": "Hiperpotasemia.", "3": "Hipomagnesemia.", "4": "Hipocalcemia.", "5": "Hipernatremia."}, "correct_option": 2, "explanations": {"1": {"exist": false, "char_ranges": [], "word_ranges": [], "text": ""}, "2": {"exist": true, "char_ranges": [[261, 440]], "word_ranges": [[41, 67]], "text": "La hiperpotasemia produce trastornos de la repolarizaci\u00f3n que se traducen en la instalaci\u00f3n de una T grande, sim\u00e9trica, de base estrecha, visible en las derivaciones precordiales."}, "3": {"exist": false, "char_ranges": [], "word_ranges": [], "text": ""}, "4": {"exist": false, "char_ranges": [], "word_ranges": [], "text": ""}, "5": {"exist": false, "char_ranges": [], "word_ranges": [], "text": ""}}} +{"id": 208, "year": 2014, "question_id_specific": 183, "full_question": "Mujer 40 semanas de gestaci\u00f3n en trabajo de parto con 6 cm de dilataci\u00f3n. Presenta patr\u00f3n fetal decelerativo en registro cardiotocogr\u00e1fico por lo que se decide realizar microtoma de sangre fetal para valorar bienestar fetal. Resultado 7.22. La conducta correcta es:", "full_answer": "\u2022 pH menor de 7.20 Valor patol\u00f3gico. Indicaci\u00f3n de extracci\u00f3n fetal por la v\u00eda m\u00e1s r\u00e1pida, en este caso una ces\u00e1rea. \u2022 pH entre 7.20-7.25 Valor prepatol\u00f3gico. Repetir microtoma en 15-20 min. \u2022 pH mayor de 7.25 . Valor dentro de los l\u00edmites de la normalidad. Observaci\u00f3n.", "type": "GINECOLOG\u00cdA Y OBSTETRICIA", "options": {"1": "Acidosis grave. Ces\u00e1rea urgente.", "2": "Valor prepatol\u00f3gico repetir microtoma 15-20 minutos.", "3": "Acidosis moderada. Repetir microtoma en 1-2 horas.", "4": "Valor en limites normales, dejar evoluci\u00f3n natural de parto.", "5": "Repetir en el momento posible error en la obtenci\u00f3n de la toma."}, "correct_option": 2, "explanations": {"1": {"exist": false, "char_ranges": [], "word_ranges": [], "text": ""}, "2": {"exist": true, "char_ranges": [[119, 190]], "word_ranges": [[22, 32]], "text": "pH entre 7.20-7.25 Valor prepatol\u00f3gico. Repetir microtoma en 15-20 min."}, "3": {"exist": false, "char_ranges": [], "word_ranges": [], "text": ""}, "4": {"exist": false, "char_ranges": [], "word_ranges": [], "text": ""}, "5": {"exist": false, "char_ranges": [], "word_ranges": [], "text": ""}}} +{"id": 232, "year": 2014, "question_id_specific": 103, "full_question": "Muchacha de 19 a\u00f1os. sin antecedentes m\u00e9dicos de inter\u00e9s, salvo un cuadro gripal autolimitado 3 semanas antes, que acude al servido de urgencias por petequias y equimosis de aparici\u00f3n espont\u00e1nea. En la exploraci\u00f3n f\u00edsica la paciente se encuentra con buen estado general, afebril, normotensa y orientada en tiempo y espacio. Se observan petequias diseminadas por EEII y abdomen y equimosis peque\u00f1as en zonas de dec\u00fabito. No se palpan adenopat\u00edas ni esplenomegalia. La anal\u00edtica realizada ofrece los siguientes hallazgos: Hb 12.6 g/dL, leucocitos 5.500/mm3, plaquetas 7000/mm3. El estudio del frotis de sangre perif\u00e9rica ofrece una morfolog\u00eda eritrocitaria normal, recuento leucocitario diferencial normal y el recuento planetario es concordante con la cifra del autoanalizador sin observarse agregados plaquetares. Bioqu\u00edmica y proteinograma, beta 2 microglobulina y LDH normal. \u00bfCu\u00e1l cree que es, de los siguientes, el tratamiento inicial m\u00e1s adecuado?", "full_answer": "P\u00farpura trombocitop\u00e9nica idiop\u00e1tica, pero con muy bajas plaquetas, por lo que hay que reponer. La dosis de esteroide propuesta es demasiado baja.", "type": "HEMATOLOG\u00cdA", "options": {"1": "Transfusi\u00f3n de plaquetas.", "2": "Rituximab semanal.", "3": "Ciclofosfamida en pulsos de 4 d\u00edas cada 21 d\u00edas.", "4": "Plasmaf\u00e9resis diaria.", "5": "Prednisona a 1 mg/d\u00eda durante 2-3 semanas."}, "correct_option": 1, "explanations": {"1": {"exist": true, "char_ranges": [[0, 94]], "word_ranges": [[0, 14]], "text": "P\u00farpura trombocitop\u00e9nica idiop\u00e1tica, pero con muy bajas plaquetas, por lo que hay que reponer."}, "2": {"exist": false, "char_ranges": [], "word_ranges": [], "text": ""}, "3": {"exist": false, "char_ranges": [], "word_ranges": [], "text": ""}, "4": {"exist": false, "char_ranges": [], "word_ranges": [], "text": ""}, "5": {"exist": true, "char_ranges": [[95, 145]], "word_ranges": [[14, 22]], "text": "La dosis de esteroide propuesta es demasiado baja."}}} +{"id": 64, "year": 2011, "question_id_specific": 210, "full_question": "Una mujer de 42 a\u00f1os con c\u00e1ncer de mama presenta met\u00e1stasis a nivel de los agujeros intervertebrales entre la 4\u00aa y 5\u00aa v\u00e9rtebras cervicales y entre la 4\u00aa y 5\u00aa v\u00e9rtebras tor\u00e1cicas. \u00bfQu\u00e9 nervios espinales estar\u00e1n lesionados?", "full_answer": "El truco de esta pregunta es que hay 8 ra\u00edces cervicales para 7 v\u00e9rtebras, por tanto la ra\u00edz C1 sale POR ENCIMA del atlas, y as\u00ed hasta C8 que sale ENTRE C7 y T1. Las ra\u00edces tor\u00e1cicas salen respectivamente por debajo de la v\u00e9rtebra con su misma numeraci\u00f3n (nervio T1 bajo T1, T2 bajo T2, etc.). Por tanto, entre C4 y C5 sale el 5\u00ba nervio cervical, y entre T4 y T5 el 4\u00ba tor\u00e1cico. La respuesta correcta es la 4.", "type": "ANATOM\u00cdA", "options": {"1": "Cuarto nervio cervical y cuarto tor\u00e1cico.", "2": "Quinto nervio cervical y quinto tor\u00e1cico.", "3": "Cuarto nervio cervical y quinto tor\u00e1cico.", "4": "Quinto nervio cervical y cuarto tor\u00e1cico.", "5": "Tercer nervio cervical y cuarto tor\u00e1cico."}, "correct_option": 4, "explanations": {"1": {"exist": false, "char_ranges": [], "word_ranges": [], "text": ""}, "2": {"exist": false, "char_ranges": [], "word_ranges": [], "text": ""}, "3": {"exist": false, "char_ranges": [], "word_ranges": [], "text": ""}, "4": {"exist": false, "char_ranges": [], "word_ranges": [], "text": ""}, "5": {"exist": false, "char_ranges": [], "word_ranges": [], "text": ""}}} +{"id": 280, "year": 2016, "question_id_specific": 32, "full_question": "Mujer de 59 a\u00f1os que presenta diarrea cr\u00f3nica acuosa de 4 meses de evoluci\u00f3n. En la endoscopia, la mucosa no mostraba aspectos relevantes. En concreto, no se observaron \u00falceras o \u00e1reas friables. Se realiz\u00f3 biopsia del colon transverso. En el estudio histopatol\u00f3gico se reconoci\u00f3 un \u00e1rea engrosada por debajo del epitelio de revestimiento superficial, que era m\u00e1s evidente mediante t\u00e9cnica de tricr\u00f3mico de Masson y que conllevaba atrofia y denudaci\u00f3n epitelial. Tambi\u00e9n se apreci\u00f3 un claro incremento en la densidad de linfocitos intraepiteliales. \u00bfEl diagn\u00f3stico de la lesi\u00f3n intestinal es?", "full_answer": "Todo lo que nos cuentan en este caso es t\u00edpico de la COLITIS COL\u00c1GENA. En principio, la cl\u00ednica y la endoscopia pueden dejarnos un poco como al principio, pero la histolog\u00eda es poco m\u00e1s que la definici\u00f3n de esta patolog\u00eda. El criterio espec\u00edfico para el diagn\u00f3stico de la colitis col\u00e1gena es la presencia adicional de una banda irregular de col\u00e1geno justo debajo del epitelio de superficie de la mucosa del colon en continuidad con la membrana basal, visible con la tinci\u00f3n de tricr\u00f3mico (ti\u00f1e las fibras de col\u00e1geno tipo I), que atrapa capilares superficiales produciendo lesiones en el epitelio de superficie. Adem\u00e1s, se acompa\u00f1a de un infiltrado inflamatorio cr\u00f3nico en la l\u00e1mina propia, compuesto fundamentalmente por linfocitos, c\u00e9lulas plasm\u00e1ticas y eosin\u00f3filos.", "type": "ANATOM\u00cdA PATOL\u00d3GICA", "options": {"1": "Colitis ulcerosa cronificada.", "2": "Colitis pseudomembranosa.", "3": "Colitis col\u00e1gena.", "4": "Enfermedad de Crohn fibrosante.", "5": null}, "correct_option": 3, "explanations": {"1": {"exist": false, "char_ranges": [], "word_ranges": [], "text": ""}, "2": {"exist": false, "char_ranges": [], "word_ranges": [], "text": ""}, "3": {"exist": true, "char_ranges": [[223, 611]], "word_ranges": [[40, 101]], "text": "El criterio espec\u00edfico para el diagn\u00f3stico de la colitis col\u00e1gena es la presencia adicional de una banda irregular de col\u00e1geno justo debajo del epitelio de superficie de la mucosa del colon en continuidad con la membrana basal, visible con la tinci\u00f3n de tricr\u00f3mico (ti\u00f1e las fibras de col\u00e1geno tipo I), que atrapa capilares superficiales produciendo lesiones en el epitelio de superficie."}, "4": {"exist": false, "char_ranges": [], "word_ranges": [], "text": ""}, "5": {"exist": false, "char_ranges": [], "word_ranges": [], "text": ""}}} +{"id": 545, "year": 2022, "question_id_specific": 122, "full_question": "Paciente de 78 a\u00f1os sin antecedentes que consulta por astenia y disnea de 3 meses de evoluci\u00f3n, que ha progresado hasta hacerse de reposo en los \u00faltimos d\u00edas. En los d\u00edas previos refiere tambi\u00e9n dolor tor\u00e1cico de caracter\u00edsticas anginosas con peque\u00f1os esfuerzos. En la exploraci\u00f3n destaca presi\u00f3n arterial de 110/80 mmHg, frecuencia card\u00edaca de 85 lpm y auscultaci\u00f3n con un soplo sist\u00f3lico \u00e1spero en segundo espacio intercostal derecho y crepitantes en ambas bases pulmonares. \u00bfCu\u00e1l de los siguientes es el diagn\u00f3stico m\u00e1s probable?:", "full_answer": "Estenosis a\u00f3rtica.", "type": "CARDIOLOG\u00cdA", "options": {"1": "Insuficiencia mitral.", "2": "Estenosis a\u00f3rtica.", "3": "Insuficiencia a\u00f3rtica.", "4": "Miocardiopat\u00eda dilatada de origen isqu\u00e9mico.", "5": null}, "correct_option": 2, "explanations": {"1": {"exist": false, "char_ranges": [], "word_ranges": [], "text": ""}, "2": {"exist": true, "char_ranges": [[0, 18]], "word_ranges": [[0, 2]], "text": "Estenosis a\u00f3rtica."}, "3": {"exist": false, "char_ranges": [], "word_ranges": [], "text": ""}, "4": {"exist": false, "char_ranges": [], "word_ranges": [], "text": ""}, "5": {"exist": false, "char_ranges": [], "word_ranges": [], "text": ""}}} +{"id": 257, "year": 2014, "question_id_specific": 95, "full_question": "En una paciente de 30 a\u00f1os se encuentra una cifra de calcio de 11 mg/dl (normal menos de 10,5 mg/dl) durante un examen de empresas rutinario. La determinaci\u00f3n de PTH fue de 45 pg/ml (VN 10-55 pg/ml). La historia es anodina, salvo por el hecho de que la madre y el abuelo paterno fueron diagnosticados de hiperparatiroidismo e intervenidos, aunque permanecieron hipercalc\u00e9micos. \u00bfQu\u00e9 prueba es m\u00e1s \u00fatil para confirmar el diagn\u00f3stico?", "full_answer": "Cociente calcio/creatinina orina. Se trata de hipercalcemia hipocalci\u00farica familiar o hipercalcemia familiar benigna. Trastorno autos\u00f3mico dominante. Asintom\u00e1tico. No se curan tras tratamiento quir\u00fargico; se debe a mutaci\u00f3n del receptor sensible al calcio. Pregunta dif\u00edcil ya que hay conocer datos de una enfermedad que no se suele preguntar en el MIR.", "type": "ENDOCRINOLOG\u00cdA", "options": {"1": "25-OH D.", "2": "1,25-OH 2D.", "3": "Cociente calcio/creatinina en orina.", "4": "Reabsorci\u00f3n tubular de fosfatos.", "5": "PTHrP."}, "correct_option": 3, "explanations": {"1": {"exist": false, "char_ranges": [], "word_ranges": [], "text": ""}, "2": {"exist": false, "char_ranges": [], "word_ranges": [], "text": ""}, "3": {"exist": true, "char_ranges": [[34, 256]], "word_ranges": [[3, 32]], "text": "Se trata de hipercalcemia hipocalci\u00farica familiar o hipercalcemia familiar benigna. Trastorno autos\u00f3mico dominante. Asintom\u00e1tico. No se curan tras tratamiento quir\u00fargico; se debe a mutaci\u00f3n del receptor sensible al calcio."}, "4": {"exist": false, "char_ranges": [], "word_ranges": [], "text": ""}, "5": {"exist": false, "char_ranges": [], "word_ranges": [], "text": ""}}} +{"id": 548, "year": 2022, "question_id_specific": 126, "full_question": "Var\u00f3n de 58 a\u00f1os con antecedentes de hipertensi\u00f3n arterial de 6 a\u00f1os de evoluci\u00f3n, que consulta por mal control de las cifras de presi\u00f3n arterial a pesar de recibir tratamiento con un inhibidor de la enzima convertidora de angiotensina, un diur\u00e9tico y un calcioantagonista. En consulta presenta cifras de 149/100 mmHg. Anal\u00edtica: creatinina 1,2 mg/dl, potasio 2,2 mEq/l y alcalosis metab\u00f3lica compensada; resto del estudio bioqu\u00edmico, hemograma, coagulaci\u00f3n y sedimento urinario normal. Se\u00f1ale la afirmaci\u00f3n correcta:", "full_answer": "La TC forma parte del estudio diagn\u00f3stico en caso de haber una confirmaci\u00f3n bioqu\u00edmica.", "type": "CARDIOLOG\u00cdA", "options": {"1": "El origen de la hipertensi\u00f3n en este caso es la secreci\u00f3n excesiva de aldosterona causada por una hiperfunci\u00f3n aut\u00f3noma de la m\u00e9dula suprarrenal.", "2": "En la mayor\u00eda de los casos el sustrato anat\u00f3mico es una hiperplasia bilateral de la corteza suprarrenal.", "3": "La TC forma parte del estudio diagn\u00f3stico en caso de haber una confirmaci\u00f3n bioqu\u00edmica.", "4": "La espironolactona est\u00e1 contraindicada en el manejo de esta patolog\u00eda.", "5": null}, "correct_option": 3, "explanations": {"1": {"exist": false, "char_ranges": [], "word_ranges": [], "text": ""}, "2": {"exist": false, "char_ranges": [], "word_ranges": [], "text": ""}, "3": {"exist": true, "char_ranges": [[0, 87]], "word_ranges": [[0, 14]], "text": "La TC forma parte del estudio diagn\u00f3stico en caso de haber una confirmaci\u00f3n bioqu\u00edmica."}, "4": {"exist": false, "char_ranges": [], "word_ranges": [], "text": ""}, "5": {"exist": false, "char_ranges": [], "word_ranges": [], "text": ""}}} +{"id": 318, "year": 2016, "question_id_specific": 140, "full_question": "Usted valora a un paciente de 66 anos con dolor inguinal acentuado con la bipedestaci\u00f3n prolongada algunos d\u00edas al mes. Una radiograf\u00eda simple de caderas muestra estrechamiento del espacio articular femoro-acetabular, esclerosis y ostefitos \u00bfCu\u00e1l es su actitud?", "full_answer": "Es una coxartrosis. El criterios quir\u00fargico depende de la sintomatolog\u00eda del paciente y del fracaso a tratamiento m\u00e9dico. Ninguna medicaci\u00f3n frena ni modifica el curso de la enfermedad. No es necesaria una RMN, no aporta nada. El tratamiento es m\u00e9dico y secuencial empezando con el primer escal\u00f3n analg\u00e9sico de la OMS.", "type": "REUMATOLOG\u00cdA", "options": {"1": "Hago el diagn\u00f3stico de coxartrosis y env\u00edo al traumat\u00f3logo para colocar una pr\u00f3tesis de cadera.", "2": "Inicio tratamiento con opioides d\u00e9biles que han demostrado evidencia en detener la progresi\u00f3n de la enfermedad.", "3": "Instauro tratamiento con paracetamol, explico que la evoluci\u00f3n es muy variable y la indicaci\u00f3n quir\u00fargica depende de la funcionalidad y control del dolor.", "4": "Por las caracter\u00edsticas radiol\u00f3gicas descritas, necesito una RMN de cadera antes de tomar una decisi\u00f3n terap\u00e9utica.", "5": null}, "correct_option": 3, "explanations": {"1": {"exist": false, "char_ranges": [], "word_ranges": [], "text": ""}, "2": {"exist": true, "char_ranges": [[122, 185]], "word_ranges": [[18, 28]], "text": "Ninguna medicaci\u00f3n frena ni modifica el curso de la enfermedad."}, "3": {"exist": true, "char_ranges": [[227, 318]], "word_ranges": [[36, 51]], "text": "El tratamiento es m\u00e9dico y secuencial empezando con el primer escal\u00f3n analg\u00e9sico de la OMS."}, "4": {"exist": true, "char_ranges": [[186, 226]], "word_ranges": [[28, 36]], "text": "No es necesaria una RMN, no aporta nada."}, "5": {"exist": false, "char_ranges": [], "word_ranges": [], "text": ""}}} +{"id": 147, "year": 2012, "question_id_specific": 157, "full_question": "Una estudiante universitaria de 19 a\u00f1os acude a la consulta acompa\u00f1ada por sus padres refierendo sentirse en los \u00faltimos dos meses progresivamente m\u00e1s ast\u00e9nica, con p\u00e9rdida de apetito y de peso y con mayores dificultades para concentrarse en los estudios. En la anamnesis tambi\u00e9n destaca que ha perdido interes en salir con las amigas, presenta ideas de muerte sin ideaci\u00f3n autol\u00edtica y cogniciones pesimistas de futuro. Su peso es el 90% del considerado ideal por edad y g\u00e9nero. No presenta fobia ponderal ni distorsi\u00f3n de la imagen corporal. El diagn\u00f3stico m\u00e1s adecuado es:", "full_answer": "En cuanto o\u00edmos lo de peso vamos hacia la anorexia nerviosa, pero la anorexia nerviosa tiene tres caracter\u00edsticas: P\u00e9rdida significativa de peso originada por el propio enfermo, distorsi\u00f3n de la imagen corporal y amenorrea. As\u00ed que quitamos la 1. Uno pierde peso cuando se pone triste, se preocupa, etc\u2026 La 3 no tiene sentido en el MIR por la cl\u00ednica -en la realidad la historia es otra y en psiquiatr\u00eda todo es evoluci\u00f3n-. La 4 ha de cumplir criterios temporales que aqu\u00ed no se cumplen. As\u00ed que estamos entre la 2 y la 5. \u00bfDepresi\u00f3n o ansiedad? No hay cl\u00ednica de ansiedad y adem\u00e1s cumple citerios de depresi\u00f3n. A la pobre chica le ha caido un diagn\u00f3stico de depresi\u00f3n. La respuesta es la 5 (otra opci\u00f3n es que estuviese estudiando el MIR).", "type": "PSIQUIATR\u00cdA", "options": {"1": "Anorexia nerviosa.", "2": "Trastorno de ansiedad.", "3": "Trastorno l\u00edmite de la personalidad.", "4": "Distimia.", "5": "Trastorno depresivo mayor."}, "correct_option": 5, "explanations": {"1": {"exist": true, "char_ranges": [[115, 246]], "word_ranges": [[18, 39]], "text": "P\u00e9rdida significativa de peso originada por el propio enfermo, distorsi\u00f3n de la imagen corporal y amenorrea. As\u00ed que quitamos la 1."}, "2": {"exist": true, "char_ranges": [[546, 611]], "word_ranges": [[97, 108]], "text": "No hay cl\u00ednica de ansiedad y adem\u00e1s cumple citerios de depresi\u00f3n."}, "3": {"exist": true, "char_ranges": [[304, 423]], "word_ranges": [[49, 73]], "text": "La 3 no tiene sentido en el MIR por la cl\u00ednica -en la realidad la historia es otra y en psiquiatr\u00eda todo es evoluci\u00f3n-."}, "4": {"exist": true, "char_ranges": [[424, 487]], "word_ranges": [[73, 85]], "text": "La 4 ha de cumplir criterios temporales que aqu\u00ed no se cumplen."}, "5": {"exist": true, "char_ranges": [[546, 611]], "word_ranges": [[97, 108]], "text": "No hay cl\u00ednica de ansiedad y adem\u00e1s cumple citerios de depresi\u00f3n."}}} +{"id": 530, "year": 2021, "question_id_specific": 167, "full_question": "Ante un paciente con enfermedad renal cr\u00f3nica avanzada (ERC G4, FGe 20 ml/min) conocida, que acude a urgencias por debilidad general y se le detecta una hiperpotasemia grave (K 7 mEq/l) con alteraciones electrocardiogr\u00e1ficas. \u00bfCu\u00e1l ser\u00eda la primera medida a tomar?:", "full_answer": "Nos describen el caso de un paciente con una emergencia hiperkal\u00e9mica (potasio >6,5 mmol/l con alteraciones en el ECG). El primer paso es la administraci\u00f3n de gluconato c\u00e1lcico iv para contrarrestar la toxicidad card\u00edaca de la hiperpotasemia (opci\u00f3n 3 correcta), d\u00e1ndonos m\u00e1s tiempo para iniciar los tratamientos hipokalemiantes (inicialmente suero glucosado con insulina, salbutamol, furosemida, y valorando di\u00e1lisis si no hay buena respuesta).", "type": "NEFROLOG\u00cdA", "options": {"1": "Administraci\u00f3n de resinas de intercambio cati\u00f3nico.", "2": "Colocaci\u00f3n de un cat\u00e9ter para iniciar di\u00e1lisis.", "3": "Administraci\u00f3n de gluconato c\u00e1lcico endovenoso.", "4": "Administraci\u00f3n de furosemida oral.", "5": null}, "correct_option": 3, "explanations": {"1": {"exist": false, "char_ranges": [], "word_ranges": [], "text": ""}, "2": {"exist": false, "char_ranges": [], "word_ranges": [], "text": ""}, "3": {"exist": true, "char_ranges": [[0, 261]], "word_ranges": [[0, 40]], "text": "Nos describen el caso de un paciente con una emergencia hiperkal\u00e9mica (potasio >6,5 mmol/l con alteraciones en el ECG). El primer paso es la administraci\u00f3n de gluconato c\u00e1lcico iv para contrarrestar la toxicidad card\u00edaca de la hiperpotasemia (opci\u00f3n 3 correcta),"}, "4": {"exist": false, "char_ranges": [], "word_ranges": [], "text": ""}, "5": {"exist": false, "char_ranges": [], "word_ranges": [], "text": ""}}} +{"id": 494, "year": 2020, "question_id_specific": 157, "full_question": "Mujer de 26 a\u00f1os que consulta por sensaci\u00f3n de debilidad generalizada que se ha instaurado progresivamente en el curso de tres semanas, haci\u00e9ndose especialmente intensa en los \u00faltimos dos d\u00edas. Refiere desde hace un par de a\u00f1os episodios de dolor articular en las manos que han precisado de la toma de antiinflamatorios, as\u00ed como la aparici\u00f3n de alguna lesi\u00f3n eritematosa de causa no aclarada en la zona del escote, principalmente en verano. En la exploraci\u00f3n f\u00edsica destaca \u00fanicamente una palidez cut\u00e1neo-mucosa evidente y una frecuencia cardiaca de 100 lpm. En el hemograma destaca: Hb 6 gr/dL, Hto 27 %, VCM 105 fL, 3.420 leucocitos/mm3 (2300 neutr\u00f3filos/mm3, 800 linfocitos/mm3, 250 monocitos/mm3, 50 eosin\u00f3filos/mm3, 20 bas\u00f3filos/mm3), plaquetas 170.000/mm3. En la bioqu\u00edmica: AST 30 UI/L, ALT 35 UI/L, GGT 59 UI/L, fosfatasa alcalina 105 UI/L, LDH 490 UI/L, urea 20 mg/dL, creatinina 0,8 mg/dL. Teniendo en cuenta la informaci\u00f3n disponible, indique cu\u00e1l de los siguientes par\u00e1metros anal\u00edticos adicionales necesitar\u00eda conocer para poder tomar la decisi\u00f3n inmediata m\u00e1s apropiada:", "full_answer": "Comentan el caso de una mujer joven con cl\u00ednica de enfermedad sist\u00e9mica de probable origen autoinmune (lupus) que presenta una anemia macroc\u00edtica, las opciones 2 y 4 son opciones para identificar la enfermedad autoinmune pero lo que nos piden es que hacer para tratar la anemia por lo cual debemos de sospechar es que la anemia sea secundaria a la autoinmune y por tanto solicitar un test de coombs directo, ya que si es positivo la paciente en principio salvo extrema gravedad no va a ser transfundida y se debe tratar la causa primaria. Adem\u00e1s habla de instauraci\u00f3n r\u00e1pida, que en el caso de la vitamina B12 es m\u00e1s progresiva.", "type": "HEMATOLOGIA", "options": {"1": "Vitamina B12.", "2": "Anticuerpos antinucleares.", "3": "Test de Coombs directo.", "4": "Anticuerpos anti-DNA.", "5": null}, "correct_option": 3, "explanations": {"1": {"exist": false, "char_ranges": [], "word_ranges": [], "text": ""}, "2": {"exist": true, "char_ranges": [[0, 277]], "word_ranges": [[0, 46]], "text": "Comentan el caso de una mujer joven con cl\u00ednica de enfermedad sist\u00e9mica de probable origen autoinmune (lupus) que presenta una anemia macroc\u00edtica, las opciones 2 y 4 son opciones para identificar la enfermedad autoinmune pero lo que nos piden es que hacer para tratar la anemia"}, "3": {"exist": true, "char_ranges": [[290, 538]], "word_ranges": [[49, 94]], "text": "debemos de sospechar es que la anemia sea secundaria a la autoinmune y por tanto solicitar un test de coombs directo, ya que si es positivo la paciente en principio salvo extrema gravedad no va a ser transfundida y se debe tratar la causa primaria."}, "4": {"exist": true, "char_ranges": [[0, 277]], "word_ranges": [[0, 46]], "text": "Comentan el caso de una mujer joven con cl\u00ednica de enfermedad sist\u00e9mica de probable origen autoinmune (lupus) que presenta una anemia macroc\u00edtica, las opciones 2 y 4 son opciones para identificar la enfermedad autoinmune pero lo que nos piden es que hacer para tratar la anemia"}, "5": {"exist": false, "char_ranges": [], "word_ranges": [], "text": ""}}} +{"id": 117, "year": 2012, "question_id_specific": 112, "full_question": "Un var\u00f3n de 64 a\u00f1os consulta por fiebre, tos, disnea y dolor pleur\u00edtico derecho de 1 semana de evoluci\u00f3n. En la Rx de t\u00f3rax se aprecia un derrame pleural derecho loculado que ocupa dos terceras partes del hemit\u00f3rax. Durante la toracocentesis se extrae un l\u00edquido amarillento cuyo an\u00e1lisis muestra: leucocitos 15,000/uL, 92% de neutr\u00f3filos, glucosa 30 mg/dl, pH 7, lactato deshidrogenasa 3500 U/L, adenosina desaminasa 45 U/L y ausencia de g\u00e9rmenes en la tinci\u00f3n de GRAM. \u00bfCu\u00e1l es la siguiente acci\u00f3n m\u00e1s apropiada en este paciente?", "full_answer": "En esta pregunta no debe haber tampoco mucha duda (por cierto, que creo que es m\u00e1s de Neumolog\u00eda que m\u00eda, pero bueno, con esto de la fiebre mi amigo Emilienko se ha desviado), puesto que la cl\u00ednica orienta a un Derrame pleural paraneum\u00f3nico, las caracter\u00edsticas macrosc\u00f3picas del l\u00edquido as\u00ed lo parecen y el an\u00e1lisis del mismo nos muestra que adem\u00e1s el derrame est\u00e1 complicado, casi a pique de ser un empiema (LDH alt\u00edsima, celularidad con predominio de PMN, consumo de glucosa). Con ese pH adem\u00e1s, (y sabiendo que el derrame ya est\u00e1 loculado) el tubo endotor\u00e1cico est\u00e1 m\u00e1s que indicado.", "type": "INFECTOLOG\u00cdA", "options": {"1": "Antibioticoterapia intravenosa.", "2": "Antibioticoterapia intravenosa y repetir la toracocentesis diagn\u00f3stica en 24 horas.", "3": "Antibioticoterapia intravenosa y repetir la toracocentesis diagn\u00f3stica si no hay mejor\u00eda en 48 horas.", "4": "Antibioticoterapia intravenosa y realizar una toracocentesis evacuadora (terap\u00e9utica) si se a\u00edsla alg\u00fan g\u00e9rmen en el cultivo del l\u00edquido pleural.", "5": "Antibioticoterapia intravenosa y colocar un tubo o cat\u00e9ter tor\u00e1cico para drenar todo el l\u00edquido pleural."}, "correct_option": 5, "explanations": {"1": {"exist": false, "char_ranges": [], "word_ranges": [], "text": ""}, "2": {"exist": false, "char_ranges": [], "word_ranges": [], "text": ""}, "3": {"exist": false, "char_ranges": [], "word_ranges": [], "text": ""}, "4": {"exist": false, "char_ranges": [], "word_ranges": [], "text": ""}, "5": {"exist": true, "char_ranges": [[187, 587]], "word_ranges": [[35, 100]], "text": "la cl\u00ednica orienta a un Derrame pleural paraneum\u00f3nico, las caracter\u00edsticas macrosc\u00f3picas del l\u00edquido as\u00ed lo parecen y el an\u00e1lisis del mismo nos muestra que adem\u00e1s el derrame est\u00e1 complicado, casi a pique de ser un empiema (LDH alt\u00edsima, celularidad con predominio de PMN, consumo de glucosa). Con ese pH adem\u00e1s, (y sabiendo que el derrame ya est\u00e1 loculado) el tubo endotor\u00e1cico est\u00e1 m\u00e1s que indicado."}}} +{"id": 27, "year": 2011, "question_id_specific": 203, "full_question": "Un paciente de 52 a\u00f1os ingresa en el Hospital por neumon\u00eda grave. Con tratamiento antibi\u00f3tico adecuado mejora del cuadro respiratorio. Tras 4 d\u00edas de estancia en planta se complica su evoluci\u00f3n por la aparici\u00f3n de un proceso diarreico grave. \u00bfCu\u00e1l ser\u00eda el microorganismo m\u00e1s frecuentemente responsable de este cuadro?", "full_answer": "En este caso y, dado el tratamiento antibi\u00f3tico instaurado as\u00ed como la gravedad de la diarrea, parece que estemos ante una infecci\u00f3n por Clostridium. La opci\u00f3n 2 es poco cre\u00edble, la verdad, por mucho que los pacientes con NAC puedan asociar diarrea, es poco probable que una bacteria sea sensible a un antibi\u00f3tico al inicio de cuadro y deje de serlo a los 3 d\u00edas.", "type": "INFECCIOSAS", "options": {"1": "Salmonella ent\u00e9rica.", "2": "La propia bacteria causante de la neumon\u00eda que se ha hecho resistente al antibi\u00f3tico.", "3": "Campylobacter jejuni.", "4": "Clostridium difficile.", "5": "Yersinia enterocol\u00edtica."}, "correct_option": 4, "explanations": {"1": {"exist": false, "char_ranges": [], "word_ranges": [], "text": ""}, "2": {"exist": true, "char_ranges": [[150, 363]], "word_ranges": [[24, 65]], "text": "La opci\u00f3n 2 es poco cre\u00edble, la verdad, por mucho que los pacientes con NAC puedan asociar diarrea, es poco probable que una bacteria sea sensible a un antibi\u00f3tico al inicio de cuadro y deje de serlo a los 3 d\u00edas."}, "3": {"exist": false, "char_ranges": [], "word_ranges": [], "text": ""}, "4": {"exist": true, "char_ranges": [[16, 149]], "word_ranges": [[4, 24]], "text": "dado el tratamiento antibi\u00f3tico instaurado as\u00ed como la gravedad de la diarrea, parece que estemos ante una infecci\u00f3n por Clostridium."}, "5": {"exist": false, "char_ranges": [], "word_ranges": [], "text": ""}}} +{"id": 471, "year": 2020, "question_id_specific": 139, "full_question": "Ni\u00f1o de 7 a\u00f1os tra\u00eddo a urgencias tras caerse de un columpio sobre la mano derecha. No tiene antecedentes de inter\u00e9s. Presenta deformidad en dorso de tenedor de la mu\u00f1eca e impotencia funcional, con situaci\u00f3n neurovascular distal normal. \u00bfQu\u00e9 lesi\u00f3n espera encontrar en la radiograf\u00eda urgente que solicita?", "full_answer": "Nos est\u00e1n contando cl\u00ednica y deformidad a nivel de la mu\u00f1eca no a nivel del codo por lo que descartamos 1 y 2. Con la deformidad y dolor en radio distal esperamos una fractura a dicho nivel por lo que marcamos 3. Sin embargo, la veo impugnable dado que tambi\u00e9n podr\u00eda tener de forma concomitante una fractura en tallo verde de c\u00fabito aunque la lesi\u00f3n principal sea la del radio.", "type": "CIRUG\u00cdA ORTOP\u00c9DICA Y TRAUMATOLOG\u00cdA", "options": {"1": "Fractura \u2013 luxaci\u00f3n de Monteggia.", "2": "Fractura de la cabeza radial.", "3": "Epifisiolisis distal del radio.", "4": "Fractura en tallo verde de c\u00fabito.", "5": null}, "correct_option": 3, "explanations": {"1": {"exist": true, "char_ranges": [[0, 110]], "word_ranges": [[0, 23]], "text": "Nos est\u00e1n contando cl\u00ednica y deformidad a nivel de la mu\u00f1eca no a nivel del codo por lo que descartamos 1 y 2."}, "2": {"exist": true, "char_ranges": [[0, 110]], "word_ranges": [[0, 23]], "text": "Nos est\u00e1n contando cl\u00ednica y deformidad a nivel de la mu\u00f1eca no a nivel del codo por lo que descartamos 1 y 2."}, "3": {"exist": true, "char_ranges": [[111, 212]], "word_ranges": [[23, 42]], "text": "Con la deformidad y dolor en radio distal esperamos una fractura a dicho nivel por lo que marcamos 3."}, "4": {"exist": false, "char_ranges": [], "word_ranges": [], "text": ""}, "5": {"exist": false, "char_ranges": [], "word_ranges": [], "text": ""}}} +{"id": 327, "year": 2016, "question_id_specific": 81, "full_question": "Acude a la consulta una mujer de 30 a\u00f1os de edad refiriendo en los tres \u00faltimos meses ansiedad, p\u00e9rdida de unos 6 kg de peso y sensaci\u00f3n de \u201cnerviosismo\u201d En la exploraci\u00f3n fisica destaca taquicardia, hiperreflexia y ausencia de bocio En la anal\u00edtica realizada los valores de la TSH son < a 0.01 microU/mL, la T4 est\u00e1 elevada y los niveles de tiroglobulina descendidos. Al realizarle una gammagraf\u00eda se detecta una ausencia de captaci\u00f3n en la regi\u00f3n tiroidea. \u00bfCu\u00e1l le parece el diagn\u00f3stico m\u00e1s probable?:", "full_answer": "Respuesta 2 incorrecta: La enfermedad de Graves se caracteriza por hipertiroidismo y una o m\u00e1s de las siguientes caracter\u00edsticas: bocio, exoftalmos y mixedema pretibial. Respuesta 4 incorrecta: El s\u00edntoma m\u00e1s obvio de la tiroiditis subaguda es el dolor en el cuello. Algunas veces, el dolor puede extenderse a la mand\u00edbula o a los o\u00eddos. El agrandamiento doloroso de la gl\u00e1ndula tiroidea puede durar semanas o meses. Y suele presentar nivel bajo de la hormona estimulante de tiroides (TSH) en suero. Nivel alto de T4 libre en suero. Nivel alto de tiroglobulina en suero. Respuesta 4 incorrecta: Tumor muy raro.", "type": "ENDOCRINOLOG\u00cdA", "options": {"1": "Tirotoxicosis facticia.", "2": "Hipertiroidismo por Enfermedad de Graves.", "3": "Teratoma de ovario (estruma ov\u00e1rico).", "4": "Tiroiditis subaguda.", "5": null}, "correct_option": 1, "explanations": {"1": {"exist": false, "char_ranges": [], "word_ranges": [], "text": ""}, "2": {"exist": true, "char_ranges": [[0, 169]], "word_ranges": [[0, 24]], "text": "Respuesta 2 incorrecta: La enfermedad de Graves se caracteriza por hipertiroidismo y una o m\u00e1s de las siguientes caracter\u00edsticas: bocio, exoftalmos y mixedema pretibial."}, "3": {"exist": false, "char_ranges": [], "word_ranges": [], "text": ""}, "4": {"exist": true, "char_ranges": [[170, 337]], "word_ranges": [[24, 54]], "text": "Respuesta 4 incorrecta: El s\u00edntoma m\u00e1s obvio de la tiroiditis subaguda es el dolor en el cuello. Algunas veces, el dolor puede extenderse a la mand\u00edbula o a los o\u00eddos."}, "5": {"exist": false, "char_ranges": [], "word_ranges": [], "text": ""}}} +{"id": 450, "year": 2018, "question_id_specific": 154, "full_question": "Un hombre presenta en la exploraci\u00f3n neurol\u00f3gica un d\u00e9ficit sensitivo termoalg\u00e9sico en la pierna izquierda asociado a una p\u00e9rdida de sensibilidad vibratoria y posicional en la pierna derecha. Al mismo tiempo presenta torpeza y p\u00e9rdida de fuerza distal en la pierna derecha y un reflejo cut\u00e1neo plantar derecho en extensi\u00f3n. \u00bfCu\u00e1l de las siguientes afirmaciones es correcta?:", "full_answer": "Describen un cuadro de hemisecci\u00f3n o Brown-Sequard t\u00edpico. Afectaci\u00f3n motora y cordonal posterior ipsilateral con afectaci\u00f3n contralateral de dolor y temperatura.", "type": "NEUROCIRUG\u00cdA", "options": {"1": "Es un s\u00edndrome centromedular tipo siringomielia.", "2": "Es un s\u00edndrome hemimedular.", "3": "Es un patr\u00f3n de lesi\u00f3n medular transversa.", "4": "Es un patr\u00f3n de lesi\u00f3n bulbar lateral.", "5": null}, "correct_option": 2, "explanations": {"1": {"exist": false, "char_ranges": [], "word_ranges": [], "text": ""}, "2": {"exist": true, "char_ranges": [[0, 162]], "word_ranges": [[0, 21]], "text": "Describen un cuadro de hemisecci\u00f3n o Brown-Sequard t\u00edpico. Afectaci\u00f3n motora y cordonal posterior ipsilateral con afectaci\u00f3n contralateral de dolor y temperatura."}, "3": {"exist": false, "char_ranges": [], "word_ranges": [], "text": ""}, "4": {"exist": false, "char_ranges": [], "word_ranges": [], "text": ""}, "5": {"exist": false, "char_ranges": [], "word_ranges": [], "text": ""}}} +{"id": 265, "year": 2014, "question_id_specific": 137, "full_question": "Hombre de 65 a\u00f1os con antecedente de neoplasia de p\u00e1ncreas en curso de quimioterapia. Consulta en Urgencias por dolor y edema de todo el miembro inferior desde ingle. \u00bfQu\u00e9 prueba diagn\u00f3stica es m\u00e1s coste-efectiva para confirmar la sospecha diagn\u00f3stica?", "full_answer": "La ecografia dopler es la prueba mas precisa y que permite posteriormente controlar la evolucion de la enfermedad de forma no invasiva.", "type": "NEUMOLOG\u00cdA", "options": {"1": "D\u00edmero D.", "2": "Resonancia magn\u00e9tica.", "3": "Flebograf\u00eda.", "4": "Ecograf\u00eda doppler venosa.", "5": "TAC helicoidal."}, "correct_option": 4, "explanations": {"1": {"exist": false, "char_ranges": [], "word_ranges": [], "text": ""}, "2": {"exist": false, "char_ranges": [], "word_ranges": [], "text": ""}, "3": {"exist": false, "char_ranges": [], "word_ranges": [], "text": ""}, "4": {"exist": true, "char_ranges": [[0, 135]], "word_ranges": [[0, 22]], "text": "La ecografia dopler es la prueba mas precisa y que permite posteriormente controlar la evolucion de la enfermedad de forma no invasiva."}, "5": {"exist": false, "char_ranges": [], "word_ranges": [], "text": ""}}} +{"id": 189, "year": 2013, "question_id_specific": 71, "full_question": "Ni\u00f1o de 10 a\u00f1os con episodios breves de distracciones (< 1 minuto) en los que no responde a llamadas y parpadea. Un EEG muestra descargas punta-onda a 3 ciclos por segundo. El tratamiento electivo de primera l\u00ednea lo har\u00eda con:", "full_answer": "Las crisis descritas y el EEG corresponden a crisis de ausencia t\u00edpicas y el tratamiento de elecci\u00f3n de entrada es el \u00e1cido valproico.", "type": "PEDIATR\u00cdA", "options": {"1": "Valproato.", "2": "Carbamacepina.", "3": "Fenito\u00edna.", "4": "Gabapentina.", "5": "Clonazepam."}, "correct_option": 1, "explanations": {"1": {"exist": true, "char_ranges": [[0, 134]], "word_ranges": [[0, 23]], "text": "Las crisis descritas y el EEG corresponden a crisis de ausencia t\u00edpicas y el tratamiento de elecci\u00f3n de entrada es el \u00e1cido valproico."}, "2": {"exist": false, "char_ranges": [], "word_ranges": [], "text": ""}, "3": {"exist": false, "char_ranges": [], "word_ranges": [], "text": ""}, "4": {"exist": false, "char_ranges": [], "word_ranges": [], "text": ""}, "5": {"exist": false, "char_ranges": [], "word_ranges": [], "text": ""}}} +{"id": 508, "year": 2021, "question_id_specific": 148, "full_question": "Mujer de 72 a\u00f1os sin antecedentes de inter\u00e9s. Tras un c\u00f3lico renal complicado se encuentra casualmente en la TC abdominal una lesi\u00f3n qu\u00edstica en cola de p\u00e1ncreas de 2 cm, junto con m\u00faltiples lesiones qu\u00edsticas renales bilaterales. La ultrasonograf\u00eda endosc\u00f3pica muestra una lesi\u00f3n poliqu\u00edstica formada por m\u00faltiples ves\u00edculas con calcificaci\u00f3n central en cola de p\u00e1ncreas sin conexi\u00f3n con el conducto de Wirsung. El an\u00e1lisis del l\u00edquido es compatible con un cistoadenoma seroso. De las siguientes, \u00bfcu\u00e1l es la actitud m\u00e1s correcta con respecto al manejo de esta paciente?:", "full_answer": "El cistoadenoma seroso de p\u00e1ncreas es una entidad benigna frecuentemente poliqu\u00edstica (tambi\u00e9n llamado adenoma microqu\u00edstico) formado por c\u00e9lulas productoras de col\u00e1geno (no mucinosas). Habitualmente es asintom\u00e1tica y se descubre como hallazgo incidental. Dada su naturaleza benigna no se recomienda la cirug\u00eda, reserv\u00e1ndose \u00fanicamente en aquellos sintom\u00e1ticos o en los que la naturaleza de la lesi\u00f3n queda en entredicho despu\u00e9s de un estudio completo con TC, RM y ecoendoscopia con biopsia. Frecuentemente es espor\u00e1dico, pero se puede asociar con el s\u00edndrome de Von-Hippel-Lindau.", "type": "DIGESTIVO", "options": {"1": "Resecci\u00f3n quir\u00fargica (pancreatectom\u00eda corporocaudal).", "2": "Punci\u00f3n guiada mediante ultrasonograf\u00eda endosc\u00f3pica y etanolizaci\u00f3n de la misma.", "3": "Seguimiento de la lesi\u00f3n mediante RM.", "4": "Trasplante de ri\u00f1\u00f3n-p\u00e1ncreas.", "5": null}, "correct_option": 3, "explanations": {"1": {"exist": true, "char_ranges": [[256, 310]], "word_ranges": [[32, 41]], "text": "Dada su naturaleza benigna no se recomienda la cirug\u00eda,"}, "2": {"exist": true, "char_ranges": [[256, 310]], "word_ranges": [[32, 41]], "text": "Dada su naturaleza benigna no se recomienda la cirug\u00eda,"}, "3": {"exist": true, "char_ranges": [[256, 310]], "word_ranges": [[32, 41]], "text": "Dada su naturaleza benigna no se recomienda la cirug\u00eda,"}, "4": {"exist": true, "char_ranges": [[256, 310]], "word_ranges": [[32, 41]], "text": "Dada su naturaleza benigna no se recomienda la cirug\u00eda,"}, "5": {"exist": false, "char_ranges": [], "word_ranges": [], "text": ""}}} +{"id": 151, "year": 2012, "question_id_specific": 70, "full_question": "Una paciente de 25 a\u00f1os con antecedentes de rash cut\u00e1neo tras exposici\u00f3n solar y poliartritis ocasionales en articulaciones de las manos que controla con antiiflamatorios no esteroideos presenta desde hace 15 d\u00edas malestar general, sensaci\u00f3n progresiva de debilidad generalizada y palidez. En la anal\u00edtica destaca una Hemoglobina de 7 g/dL, VCM de 108 mm/h, 150000 plaquetas/mm3, 3000 leucocitos/mm3, LDH elevada, haptoglobina indectectable. En el caso de elegir una sola prueba diagn\u00f3stica indique cu\u00e1l de las siguientes determinaciones debe realizarse en primer lugar:", "full_answer": "El cuadro cl\u00ednico y biol\u00f3gico de esta paciente hace sospechar una anemia hemol\u00edtica en el contexto de un lupus eritematoso sist\u00e9mico. Si hay que escoger una sola prueba diagn\u00f3stica, y que \u00e9sta sea la primera, nos interesa confirmar la presencia de una anemia hemol\u00edtica autoinmune con el test de Coombs directo para iniciar tratamiento. El resto de pruebas pueden esperar...", "type": "REUMATOLOG\u00cdA", "options": {"1": "Test de Coombs directo.", "2": "Anticuerpos antinucleares (ANA).", "3": "Vitamina B12.", "4": "Ferritina.", "5": "\u00c1cido f\u00f3lico."}, "correct_option": 1, "explanations": {"1": {"exist": true, "char_ranges": [[0, 336]], "word_ranges": [[0, 54]], "text": "El cuadro cl\u00ednico y biol\u00f3gico de esta paciente hace sospechar una anemia hemol\u00edtica en el contexto de un lupus eritematoso sist\u00e9mico. Si hay que escoger una sola prueba diagn\u00f3stica, y que \u00e9sta sea la primera, nos interesa confirmar la presencia de una anemia hemol\u00edtica autoinmune con el test de Coombs directo para iniciar tratamiento."}, "2": {"exist": true, "char_ranges": [[337, 374]], "word_ranges": [[54, 60]], "text": "El resto de pruebas pueden esperar..."}, "3": {"exist": true, "char_ranges": [[337, 374]], "word_ranges": [[54, 60]], "text": "El resto de pruebas pueden esperar..."}, "4": {"exist": true, "char_ranges": [[337, 374]], "word_ranges": [[54, 60]], "text": "El resto de pruebas pueden esperar..."}, "5": {"exist": true, "char_ranges": [[337, 374]], "word_ranges": [[54, 60]], "text": "El resto de pruebas pueden esperar..."}}} +{"id": 404, "year": 2016, "question_id_specific": 139, "full_question": "Mujer de 70 a\u00f1os con antecedentes de anorexia, p\u00e9rdida de peso, molestias en la musculatura y articulaciones proximales m\u00e1s dolorimiento en la regi\u00f3n temporo-mandibular que acude al servicio de urgencias por p\u00e9rdida de visi\u00f3n unilateral (movimiento de manos), de aparici\u00f3n brusca e indolora (defecto pupilar aferente). \u00bfQu\u00e9 prueba solicitar\u00eda en primer lugar con fines diagn\u00f3sticos?:", "full_answer": "Se trata de una mujer con p\u00e9rdida brusca y indolora y grave (s\u00f3lo ve el movimiento de las manos) de la visi\u00f3n de un ojo. No nos da mucha informaci\u00f3n de la exploraci\u00f3n ocular, s\u00f3lo que hay un defecto pupilar aferente. Nos tenemos que ayudar de los s\u00edntomas sist\u00e9micos para diagnosticar el problema. Se trata de una mujer de edad avanzada con s\u00edntomas t\u00edpicos de polimialgia reum\u00e1tica, y el dolor en la regi\u00f3n temporomandibular es muy sugerente. Se trata de una arteritis de c\u00e9lulas gigantes o arteritis de la temporal. La manifestaci\u00f3n ocular m\u00e1s habitual es la neuropat\u00eda \u00f3ptica isqu\u00e9mica anterior. En el fondo de ojo seguramente ver\u00edamos un edema de papila de ese ojo, de coloraci\u00f3n blanco yeso. Es muy caracter\u00edstico de estas neuropat\u00edas \u00f3pticas isqu\u00e9micas tan severas el defecto pupilar aferente. Sospechando una arter\u00edtis de c\u00e9lulas gigantes, hay tres marcadores anal\u00edticos que est\u00e1n caracter\u00edsticamente altos: la PCR, la VSG y el recuento plaquetario. En primer lugar solicitaremos alguno de estos par\u00e1metros y ponerle tratamiento muchas veces sin esperar confirmaci\u00f3n por la biopsia. Por lo tanto, la opci\u00f3n 2 es la correcta.", "type": "OFTALMOLOG\u00cdA (ECT\u00d3PICO)", "options": {"1": "Punci\u00f3n lumbar.", "2": "Prote\u00edna C reactiva.", "3": "Angio Resonancia Magn\u00e9tica.", "4": "Ecograf\u00eda carot\u00eddea.", "5": null}, "correct_option": 2, "explanations": {"1": {"exist": false, "char_ranges": [], "word_ranges": [], "text": ""}, "2": {"exist": true, "char_ranges": [[800, 956]], "word_ranges": [[133, 155]], "text": "Sospechando una arter\u00edtis de c\u00e9lulas gigantes, hay tres marcadores anal\u00edticos que est\u00e1n caracter\u00edsticamente altos: la PCR, la VSG y el recuento plaquetario."}, "3": {"exist": false, "char_ranges": [], "word_ranges": [], "text": ""}, "4": {"exist": false, "char_ranges": [], "word_ranges": [], "text": ""}, "5": {"exist": false, "char_ranges": [], "word_ranges": [], "text": ""}}} +{"id": 215, "year": 2014, "question_id_specific": 106, "full_question": "Paciente de 68 a\u00f1os que consulta por edemas y astenia. En la anal\u00edtica realizada se constata creatinina de 5 mg/dl, hemoglobina de 10 gr/dl y una marcada hipogammaglobulinemia en suero a expensas de IgG, IgA e IgM. Un an\u00e1lisis de orina revela la presencia de cadenas ligeras kappa. \u00bfCu\u00e1l es su sospecha diagn\u00f3stica?", "full_answer": "Esta pregunta es mixta hemato-nefro. Es complicada, as\u00ed que vamos a ir descartando opciones una por una. La opci\u00f3n 2, s\u00edndrome nefr\u00f3tico, aparte de ser amplia e inespec\u00edfica, no ser\u00eda valorable, ya que no nos dan la cuantificaci\u00f3n de prote\u00ednas en orina (s\u00f3lo nos dicen que el an\u00e1lisis de orina revela la presencia de cadenas ligeras kappa, no la cantidad). La opci\u00f3n 3 ser\u00eda poco probable; en la amiloidosis, las cadenas ligeras que suelen depositarse son del tipo lambda. La opci\u00f3n 4 parece tambi\u00e9n poco probable, ya que en un mieloma IgA esperar\u00edamos un pico monoclonal de dicha Ig, que en este caso no aparece. La duda que me surge es entre la 1 y la 5; yo optar\u00eda por la 1, porque esta entidad cumple con las caracter\u00edsticas descritas (ausencia de pico monoclonal en la mayor\u00eda de los casos, fracaso renal agudo y presencia de cadenas kappa en orina), siendo el mieloma de cadenas ligeras una entidad algo m\u00e1s amplia, pero igualmente se podr\u00eda dar como v\u00e1lida la opci\u00f3n 5.", "type": "NEFROLOG\u00cdA", "options": {"1": "Enfermedad por dep\u00f3sito de cadenas ligeras kappa.", "2": "S\u00edndrome nefr\u00f3tico.", "3": "Amiloidosis.", "4": "Mieloma IgA con proteinuria Bence-Jones.", "5": "Mieloma de cadenas ligeras."}, "correct_option": 1, "explanations": {"1": {"exist": true, "char_ranges": [[657, 854]], "word_ranges": [[117, 150]], "text": "yo optar\u00eda por la 1, porque esta entidad cumple con las caracter\u00edsticas descritas (ausencia de pico monoclonal en la mayor\u00eda de los casos, fracaso renal agudo y presencia de cadenas kappa en orina),"}, "2": {"exist": true, "char_ranges": [[105, 356]], "word_ranges": [[17, 60]], "text": "La opci\u00f3n 2, s\u00edndrome nefr\u00f3tico, aparte de ser amplia e inespec\u00edfica, no ser\u00eda valorable, ya que no nos dan la cuantificaci\u00f3n de prote\u00ednas en orina (s\u00f3lo nos dicen que el an\u00e1lisis de orina revela la presencia de cadenas ligeras kappa, no la cantidad)."}, "3": {"exist": true, "char_ranges": [[357, 472]], "word_ranges": [[60, 79]], "text": "La opci\u00f3n 3 ser\u00eda poco probable; en la amiloidosis, las cadenas ligeras que suelen depositarse son del tipo lambda."}, "4": {"exist": true, "char_ranges": [[473, 613]], "word_ranges": [[79, 105]], "text": "La opci\u00f3n 4 parece tambi\u00e9n poco probable, ya que en un mieloma IgA esperar\u00edamos un pico monoclonal de dicha Ig, que en este caso no aparece."}, "5": {"exist": true, "char_ranges": [[856, 920]], "word_ranges": [[150, 161]], "text": "siendo el mieloma de cadenas ligeras una entidad algo m\u00e1s amplia,"}}} +{"id": 264, "year": 2014, "question_id_specific": 135, "full_question": "Una mujer de 24 a\u00f1os es encontrada tirada en la calle por unos transe\u00fantes. Al llegar el quipo de emergencias la encontraron con una saturaci\u00f3n de ox\u00edgeno del 88% respirando aire ambiente y al examen pupilas puntiformes. Se la traslada a la urgencia del hospital m\u00e1s cercano, donde la gasometr\u00eda arterial basal muestra: pH 7,25, PaC02 60 mmHg, Pa02 58 mmHg, bicarbonato de 26 mEq/1 y exceso de bases de -1. En sangre el sodio es 137 mEq/1 y el cloruro 100 mEq/1. Desde cl punto de vista gasom\u00e9trico la paciente tiene:", "full_answer": "La acidosis con PCO2 elevada, hipoxemia y bicarbonato normal, sugiere una acidosis respiratoria aguda de instauracion r\u00e1pida probablemente por intoxicacion por drogas o farmacos.", "type": "NEUMOLOG\u00cdA", "options": {"1": "Insuficiencia respiratoria parcial.", "2": "Acidosis metab\u00f3lica.", "3": "Acidosis respiratoria pura.", "4": "Alcalosis respiratoria por falta de cloro.", "5": "La gasometr\u00eda solo puede ser de sangre venosa."}, "correct_option": 3, "explanations": {"1": {"exist": false, "char_ranges": [], "word_ranges": [], "text": ""}, "2": {"exist": false, "char_ranges": [], "word_ranges": [], "text": ""}, "3": {"exist": true, "char_ranges": [[0, 178]], "word_ranges": [[0, 24]], "text": "La acidosis con PCO2 elevada, hipoxemia y bicarbonato normal, sugiere una acidosis respiratoria aguda de instauracion r\u00e1pida probablemente por intoxicacion por drogas o farmacos."}, "4": {"exist": false, "char_ranges": [], "word_ranges": [], "text": ""}, "5": {"exist": false, "char_ranges": [], "word_ranges": [], "text": ""}}} +{"id": 594, "year": 2022, "question_id_specific": 80, "full_question": "Lactante de 2 meses de edad con catarro de v\u00edas altas de 3 d\u00edas de evoluci\u00f3n, que comienza con dificultad respiratoria moderada y auscultaci\u00f3n pulmonar con sibilancias espiratorias. La saturaci\u00f3n de ox\u00edgeno es de 89 %. Se a\u00edsla virus respiratorio sincitial en el exudado nasofar\u00edngeo. \u00bfCu\u00e1l de los siguientes tratamientos considera m\u00e1s indicado para este cuadro?:", "full_answer": "Nos presentan el caso de una bronquiolitis VRS positivo. Seg\u00fan las recomendaciones actuales, lo indicado ser\u00eda iniciar oxigenoterapia suplementaria.", "type": "PEDIATR\u00cdA", "options": {"1": "Rivabirina oral.", "2": "Salbutamol nebulizado.", "3": "Ox\u00edgeno suplementario.", "4": "Corticoides intravenosos.", "5": null}, "correct_option": 3, "explanations": {"1": {"exist": false, "char_ranges": [], "word_ranges": [], "text": ""}, "2": {"exist": false, "char_ranges": [], "word_ranges": [], "text": ""}, "3": {"exist": true, "char_ranges": [[0, 148]], "word_ranges": [[0, 19]], "text": "Nos presentan el caso de una bronquiolitis VRS positivo. Seg\u00fan las recomendaciones actuales, lo indicado ser\u00eda iniciar oxigenoterapia suplementaria."}, "4": {"exist": false, "char_ranges": [], "word_ranges": [], "text": ""}, "5": {"exist": false, "char_ranges": [], "word_ranges": [], "text": ""}}} +{"id": 236, "year": 2014, "question_id_specific": 108, "full_question": "Una paciente de 67 a\u00f1os en tratamiento con ticlopidina acude a urgencias con cefalea, astenia y petequias en extremidades inferiores. En la anal\u00edtica presenta hemoglobina 8,2 g/dl, VCM 100 fl, plaquetas 25000/ul y leucocitos 7500/ul con formula normal. La cifra reticulocitos est\u00e1 elevada y el frotis de sangre se observa numerosos esquistocitos. Los estudios de coagulaci\u00f3n (TTPA, TP y fibrin\u00f3geno) son normales. En la bioqu\u00edmica destaca LDH 2700 UI/l y bilirrubina 2,6 mg/dl. \u00bfCu\u00e1l es el diagn\u00f3stico m\u00e1s probable?", "full_answer": "T\u00edpico de la ticlopidina. No solo trombocitopenia sino tambi\u00e9n cefalea, esquistocitos, LDH alta y bilirrubina alta.", "type": "HEMATOLOG\u00cdA", "options": {"1": "Purpura trombocitop\u00e9nica autoinmune.", "2": "Purpura tromb\u00f3tica trombocitop\u00e9nica.", "3": "Aplasia medular.", "4": "Trombopenia inducida por f\u00e1rmacos.", "5": "Coagulaci\u00f3n intravascular diseminada."}, "correct_option": 2, "explanations": {"1": {"exist": false, "char_ranges": [], "word_ranges": [], "text": ""}, "2": {"exist": true, "char_ranges": [[0, 115]], "word_ranges": [[0, 16]], "text": "T\u00edpico de la ticlopidina. No solo trombocitopenia sino tambi\u00e9n cefalea, esquistocitos, LDH alta y bilirrubina alta."}, "3": {"exist": false, "char_ranges": [], "word_ranges": [], "text": ""}, "4": {"exist": false, "char_ranges": [], "word_ranges": [], "text": ""}, "5": {"exist": false, "char_ranges": [], "word_ranges": [], "text": ""}}}